CBSE 2023 MATHS (BASIC) Board Questions Papers
CBSE 2023 MATHS (BASIC) Board Questions Papers
J{UV (~w{Z`mXr)
MATHEMATICS (BASIC)
*
:3 : 80
Time allowed : 3 hours Maximum Marks : 80
NOTE :
(i) - 27
Please check that this question paper contains 27 printed pages.
(ii) - - -
-
Q.P. Code given on the right hand side of the question paper should be written on the title
page of the answer-book by the candidate.
(iii) - 38
Please check that this question paper contains 38 questions.
(iv) -
Please write down the serial number of the question in the answer-book before
attempting it.
(v) - 15 -
10.15 10.15 10.30 -
-
15 minute time has been allotted to read this question paper. The question paper will be
distributed at 10.15 a.m. From 10.15 a.m. to 10.30 a.m., the students will read the
question paper only and will not write any answer on the answer-book during this period.
SECTION A
(a) 36
(b) 6
(c) 36
(d) 6
(a) 30 (b) 15
(c) 45 (d) 10
7
3. If cosec A = , then value of tan A . cos A is :
5
7 2 6
(a) (b)
5 5
24 5
(c) (d)
49 7
20 5
30 13
40 15
50 16
(a) 30 40 (b ) 40 50
(c) 20 30 (d) 10 20
a2 b2
(c) (d) a2 + b2
2 2
a b
(a) 2 (r 21 r 22 ) (b) (r 21 r 22 )
(c) 3 r 22 r 21 (d) 3 r 21 r 22
(a) c 2 units
(b) 2c units
(c) 2 c units
(d) c units
12. Two cubes each of 5 cm edge are joined end to end. The surface area of
the resulting cuboid is :
13. A die is rolled once. The probability that the obtained number is more
than 5, is :
5 1
(a) (b)
6 6
2 1
(c) (d)
3 3
14. If the mean and the mode of a distribution are 17 and 20 respectively,
then the median of the distribution, using empirical formula, is :
31
(a) 20 (b)
3
(c) 18 (d) 17
r2 1 2
(a) r
2 2
2 r 1 2
(b) r
4 2
r2 1 2
(c) r
4 2
2 r
(d) r 2 sin 90
4
18. At some time of the day, the height and length of the shadow of a man
:
(a) 30 (b) 45
(c) 60 (d) 90
(a) Both Assertion (A) and Reason (R) are true and Reason (R) is the
correct explanation of the Assertion (A).
(b) Both Assertion (A) and Reason (R) are true, but Reason (R) is not
the correct explanation of the Assertion (A).
19. Assertion (A) : PA and PB are tangents to the circle centred at O and
OPA = 30 . Then, PAB is an equilateral triangle.
This section comprises very short answer (VSA) type questions of 2 marks each.
21. Find the ratio in which a line segment joining points (1, 4) and (6, 5) is
divided by the x-axis.
n
22. (a) Show that 8 can never end with the digit 0 for any natural
number n.
OR
(b) Find LCM and HCF of 96 and 160, using prime factorisation
method.
tan B tan A
tan (B A) =
1 tan A tan B
OR
(b) Evaluate :
1
sin2 60 2 cos2 45 + cosec2 30
2
25. Find the centre and radius of a circle having end points of its diameter as
(3, 10) and (1, 4).
This section comprises short answer (SA) type questions of 3 marks each.
BC BE
26. (a) In the given figure, = and ABD = ACD. Show that
BD AC
ABD EBC.
OR
(b) In the given figure, ABC and AED are two right triangles, right
angled at B and E respectively. Prove that :
(ii) AB AD = AC AE
27. Find the zeroes of the polynomial x2 11x + 18 and verify the
relationship between the zeroes and the coefficients.
30. Sum of the areas of two squares is 468 m2. If the difference between their
sides is 6 m, then find the sides of the two squares.
31. In the given figure, two concentric circles are shown, centred at O. The
radii of the circles are OA = 3 cm and OB = 6 cm.
SECTION D
This section comprises long answer (LA) type questions of 5 marks each.
34. If AD and PM are medians of triangles ABC and PQR respectively, where
AB AD
ABC PQR, prove that = .
PQ PM
35. (a) The following table gives production yield per hectare of wheat of
100 farms of a village :
Production yield Number of
(in kg/ha) farms
50 55 2
55 60 8
60 65 12
65 70 24
70 75 38
75 80 16
OR
(b) Find the mean and the mode of the data given below :
Weight Number of
(in kg) students
40 45 5
45 50 11
50 55 20
55 60 24
60 65 28
65 70 12
37. A fashion designer is designing a fabric pattern. In each row, there are
some shaded squares and unshaded triangles.
(iii) (a) If each shaded square is of side 2 cm, then find the shaded
OR
(iii) (b)
number of rows. Hence, find S10. 2
38. Totem poles are made from large trees. These poles are carved with
symbols or figures and mostly found in western Canada and
northwestern United States.
In the given picture, two such poles of equal heights are standing 28 m
apart. From a point somewhere between them in the same line, the
angles of elevation of the top of the two poles are 60 and 30
respectively.
OR
(ii) (b) If the distances of the top of the poles from the point of
observation are taken as p and q, then find a relation
between p and q. 2
(iii) Find the location of the point of observation. 1
SECTION A
430/C/1 3 P.T.O
4
430/C/1 4 P.T.O
9
10
Ans (b) p2 = - 4q 1
11
12
430/C/1 5 P.T.O
14
Ans (c) 18 1
15
Ans 𝜋𝑟 2 1 1
(c) − 𝑟2
4 2
16
17
Ans (c) 2 : 5 1
18
430/C/1 6 P.T.O
19
Ans (a) Both Assertion (A) and Reason (R) are true and Reason (R) is the 1
correct explanation of the Assertion (A).
20
Ans (b) Both Assertion (A) and Reason (R) are true, but Reason (R) is not the 1
correct explanation of the Assertion (A).
430/C/1 7 P.T.O
SECTION B
21
6k 1 5k 4
Point P is ,
k 1 k 1
P lies on x- axis so y = 0
5𝑘−4
=0 1+ ½
𝑘+1
k= 4/5 ½
Soln 8n = (2 2 2)n 1
For a number to end with zero it should have both 2 and 5 as its prime
1
factors but 8n has only 2 as its prime factor. So, it can not end with zero.
22(b)
1⁄
Soln 96 = 2 2 2 2 2 3 2
160 = 2 2 2 2 2 5 1⁄
2
HCF = 32 1⁄
2
LCM = 480 1⁄
2
23
1
ii) P (at most 2 heads) = 7/8
24(a)
430/C/1 8 P.T.O
Soln LHS= tan (600 - 300) = tan 300 = 1/√3 1⁄
2
Soln 1 3 2
1 1 2
2
2 2 2
2
1⁄
= 7/4 2
25
Soln 3 1 10 4 1
Centre = , = (2, -3)
2 2
Radius = 2 12 3 4 2 = √50 = 5√2 units. 1
SECTION C
26(a)
𝐵𝐶 𝐵𝐸 𝐵𝐶 𝐵𝐸
= = 1
𝐵𝐷 𝐴𝐶 𝐵𝐷 𝐵𝐴
430/C/1 9 P.T.O
∠B = ∠B (common) 1
27
18 𝑐𝑜𝑛𝑠𝑡𝑎𝑛𝑡 𝑡𝑒𝑟𝑚
Product of zeroes = 9 2 = = 𝑐𝑜𝑒𝑓𝑓𝑖𝑐𝑖𝑒𝑛𝑡 𝑜𝑓 𝑥 2 1⁄
1 2
28
430/C/1 10 P.T.O
Soln sin cos
1
LHS cos sin
cos sin
1 1
sin cos
sin
2
cos 2
sin cos cos cos sin sin
sin 3 cos3
1
sin cos (sin cos )
sin 2 cos 2 sin cos
sin cos ½
sec cos ec 1 ½
= RHS
29(a)
430/C/1 11 P.T.O
31
Soln Let R and r be the radii of the outer and inner circles.
θ2πR θ2πr
Perimeter of the shaded region = + + 2(R − r)
360 360
300×2×3.14
= × (6 + 3) + 2 × (6 − 3) 2
360
= 47.10 + 6 = 53.1 cm 1
SECTION D
32(a)
Soln
32(b)
Soln Let the cost of one chair be ₹ x and that of one table be ₹ y
5x + y = 1750 ---------------(i) 1
430/C/1 12 P.T.O
4x + 3y = 1950 ---------------(ii) 1
Solving (i) and (ii) and getting x = 300 and y =250 1+1
Total cost of 10 chairs and 10 tables = 10 (300 + 250) = ₹ 5500 1
33
Soln
1 for figure
Given : TP and TQ are two tangents drawn from an external point T to the
circle C(O,r) 1
To prove: TP = TQ
Soln
1 for figure
ΔABC ∼ ΔPQR
1⁄
⇒ ∠ABC = ∠PQR (corresponding angles of similar triangles) --------- (i) 2
430/C/1 13 P.T.O
⇒ AB/PQ = BD/QM (D and M are mid points of BC and QR) ------------ (ii) 1
In ΔABD and ΔPQM,
∠ABD = ∠PQM (from i)
1½
AB/PQ = BD/QM (from ii)
⇒ ΔABD ∼ ΔPQM [SAS similarity criterion]
⇒ AB/PQ = AD/PM (Corresponding sides of similar triangles are proportional) 1
Hence proved.
35(a)
Soln xi – 67·5
Class xi fi ui = fiui cf
5
50 – 55 52·5 2 –3 –6 2
55 – 60 57·5 8 –2 – 16 10
60 – 65 62·5 12 -1 -12 22
65 – 70 67·5 = a 24 0 0 46 2
for correct
70 – 75 72·5 38 1 38 84 table
75 – 80 77·5 16 2 32 100
100 36
∑𝑓𝑖 𝑢𝑖
Mean = a + ×h
∑𝑓𝑖
𝟏
36 1+𝟐
= 67·5 + ( × 5) = 69·3
100
430/C/1 14 P.T.O
N
– cf
Median = l + 2 ×h
f
𝟏
50 46 1+𝟐
= 70 + 5 70.5
38
35(b)
𝑥𝑖 – 57·5
Soln Class xi fi ui = fiui
5
40 – 45 42·5 5 –3 – 15
𝟏
45 – 50 47·5 11 –2 – 22 1𝟐
50 – 55 52·5 20 -1 -20
55 – 60 57·5= a 24 0 0 For correct
table
60 – 65 62·5 28 1 28
65 – 70 67·5 12 2 24
100 -5
∑𝑓𝑖 𝑢𝑖
Mean = a + ×h
∑𝑓𝑖
5 1+ ½
= 57.5 + 5 = 57.25
100
f1 – f0
Mode = l + × h
2f1 – f0 – f2
𝟐𝟖−𝟐𝟒
= 60 + 5 = 61 1½ +½
𝟐(𝟐𝟖)−𝟐𝟒−𝟏𝟐
SECTION E
430/C/1 15 P.T.O
36
430/C/1 16 P.T.O
37
Soln (i) A.P. for the number of squares in each row is 1, 3, 5, 7, 9…. 1
(ii) A.P. for the number of triangles in each row is 2, 6, 10, 14…. 1
(iii) (a) Area of each square = 2 2 = 4 cm2 1⁄
2
15
Number of squares in 15 rows = 2 14 2 = 225 1
2
Shaded area = 225 4 =900 cm2 1⁄
2
OR
n
(b) Sn 4 (n 1)4 2n 2 1
2
S10 2 102 = 200 1
430/C/1 17 P.T.O
38
Soln (i) Let AB and CD be the 2 poles and M be a point somewhere between their bases
in the same line.
1
for correct
figure
(ii) (a)
430/C/1 18 P.T.O
h
tan 600 hx 3 ½
x
tan 300
h
h
28 x ½
28 x 3
h 7 3 m 1
OR
h q
sin 300 h 1⁄
q 2 2
p 3 q
q 3p 1
2 2
7 3 ½
tan 600 x 7m AM
(iii) x
MC 28 x 21 m ½
430/C/1 19 P.T.O
Series WX1YZ/C SET~2
J{UV (~w{Z`mXr)
MATHEMATICS (BASIC)
*
:3 : 80
Time allowed : 3 hours Maximum Marks : 80
NOTE :
(i) - 27
Please check that this question paper contains 27 printed pages.
(ii) - - -
-
Q.P. Code given on the right hand side of the question paper should be written on the title
page of the answer-book by the candidate.
(iii) - 38
Please check that this question paper contains 38 questions.
(iv) -
Please write down the serial number of the question in the answer-book before
attempting it.
(v) - 15 -
10.15 10.15 10.30 -
-
15 minute time has been allotted to read this question paper. The question paper will be
distributed at 10.15 a.m. From 10.15 a.m. to 10.30 a.m., the students will read the
question paper only and will not write any answer on the answer-book during this period.
SECTION A
1. Two cubes each of 5 cm edge are joined end to end. The surface area of
the resulting cuboid is :
(a) 200 cm2
(b) 300 cm2
(c) 125 cm2
(d) 250 cm2
430/C/2 JJJJ Page 3 P.T.O.
2. Two coins are tossed together. The probability of getting at least one tail,
is :
3 1
(a) (b)
4 4
1
(c) (d) 1
2
(c) 18 (d) 17
r2 1 2
(a) r
2 2
2 r 1 2
(b) r
4 2
r2 1 2
(c) r
4 2
2 r
(d) r 2 sin 90
4
5. At some time of the day, the height and length of the shadow of a man
:
(a) 30 (b) 45
(c) 60 (d) 90
(a) 36
(b) 6
(c) 36
(d) 6
(a) 30 (b) 15
(c) 45 (d) 10
430/C/2 JJJJ Page 7 P.T.O.
5
10. If cos A = , then value of cot A . sin A is :
8
5 5
(a) (b)
39 8
8 8
(c) (d)
39 5
11. For the following distribution
Number of
Marks below
Students
10 1
20 5
30 13
40 15
50 16
(a) 3 (b ) 4
(c) 5 (d) 2
430/C/2 JJJJ Page 9 P.T.O.
14. Total surface area (internal and external) of a hemispherical bowl having
radii r1 and r2 (r2 > r1) is :
(a) 2 (r 21 r 22 ) (b) (r 21 r 22 )
(c) 3 r 22 r 21 (d) 3 r 21 r 22
a2 b2
(c) (d) a2 + b2
2 2
a b
(a) p2 = 4q (b) p2 = 4q
(c) p2 = 2q (d) p2 = 2q
(a) no solution
(a) Both Assertion (A) and Reason (R) are true and Reason (R) is the
correct explanation of the Assertion (A).
(b) Both Assertion (A) and Reason (R) are true, but Reason (R) is not
the correct explanation of the Assertion (A).
19. Assertion (A) : PA and PB are tangents to the circle centred at O and
OPA = 30 . Then, PAB is an equilateral triangle.
This section comprises very short answer (VSA) type questions of 2 marks each.
21. Find the centre and radius of a circle having end points of its diameter as
(3, 10) and (1, 4).
tan B tan A
tan (B A) =
1 tan A tan B
OR
(b) Evaluate :
1
sin2 60 2 cos2 45 + cosec2 30
2
n
23. (a) Show that 8 can never end with the digit 0 for any natural
number n.
OR
(b) Find LCM and HCF of 96 and 160, using prime factorisation
method.
24. Find the ratio in which the point P(m, 6) divides the line segment joining
A( 4, 3) and B(2, 8). Also, find the value of m.
25. All kings and queens are removed from a deck of 52 playing cards. Now, a
card is withdrawn at random from the remaining cards. Find the
probability that :
(ii) it is an ace.
This section comprises short answer (SA) type questions of 3 marks each.
26. The length of a rectangular field is 30 m more than its breadth and the
diagonal is 60 m more than its breadth. Find the dimensions of the field.
BC BE
28. (a) In the given figure, = and ABD = ACD. Show that
BD AC
ABD EBC.
OR
(b) In the given figure, ABC and AED are two right triangles, right
angled at B and E respectively. Prove that :
(ii) AB AD = AC AE
430/C/2 JJJJ Page 17 P.T.O.
29. In the given figure, two concentric circles are shown, centred at O. The
radii of the circles are OA = 3 cm and OB = 6 cm.
31. Find the zeroes of the polynomial 4x2 + 17x 15 and verify the
relationship between the zeroes and the coefficients.
SECTION D
This section comprises long answer (LA) type questions of 5 marks each.
32. Prove that the lengths of the tangents drawn from an external point to a
circle are equal.
OR
(b) 36 pens and 24 pencils together cost < 780, while 24 pens and
36 pencils together cost < 720. Find the cost of one pen and of one
pencil.
50 55 2
55 60 8
60 65 12
65 70 24
70 75 38
75 80 16
OR
(b) Find the mean and the mode of the data given below :
Weight Number of
(in kg) students
40 45 5
45 50 11
50 55 20
55 60 24
60 65 28
65 70 12
OR
(ii) (b) If the distances of the top of the poles from the point of
observation are taken as p and q, then find a relation
between p and q. 2
(iii) Find the location of the point of observation. 1
Case Study 2
37. Age of a tree : The most accurate way to determine the age of a tree is to
count the annual rings of wood growth. One such trunk has been shown
here.
Case Study 3
38. A fashion designer is designing a fabric pattern. In each row, there are
some shaded squares and unshaded triangles.
(iii) (a) If each shaded square is of side 2 cm, then find the shaded
area when 15 rows have been designed. 2
OR
(iii) (b)
number of rows. Hence, find S10. 2
Answer 3 1
(a)
4
3.
Answer (c) 18 1
4.
Answer 𝜋𝑟 2 1 1
(c) − 𝑟2
4 2
Answer (c) 2 : 5 1
7.
430/C/2 Page |4
9.
Answer 5 1
(b)
8
11.
Answer (b) 4 1
14.
Answer (b) p2 = - 4q 1
17.
430/C/2 Page |6
18.
19.
Answer (a) Both Assertion (A) and Reason (R) are true and Reason (R) is the 1
correct explanation of the Assertion (A).
20.
3 1 10 4
Centre =
Solution: 1
, = (2, -3)
2 2
Radius = 2 1 3 4 2 = √50 = 5√2 units.
2
1
22.
Solution: 3
22
1 1 1 2
2 2 2 2 2
sin 60 2cos 45 cos ec 30 =
2 0 2 02 0
11⁄2
2
1⁄
= 7/4 2
23.
Solution: 8n = (2 2 2)n 1
For a number to end with zero it should have both 2 and 5 as its prime
1
factors but 8n has only 2 as its prime factor. So, it can not end with zero.
430/C/2 Page |8
1⁄
Solution: 96 = 2 2 2 2 2 3 2
160 = 2 2 2 2 2 5 1⁄
2
HCF = 32 1⁄
2
LCM = 480 1⁄
2
24.
2 k 4 8k 3
Point P is ,
k 1 k 1
8k 3
6
k 1
3
k= 1
2
P divides AB in the ratio 3 : 2
3
2 4
2 2
Also m =
3 5 1
1
2
25.
𝐵𝐶 𝐵𝐸 𝐵𝐶 𝐵𝐸 1
= =
𝐵𝐷 𝐴𝐶 𝐵𝐷 𝐵𝐴
∠B = ∠B (common) 1
430/C/2 P a g e | 10
ABD ∽ EBC [SAS similarity criterion]
Solution: Let R and r be the radii of the outer and inner circles.
430/C/2 P a g e | 11 P.T.O.
θ2πR θ2πr
Perimeter of the shaded region = + + 2(R − r)
360 360
300×2×3.14
= × (6 + 3) + 2 × (6 − 3) 2
360
= 47.10 + 6 = 53.1 cm 1
30.
Solution:
1 for figure
430/C/2 P a g e | 12
Given : TP and TQ are two tangents drawn from an external point T to the
circle C(O,r) 1
To prove: TP = TQ
Proof : In Δ OPT and Δ OQT
OT = OT (common)
0 2
∠OPT = ∠OQT = 90 (Tangent and radius are perpendicular at point of contact)
OP = OQ (radii of the same circle)
∴ Δ OPT ≅ Δ OQT (RHS congruence)
1
∴ TP = TQ (by c.p.c.t)
So, length of the tangents drawn from an external point to circle are equal.
33.
Solution:
Plotting the graph of each line. 2+2
Solution is x = 0 and y = 2 1
33.
Solution: Let the cost of each pen = ₹x and the cost of each pencil = ₹y
36x + 24y = 780 or 3x + 2y = 65 1½
and 24x + 36y = 720 or 2x + 3y = 60 1½
Solving and getting x = 15 and y = 10 1+1
cost of each pen = ₹15 and the cost of each pencil = ₹10
430/C/2 P a g e | 13 P.T.O.
34.
Solution:
xi – 67·5
Class xi fi ui = fiui cf
5
50 – 55 52·5 2 –3 –6 2
55 – 60 57·5 8 –2 – 16 10
60 – 65 62·5 12 -1 -12 22
65 – 70 67·5 = a 24 0 0 46 2
for correct
70 – 75 72·5 38 1 38 84 table
75 – 80 77·5 16 2 32 100
100 36
∑𝑓𝑖 𝑢𝑖
Mean = a + ×h
∑𝑓𝑖
36
= 67·5 + ( × 5)= 69·3 𝟏
100 1+𝟐
N
– cf
Median = l + 2 ×h
f
50 46 1+𝟐
𝟏
= 70 + 5 70.5
38
430/C/2 P a g e | 14
Solution: 𝑥𝑖 – 57·5
Class xi fi ui = fiui
5
40 – 45 42·5 5 –3 – 15
𝟏
45 – 50 47·5 11 –2 – 22 1𝟐
50 – 55 52·5 20 -1 -20
55 – 60 57·5= a 24 0 0 For correct
table
60 – 65 62·5 28 1 28
65 – 70 67·5 12 2 24
100 -5
∑𝑓𝑖 𝑢𝑖
Mean = a + ×h
∑𝑓𝑖
5 1+ ½
= 57.5 + 5 = 57.25
100
f1 – f 0
Mode = l + × h
2f1 – f 0 – f 2
𝟐𝟖−𝟐𝟒
= 60 + 𝟐(𝟐𝟖)−𝟐𝟒−𝟏𝟐 5 = 61 1½ +½
35.
430/C/2 P a g e | 15 P.T.O.
Solution: (i) In AEP and CDP
APE = CPD (Vertically opposite angles)
AEP = PDC (90o each)
AEP CDP (By AA similarity criteria)
1
(i) In ABD and CBE
B = B (Common angle)
ADB = CEB (90o each)
ABD CBE (By AA similarity criteria) 2
Solution: (i) Let AB and CD be the 2 poles and M be a point somewhere between their
bases in the same line.
430/C/2 P a g e | 16
1
for correct
figure
(ii) (a)
h ½
tan 600 hx 3
x
tan 300
h
h
28 x
28 x 3 ½
h 7 3 m 1
OR
p 3 q 1
q 3p
2 2
(iii)
7 3 ½
tan 600 x 7m AM
x
½
MC 28 x 21 m
430/C/2 P a g e | 17 P.T.O.
37.
430/C/2 P a g e | 18
38.
Solution: (i) A.P. for the number of squares in each row is 1, 3, 5, 7, 9…. 1
(ii) A.P. for the number of triangles in each row is 2, 6, 10, 14…. 1
(iii) (a) Area of each square = 2 2 = 4 cm2 1⁄
2
15
Number of squares in 15 rows = 2 14 2 = 225 1
2
Shaded area = 225 4 =900 cm2 1⁄
2
OR
n
(b) Sn 4 (n 1)4 2n 2 1
2
S10 2 102 = 200 1
430/C/2 P a g e | 19 P.T.O.
Series WX1YZ/C SET~3
J{UV (~w{Z`mXr)
MATHEMATICS (BASIC)
*
:3 : 80
Time allowed : 3 hours Maximum Marks : 80
NOTE :
(i) - 27
Please check that this question paper contains 27 printed pages.
(ii) - - -
-
Q.P. Code given on the right hand side of the question paper should be written on the title
page of the answer-book by the candidate.
(iii) - 38
Please check that this question paper contains 38 questions.
(iv) -
Please write down the serial number of the question in the answer-book before
attempting it.
(v) - 15 -
10.15 10.15 10.30 -
-
15 minute time has been allotted to read this question paper. The question paper will be
distributed at 10.15 a.m. From 10.15 a.m. to 10.30 a.m., the students will read the
question paper only and will not write any answer on the answer-book during this period.
SECTION A
1. Two cubes each of 5 cm edge are joined end to end. The surface area of
the resulting cuboid is :
(a) c 2 units
(b) 2c units
(c) 2 c units
(d) c units
4. The HCF of two numbers is 27 and their LCM is 162. If one of the
numbers is 54, the other number is :
(a) 36 (b) 45
(c) 9 (d) 81
5. At some time of the day, the height and length of the shadow of a man
:
(a) 30 (b) 45
(c) 60 (d) 90
(a) 36
(b) 6
(c) 36
(d) 6
20 5
30 13
40 15
50 16
(a) 30 40 (b ) 40 50
(c) 20 30 (d) 10 20
7 2 6
(a) (b)
5 5
24 5
(c) (d)
49 7
(a) x (x2 + 6) (b ) 6x (x + 6)
r2 1 2
(a) r
2 2
2 r 1 2
(b) r
4 2
r2 1 2
(c) r
4 2
2 r
(d) r 2 sin 90
4
a2 b2
(c) (d) a2 + b2
2 2
a b
16. Total surface area (internal and external) of a hemispherical bowl having
radii r1 and r2 (r2 > r1) is :
(a) 2 (r 21 r 22 ) (b) (r 21 r 22 )
(c) 3 r 22 r 21 (d) 3 r 21 r 22
18. If the mean and the mode of a distribution are 17 and 20 respectively,
then the median of the distribution, using empirical formula, is :
31
(a) 20 (b)
3
(c) 18 (d) 17
(a) Both Assertion (A) and Reason (R) are true and Reason (R) is the
correct explanation of the Assertion (A).
(b) Both Assertion (A) and Reason (R) are true, but Reason (R) is not
the correct explanation of the Assertion (A).
20. Assertion (A) : PA and PB are tangents to the circle centred at O and
OPA = 30 . Then, PAB is an equilateral triangle.
This section comprises very short answer (VSA) type questions of 2 marks each.
22. Find the centre and radius of a circle having end points of its diameter as
(3, 10) and (1, 4).
23. Find the ratio in which a line segment joining the points ( 1, 1) and (5, 4)
is divided by the y-axis.
n
24. (a) Show that 12
number.
OR
tan B tan A
tan (B A) =
1 tan A tan B
OR
(b) Evaluate :
1
sin2 60 2 cos2 45 + cosec2 30
2
This section comprises short answer (SA) type questions of 3 marks each.
26. Sum of the areas of two squares is 468 m2. If the difference between their
sides is 6 m, then find the sides of the two squares.
BC BE
28. (a) In the given figure, = and ABD = ACD. Show that
BD AC
ABD EBC.
OR
(b) In the given figure, ABC and AED are two right triangles, right
angled at B and E respectively. Prove that :
(ii) AB AD = AC AE
SECTION D
This section comprises long answer (LA) type questions of 5 marks each.
32. (a) The following table gives production yield per hectare of wheat of
100 farms of a village :
Production yield Number of
(in kg/ha) farms
50 55 2
55 60 8
60 65 12
65 70 24
70 75 38
75 80 16
Find the mean and median of the data.
OR
430/C/3 JJJJ Page 19 P.T.O.
(b) Find the mean and the mode of the data given below :
Weight Number of
(in kg) students
40 45 5
45 50 11
50 55 20
55 60 24
60 65 28
65 70 12
OR
(b) 7 chairs and 4 tables for a classroom together cost < 7,000; while
5 chairs and 3 tables together cost < 5,080. Find the cost of each
chair and each table.
34. Prove that the lengths of the tangents drawn from an external point to a
circle are equal.
35. In the given figure, ACB = 90 and CD AB. Prove that CD2 = BD AD.
Case Study 1
36. A fashion designer is designing a fabric pattern. In each row, there are
some shaded squares and unshaded triangles.
(iii) (a) If each shaded square is of side 2 cm, then find the shaded
area when 15 rows have been designed. 2
OR
OR
38. Totem poles are made from large trees. These poles are carved with
symbols or figures and mostly found in western Canada and
northwestern United States.
In the given picture, two such poles of equal heights are standing 28 m
apart. From a point somewhere between them in the same line, the
angles of elevation of the top of the two poles are 60 and 30
respectively.
OR
(ii) (b) If the distances of the top of the poles from the point of
observation are taken as p and q, then find a relation
between p and q. 2
Answer (b) p2 = - 4q 1
Answer 1
(d) 81
5
430/C/3 3 P.T.O
6
Answer (c) 2 : 5 1
Answer 1
(a)1300
9
Answer 5 1
(d)
7
430/C/3 4 P.T.O
11
Answer 1
(b) 6x(x+6)
12
Answer 𝜋𝑟 2 1 1
(c) 4
− 2 𝑟2
13
14
15
430/C/3 5 P.T.O
17
Answer 2
(b)
7
18
Answer (c) 18 1
19
Answer (b) Both Assertion (A) and Reason (R) are true, but Reason (R) is not the correct explanation 1
of the Assertion (A).
20
Answer (a) Both Assertion (A) and Reason (R) are true, and Reason (R) is the correct explanation of 1
the Assertion (A).
430/C/3 6 P.T.O
SECTION B
21
22
Solution 3 1 10 4 1
Centre = , = (2, -3)
2 2
Radius = 2 12 3 4 2 = √50 = 5√2 units. 1
23
5k 1 4k 1
Point P is ,
k 1 k 1
P lies on y- axis so x = 0
5𝑘−1 1+ ½
=0
𝑘+1
1 ½
k = 5
P divides AB in the ratio 1:5
430/C/3 7 P.T.O
24(a)
Solution
12n = (2 2 3)n
(a) 1
For a number to end with zero it should have both 2 and 5 as its prime
factors but 12n has only prime numbers 2 and 3 as its factors so it can not
end with zero. 1
(b)
25(a)
(b)
Solution 3
2 2
1 1 1 2
2 2 2 2 2
(b) sin 60 2cos 45 cos ec 30 =
2 0 2 0 2 0
11⁄2
2
7 1⁄
= 2
4
430/C/3 8 P.T.O
SECTION C
26
27(a)
430/C/3 9 P.T.O
28(a)
430/C/3 10 P.T.O
29
Solution Let R and r be the radii of the outer and inner circles.
θ2πR θ2πr
Perimeter of the shaded region = + + 2(R − r)
360 360
=
300×2×3.14
× (6 + 3) + 2 × (6 − 3) 2
360
= 47.10 + 6 = 53.1 cm 1
30
Solution cos 2 1
1 1+ ½
sin cos
2
cos
L.H .S
1 sin (1 cos )(1 sin )
1 (1 sin ) cos (1 sin ) cos 1
R.H .S
cos
2
1 cos (1 sin 2 )(1 cos )
cos
½
(1 cos )(1 sin )
L.H.S = R.H. S
31
Solution 8x2+3x -5
= (x + 1) (8x - 5) 1
5
x= -1 and x= are the zeroes of the polynomial 1
8
−3 coefficient of x
Sum of zeroes = = 1⁄
2
8 coefficient of x 2
−5 1⁄
𝑐𝑜𝑛𝑠𝑡𝑎𝑛𝑡 𝑡𝑒𝑟𝑚 2
Product of zeroes = = 𝑐𝑜𝑒𝑓𝑓𝑖𝑐𝑖𝑒𝑛𝑡 𝑜𝑓 𝑥 2
8
430/C/3 11 P.T.O
SECTION D
32 (a)
Solution
(a) xi – 67·5
Class xi fi ui = fiui cf
5
50 – 55 52·5 2 –3 –6 2
55 – 60 57·5 8 –2 – 16 10
60 – 65 62·5 12 -1 -12 22
65 – 70 67·5 24 0 0 46 2
for correct
70 – 75 72·5 38 1 38 84 table
75 – 80 77·5 16 2 32 100
100 36
∑𝑓𝑖 𝑢𝑖
Mean = a + ×h
∑𝑓𝑖
𝟏
36 1+𝟐
= 67·5 + ( ×5 )= 69·3
100
N
– cf
Median = l + 2 ×h
f
50 46
= 70 + 5 70.5 1+𝟐
𝟏
38
430/C/3 12 P.T.O
(b)
Solution 𝑥𝑖 – 57·5
(b)
Class xi fi ui = fi u i
5
40 – 45 42·5 5 –3 – 15 𝟏
1𝟐
45 – 50 47·5 11 –2 – 22
50 – 55 52·5 20 -1 -20 for correct
57·5= table
55 – 60 24 0 0
a
60 – 65 62·5 28 1 28
65 – 70 67·5 12 2 24
100 -5
∑𝑓𝑖 𝑢𝑖
Mean = a + ×h
∑𝑓𝑖
5 1+ ½
= 57.5 +
5 57.25
100
f1 – f0
Mode = l +
2f – f – f × h
1 0 2
= 60 +
𝟐𝟖−𝟐𝟒
5 = 61 1½ +½
𝟐(𝟐𝟖)−𝟐𝟒−𝟏𝟐
33(a)
Solution
(a)
2+2
1
430/C/3 13 P.T.O
(b)
Solution Let the cost of one chair be ₹ x and that of one table be ₹ y
(b) 7x+4y=7000 ---------------(i) 1½
5x+3y=5080 ---------------(ii) 1½
Solution
1 for figure
Given : TP and TQ are two tangents drawn from an external point T to the 1
circle C(O,r)
To prove: TP = TQ
Proof : In Δ OPT and Δ OQT
OT = OT (common)
∠OPT = ∠OQT = 900 (Tangent and radius are perpendicular at point of contact) 2
OP = OQ (radii of the same circle)
∴ Δ OPT ≅ Δ OQT (RHS congruence)
∴ TP = TQ (by c.p.c.t) 1
So, length of the tangents drawn from an external point to circle are equal.
35
Solution LetA
ACD 900 BCD -------------(i) 1
In CDA and BDC
A BCD (using (i)) 2
CDA BDC 90 0
430/C/3 14 P.T.O
CD DA 1
BD DC
1
CD 2 BD DA
SECTION E
36
Solution (i) A.P. for the number of squares in each row is 1, 3, 5, 7, 9…. 1
(ii) A.P. for the number of triangles in each row is 2, 6, 10, 14…. 1
(iii) (a) Area of each square = 2 2 = 4 cm2 1⁄
2
15
Number of squares in 15 rows = 2 14 2 =225 1
2
Shaded area = 225 4 =900 cm2 1⁄
2
OR
430/C/3 15 P.T.O
n
(b) Sn 4 (n 1)4 2n2 1
2
S10 2 102 = 200 1
37
1 22 1½
(iii) (a) Area of segment ACBA = 4 7 21 21 – 220.5 ½
= 126 cm2.
OR
1 22
(b) Length of arc AB = 4 7 2 21 =33 cm. 1
Perimeter of sector OACBO =21+33+21 =75 cm. 1
430/C/3 16 P.T.O
38
Solution
(i) Let AB and CD be the 2 poles and M be a point somewhere between their bases in the
same line.
1
for correct
figure
430/C/3 17 P.T.O
(ii) (a)
h 1⁄
tan 600 hx 3 2
x
tan 300
h
h
28 x
1⁄
28 x 3 2
h 7 3 m 1
OR
(iii)
7 3 ½
tan 600 x 7m AM
x
½
MC 28 x 21 m
430/C/3 18 P.T.O
430/1/1
MARKING SCHEME
MATHEMATICS (BASIC)
SECTION A
Ans. (c) 25 32 1
__________________________________________________________________________
1
Ans. (d) 1
2
__________________________________________________________________________
Ans. (b) 49 1
__________________________________________________________________________
Ans. (a) 0 1
__________________________________________________________________________
Ans. (a) 4 : 7 1
__________________________________________________________________________
430/1/1 4
6
Ans. (c) cm 1
5
__________________________________________________________________________
Ans. (b) – 5, 6 1
__________________________________________________________________________
Ans. (b) 1 1
__________________________________________________________________________
430/1/1 5 P.T.O.
Ans. (a) 7 cm 1
__________________________________________________________________________
Ans. (a) 30 cm 1
__________________________________________________________________________
430/1/1 6
Ans. (c) 24 cm 1
__________________________________________________________________________
Ans. (b) Both Assertion (A) and Reason (R) are true but Reason (R) is not the
correct explanation of Assertion (A) 1
__________________________________________________________________________
Ans. (a) Both Assertion (A) and Reason (R) are true and Reason (R) is the
correct explanation of Assertion (A) 1
__________________________________________________________________________
SECTION B
Solution:
430/1/1 7 P.T.O.
Let P(x, y) divide AB internally in the ratio 2 : 3
2 – 3 3 7 15
x= = =3 1
23 5
24 3– 1 5
y= = =1 1
23 5
OR
3
2
2 2 1 1
= – 2(2) – 2 1
3 2
17 1
=–
3 2
__________________________________________________________________________
1
Solution: 92 = 2 2 23
2
1
510 = 2 3 5 17
2
1
HCF = 2
2
430/1/1 8
1
LCM = 2 2 3 5 17 23 = 23460
2
__________________________________________________________________________
22 8
x= and y = 1+1
5 5
OR
5 3 11 1 1 1 1
Solution: – = – or – = – 1
10 6 22 2 2 2 2
1
given pair of linear equations is inconsistent
2
__________________________________________________________________________
430/1/1 9 P.T.O.
Solution: LHS = sec (1 – sin ) (sec + tan )
1 1 sin
= (1 – sin ) 1
cos cos cos
1 1 sin
= (1 – sin ) 1
cos cos
1 – sin 2 cos2 1 1
= = = 1 = RHS +
2 2
cos cos 2 2
OR
1
1
1 sec cos = 1 + cos
Solution: LHS = = 1
sec 1
cos
(1 – cos ) (1 cos )
= 1
(1 – cos )
1 – cos2 sin 2
= = = RHS 1
1 – cos 1 – cos
__________________________________________________________________________
Solution: AB = (4 – 1) 2 (2 – 7) 2 = 34
BC = (4 1)2 (2 1)2 = 34 2
CD = (– 4 1)2 (4 1)2 = 34
DA = (– 4 – 1)2 (4 – 7)2 = 34
AB = BC = CD = DA
AC = (1 1)2 (7 1)2 = 68 1
BD = (4 4)2 (2 – 4)2 = 68
AC = BD
Hence, ABCD is a square.
__________________________________________________________________________
430/1/1 10
Solution:
1
(For Fig.)
Solution: p(x) = x2 + 3x + 2
, are its zeroes
1 1
+ = – 3, = 2 +
2 2
Now,
1
( + 1) + ( + 1) = + + 2 = – 3 + 2 = – 1
2
( + 1) ( + 1) = + ( + ) + 1 = + 2 – 3 + 1 = 0 1
1
Required Polynomial is k(x2 + x) or x2 + x
2
__________________________________________________________________________
OR
Solution:
430/1/1 14
h 1
In ABC, tan 60 = h= 3x 1+
x 2
h 1 h 1
In ABD, tan 30 = = 1+
20 x 3 20 x 2
3 h = 20 + x
3 3 x = 20 + x
1
x = 10
2
h= 3 x = 10 3
1
Height of tower = 10 3 m or 17·3 m
2
OR
Solution:
h 1
In APC, = tan 45 h = x 1+
x 2
4000 4000 1
In APC, = tan 60 x = 1+
x 3 2
4000
h=x=
3
4000 1
Distance between the aeroplanes = 4000 –
3 2
1
= 40001
3
5080 1
= m or 1693.33 m (approx.)
3 2
(Note: ½ mark to be deducted for not using 3 =1.73)
__________________________________________________________________________
430/1/1 15 P.T.O.
SECTION E
Solution: a = 2, d = 3
(i) Number of pots in the 10th row
= a10 = a + 9d = 29 1
(ii) a5 – a2 = (a + 4d) – (a + d) = 3d = 9 1
n
(iii) Sn = 100 [2(2) + (n – 1)3] = 100 1
2
2
3n + n – 200 = 0 (3n + 25) (n – 8) = 0
25
n = 8 (n = – rejected), 1
3
OR
12
(iii) S12 = [2(2) + 11(3)] 1
2
= 222 1
__________________________________________________________________________
430/1/1 16
Solution: (i) Area of square ABCD = (40)2 = 1600 cm2 1
22
(ii) Area of circle = r2 = 10 10
7
2200
= cm2 or 314.28 cm2 1
7
1 2200
(iii) Area of 4 quadrants = 4( r2) = cm2 1
4 7
2200 2200
Remaining area = 1600 –
7 7
430/1/1 17 P.T.O.
4400 6800
= 1600 – = cm2 or 971.43 cm2 1
7 7
OR
1 2 2200
(iii) Area of 4 quadrants = 4( r ) = cm2 1
4 7
Combined area of circle + 4 quadrants
2200 2200 4400
= + = cm2 or 628.57 cm2 1
7 7 7
430/1/1 18
21
Solution: (i) P(type O) = 1
50
1
(ii) No. of people with AB type blood group = 50 – (21 + 22 + 5) = 2
2
2 1 1
P(type AB) = or
50 25 2
21 2 23
(iii) P(neither type A nor type B) = = 1+1
50 50
OR
21 22 5 24
(iii) P(type A or type B or type O) = = 1+1
50 25
__________________________________________________________________________
430/1/1 19 P.T.O.
430/1/2
MARKING SCHEME
MATHEMATICS (BASIC)
SECTION A
Ans. (b) 1 1
__________________________________________________________________________
6
Ans. (c) cm 1
5
__________________________________________________________________________
Ans. (b) 4 1
__________________________________________________________________________
Ans. (b) 24 33 1
__________________________________________________________________________
430/1/2 3 P.T.O.
__________________________________________________________________________
1
Ans. (d) 1
52
__________________________________________________________________________
Ans. (c) 9 1
__________________________________________________________________________
Ans. (b) 2 1
__________________________________________________________________________
Ans. (c) 24 cm 1
__________________________________________________________________________
430/1/2 4
Ans. (a) 4 : 7 1
__________________________________________________________________________
Ans. (a) 0 1
__________________________________________________________________________
Ans. (a) 7 cm 1
__________________________________________________________________________
430/1/2 5 P.T.O.
Ans. (a) 30 cm 1
__________________________________________________________________________
Ans. (d) – 2, – 1 1
__________________________________________________________________________
430/1/2 6
Ans. (a) Both Assertion (A) and Reason (R) are true and Reason (R) is the
correct explanation of Assertion (A) 1
__________________________________________________________________________
Ans. (b) Both Assertion (A) and Reason (R) are true but Reason (R) is not the
correct explanation of Assertion (A) 1
__________________________________________________________________________
SECTION B
5 3 11 1 1 1 1
Solution: – = – or – = – 1
10 6 22 2 2 2 2
1
given pair of linear equations is inconsistent
2
__________________________________________________________________________
430/1/2 7 P.T.O.
Solution:
OR
430/1/2 8
1
Solution: 92 = 2 2 23
2
1
510 = 2 3 5 17
2
1
HCF = 2
2
1
LCM = 2 2 3 5 17 23 = 23460
2
__________________________________________________________________________
SECTION C
430/1/2 9 P.T.O.
Solution: (b)
Solution: p(x) = x2 + 3x + 2
, are its zeroes
1 1
+ = – 3, = 2 +
2 2
Now,
1
( + 1) + ( + 1) = + + 2 = – 3 + 2 = – 1
2
( + 1) ( + 1) = + ( + ) + 1 = + 2 – 3 + 1 = 0 1
1
Required Polynomial is k(x2 + x) or x2 + x
2
__________________________________________________________________________
Solution:
1
(For Fig.)
430/1/2 11 P.T.O.
AC = (1 1)2 (7 1)2 = 68 1
BD = (4 4)2 (2 – 4)2 = 68
AC = BD
Hence, ABCD is a square.
__________________________________________________________________________
SECTION D
Solution:
h 1
In ABC, tan 60 = h= 3x 1+
x 2
h 1 h 1
In ABD, tan 30 = = 1+
20 x 3 20 x 2
3 h = 20 + x
1
x = 10
2
h= 3 x = 10 3
1
Height of tower = 10 3 m or 17·3 m
2
OR
Solution:
430/1/2 12
h 1
In APB, = tan 45 h = x 1+
x 2
4000 4000 1
In APB, = tan 60 x = 1+
x 3 2
4000
h=x=
3
4000 1
Distance between the aeroplanes = 4000 –
3 2
1
= 40001
3
5080 1
= m or 1693.33 m (approx.)
3 2
(Note: ½ mark to be deducted for not using 3 =1.73)
__________________________________________________________________________
Solution:
Daily Exp. (₹) No. of household xi fixi
(fi)
100 – 150 4 125 500
150 – 200 5 175 875
200 – 250 12 225 2700
250 – 300 2 275 550
300 – 350 2 325 650
25 5275
– f i xi 5275 1
Mean x = = = 211 1
fi 25 2
1
Mode: Modal Class = 200 – 250
2
l = 200, f1 = 12, f0 = 5, f2 = 2, h = 50
f1 – f0
Mode = l + h
2f1 – f0 – f2
430/1/2 13 P.T.O.
12 – 5
= 200 + 50 1
24 – 5 – 2
3750 1
= or 220·59
17 2
__________________________________________________________________________
430/1/2 14
Solution: r + h = 37 cm
2rh + 2r2 = 1628 or 2r (h + r) = 1628 1
22
2 r 37 = 1628 1
7
r = 7 cm 1
1
and h = 37 – 7 = 30 cm
2
Volume of cylinder = r2h
22
= 7 7 30 1
7
1
= 4620 cm3
2
__________________________________________________________________________
SECTION E
430/1/2 15 P.T.O.
21
Solution: (i) P(type O) = 1
50
1
(ii) No. of people with AB type blood group = 50 – (21 + 22 + 5) = 2
2
2 1 1
P(type AB) = or
50 25 2
21 2 23
(iii) P(neither type A nor type B) = = 1+1
50 50
OR
21 22 5 24
(iii) P(type A or type B or type O) = = 1+1
50 25
__________________________________________________________________________
430/1/2 16
Solution: a = 2, d = 3
(i) Number of pots in the 10th row
= a10 = a + 9d = 29 1
(ii) a5 – a2 = (a + 4d) – (a + d) = 3d = 9 1
n
(iii) Sn = 100 [2(2) + (n – 1)3] = 100 1
2
2
3n + n – 200 = 0 (3n + 25) (n – 8) = 0
25
n = 8 (n = – rejected), 1
3
OR
12
(iii) S12 = [2(2) + 11(3)] 1
2
= 222 1
__________________________________________________________________________
430/1/2 17 P.T.O.
Solution: (i) Area of square ABCD = (40)2 = 1600 cm2 1
22
(ii) Area of circle = r2 = 10 10
7
2200
= cm2 or 314.28 cm2 1
7
1 2200
(iii) Area of 4 quadrants = 4( r2) = cm2 1
4 7
2200 2200
Remaining area = 1600 –
7 7
4400 6800
= 1600 – = cm2 or 971.43 cm2 1
7 7
OR
1 2200
(iii) Area of 4 quadrants = 4( r2) = cm2 1
4 7
Combined area of circle + 4 quadrants
2200 2200 4400
= + = cm2 or 628.57cm2 1
7 7 7
__________________________________________________________________________
430/1/2 18
430/1/3
MARKING SCHEME
MATHEMATICS (BASIC)
SECTION A
Ans. (c) 26 33 1
__________________________________________________________________________
Ans. (d) 28 1
__________________________________________________________________________
430/1/3 3 P.T.O.
Ans. (d) r3 1
__________________________________________________________________________
Ans. (c) 24 cm 1
__________________________________________________________________________
430/1/3 4
Ans. (b) 49 1
__________________________________________________________________________
Ans. (a) 30 cm 1
__________________________________________________________________________
6
Ans. (c) cm 1
5
__________________________________________________________________________
430/1/3 5 P.T.O.
Ans. (a) 5 1
__________________________________________________________________________
5 2
Ans. (a) , 1
3 3
__________________________________________________________________________
1
Ans. (c) 1
3
__________________________________________________________________________
Ans. (a) 4 : 7 1
__________________________________________________________________________
Ans. (a) 7 cm 1
__________________________________________________________________________
1
Ans. (d) 1
2
__________________________________________________________________________
430/1/3 6
Ans. (a) Both Assertion (A) and Reason (R) are true and Reason (R) is the
correct explanation of Assertion (A) 1
__________________________________________________________________________
Ans. (b) Both Assertion (A) and Reason (R) are true but Reason (R) is not the
correct explanation of Assertion (A) 1
__________________________________________________________________________
SECTION B
2
3 1 2 1 2 1 1 1
Solution: – 2(0) – (2) + +
2 3 2 2 2 2
3 1
=–
2 2
__________________________________________________________________________
430/1/3 7 P.T.O.
Solution: In ABC and AMP,
1
ABC = AMP (90 each)
2
BAC = MAP (common) 1
By AA Similarity
1
ABC AMP
2
__________________________________________________________________________
Solution:
1
Solution: 92 = 2 2 23
2
1
510 = 2 3 5 17
2
1
HCF = 2
2
1
LCM = 2 2 3 5 17 23 = 23460
2
__________________________________________________________________________
5 3 11 1 1 1 1
Solution: – = – or – = – 1
10 6 22 2 2 2 2
1
given pair of linear equations is inconsistent
2
__________________________________________________________________________
SECTION C
Solution: p(x) = x2 + 3x + 2
, are its zeroes
1 1
+ = – 3, = 2 +
2 2
Now,
1
( + 1) + ( + 1) = + + 2 = – 3 + 2 = – 1
2
( + 1) ( + 1) = + ( + ) + 1 = + 2 – 3 + 1 = 0 1
1
Required Polynomial is k(x2 + x) or x2 + x
2
__________________________________________________________________________
430/1/3 9 P.T.O.
Solution: In ABE, DF AE (given), hence by BPT
BD BF 1
= _________ (i) 1
DA FE 2
In ABC, DE AC (given), hence by BPT
BD BE
= _________ (ii) 1
DA EC
From (i) and (ii)
BF BE 1
=
FE EC 2
OR
430/1/3 10
Solution:
1
(For Fig.)
cos A 1 sin A
Solution: (a) LHS = +
1 sin A cos A
cos A (1 sin A) 2
2
= 1
(1 sin A)cos A
cos2 A 1 sin 2 A 2 sin A
= 1
(1 sin A)cos A
1 1 2 sin A 2(1 sin A) 1
= =
(1 sin A)cos A (1 sin A)cos A 2
430/1/3 11 P.T.O.
2 1
= = 2 sec A = RHS
cos A 2
OR
430/1/3 12
8– 7 1
=3+ 2 1
16 – 7 – 2 2
23 1
= or 3.287
7 2
__________________________________________________________________________
7 1
Solution: Radius of cone = 12 m, h= m
2 2
1 2
Volume of cone = r h
3
1 22 7 1
= 12 12 1
3 7 2 2
1
= 528 m3
2
25
l (slant height) = h2 r2 = m 1
2
Curved surface area of cone = rl
22 25
= 12 1
7 2
3300 2 1
= m or 471·43 m2
7 2
Cloth required to cover the heap = 471·43 m2
__________________________________________________________________________
Solution:
h 1
In ABC, tan 60 = h= 3x 1+
x 2
h 1 h 1
In ABD, tan 30 = = 1+
20 x 3 20 x 2
430/1/3 13 P.T.O.
3 h = 20 + x
1
x = 10
2
h= 3 x = 10 3
1
Height of tower = 10 3 m or 17.3 m
2
OR
Solution:
h 1
In APB, = tan 45 h = x 1+
x 2
4000 4000 1
In APB, = tan 60 x = 1+
x 3 2
4000
h=x=
3
4000 1
Distance between the aeroplanes = 4000 –
3 2
1
= 40001
3
5080 1
= m or 1693.33 m (approx.)
3 2
(Note: ½ mark to be deducted for not using 3 =1.73)
__________________________________________________________________________
430/1/3 15 P.T.O.
Solution: (i) Area of square ABCD = (40)2 = 1600 cm2 1
22
(ii) Area of circle = r2 = 10 10
7
2200
= cm2 or 314.28 cm2 1
7
1 2200
(iii) Area of 4 quadrants = 4( r2) = cm2 1
4 7
2200 2200
Remaining area = 1600 –
7 7
4400 6800
= 1600 – = cm2 or 971.43 cm2 1
7 7
OR
1 2200
(iii) Area of 4 quadrants = 4( r2) = cm2 1
4 7
Combined area of circle + 4 quadrants
2200 2200 4400
= + = cm2 or 628.57 cm2 1
7 7 7
__________________________________________________________________________
430/1/3 16
21
Solution: (i) P(type O) = 1
50
1
(ii) No. of people with AB type blood group = 50 – (21 + 22 + 5) = 2
2
2 1 1
P(type AB) = or
50 25 2
21 2 23
(iii) P(neither type A nor type B) = = 1+1
50 50
OR
21 22 5 24
(iii) P(type A or type B or type O) = = 1+1
50 25
__________________________________________________________________________
430/1/3 17 P.T.O.
Solution: a = 2, d = 3
(i) Number of pots in the 10th row
= a10 = a + 9d = 29 1
(ii) a5 – a2 = (a + 4d) – (a + d) = 3d = 9 1
n
(iii) Sn = 100 [2(2) + (n – 1)3] = 100 1
2
2
3n + n – 200 = 0 (3n + 25) (n – 8) = 0
25
n = 8 (n = – rejected), 1
3
OR
12
(iii) S12 = [2(2) + 11(3)] 1
2
= 222 1
__________________________________________________________________________
430/1/3 18
Series WX1YZ/2 SET~1
J{UV (~w{Z`mXr)
MATHEMATICS (BASIC)
*
:3 : 80
Time allowed : 3 hours Maximum Marks : 80
NOTE :
(i) - 27
Please check that this question paper contains 27 printed pages.
(ii) - - -
-
Q.P. Code given on the right hand side of the question paper should be written on the title
page of the answer-book by the candidate.
(iii) - 38
Please check that this question paper contains 38 questions.
(iv) -
Please write down the serial number of the question in the answer-book before
attempting it.
(v) - 15 -
10.15 10.15 10.30 -
-
15 minute time has been allotted to read this question paper. The question paper will be
distributed at 10.15 a.m. From 10.15 a.m. to 10.30 a.m., the students will read the
question paper only and will not write any answer on the answer-book during this period.
SECTION A
2. In what ratio does x-axis divide the line segment joining the points
A(2, 3) and B(5, 6) ?
(a) 2:3 (b) 2:1
(c) 3:4 (d) 1:2
(a) 9 (b) 0
1
(c) 8 (d)
9
100
(a) 50 3 m (b ) m
3
50
(c) m (d) 25 3 m
3
(c) 65 (d) 90
(a) 2 R (b) 2 R
180 360
2 2
(c) R (d) R
180 360
9. A die is rolled once. The probability that a composite number comes up,
is :
1 2
(a) (b)
2 3
1
(c) (d) 0
3
1
10. If the quadratic equation 9x2 + bx + = 0 has equal roots, then the
4
value of b is :
(a) 0 (b) 3 only
(c) 3 only (d) ±3
4 3
(a) r3 (b) r
3
2 3
(c) 3 r3 (d) r
3
(a) 2 (b) 3
(c) 4 (d) 5
(a) is inconsistent
(a) 27 (b) 29
29
(c) 18 (d)
3
2 1 cos2 A
18. If tan A = , then the value of is :
5 1 sin2 A
25 4
(a) (b)
4 25
4 5
(c) (d)
5 4
20. Assertion (A) : The probability of getting a prime number, when a die is
2
thrown once, is .
3
SECTION B
This section comprises very short answer (VSA) type questions of 2 marks each.
BE BC
21. In the given figure, DE || AC and . Prove that DC ||AP.
EC CP
OR
n
(b) Show that (15) cannot end with the digit 0 for any natural
23. (a)
Q and R are (6, 1), (1, 3) and (x, 8) respectively.
OR
(b) The vertices of a triangle are ( 2, 0), (2, 3) and (1, 3). Is the
24. 2 + 2kx + x k 5
25. From a well-shuffled deck of 52 playing cards, all diamond cards are
removed. Now, a card is drawn from the remaining pack at random. Find the
This section comprises short answer (SA) type questions of 3 marks each.
26. In the given figure, two concentric circles with centre O are shown. Radii
of the circles are 2 cm and 5 cm respectively. Find the area of the shaded
region.
OR
OR
30. Find the zeroes of the polynomial p(x) = 2x2 7x 15 and verify the
relationship between its coefficients and zeroes.
31. Prove that the points A( 1, 0), B(3, 1), C(2, 2) and D( 2, 1) are the
vertices of a parallelogram ABCD. Is it also a rectangle ?
SECTION D
This section comprises long answer (LA) type questions of 5 marks each.
32. (a) From the top of a building 60 m high, the angles of depression of
the top and bottom of a tower are observed to be 30 and 60
respectively. Find the height of the tower. Also, find the distance
between the building and the tower. (Use 3 = 1·732)
OR
(b) The angle of elevation of the top of a building from a point A on the
ground is 30 . On moving a distance of 30 m towards its base to the
point B, the angle of elevation changes to 45 . Find the height of
the building and the distance of its base from point A.
(Use 3 = 1·732)
Number of
Marks
Students
0 10 3
10 20 5
20 30 16
30 40 12
40 50 13
50 60 20
60 70 6
70 80 5
34. If a line is drawn parallel to one side of a triangle to intersect the other
two sides in distinct points, then prove that the other two sides are
divided in the same ratio.
35. (a) If the sum of the first 7 terms of an A.P. is 14 and that of
11 terms is
OR
Yellow flowers
Red roses
It is given that the combined area of the two segments (of flowers) is
2
256 sq m.
3
Based on the above, answer the following questions :
(i) Write an equation representing the total area of the two segments
in terms of r . 1
(ii) Find the value of 1
(iii) (a) Find the area of the segment with red roses. 2
OR
(iii) (b) Find the area of the segment with yellow flowers. 2
37. Circles play an important part in our life. When a circular object is hung
on the wall with a cord at nail N, the cords NA and NB work like
(iii) (a) Find the total length of cords NA, NB and the chord AB. 2
OR
38. A wooden toy is shown in the picture. This is a cuboidal wooden block of
dimensions 14 cm 17 cm 4 cm. On its top there are seven cylindrical
hollows for bees to fit in. Each cylindrical hollow is of height 3 cm and
radius 2 cm.
(i) Find the volume of wood carved out to make one cylindrical hollow. 1
(ii) Find the lateral surface area of the cuboid to paint it with green
colour. 1
(iii) (a) Find the volume of wood in the remaining cuboid after
carving out seven cylindrical hollows. 2
OR
(iii) (b) Find the surface area of the top surface of the cuboid to be
painted yellow. 2
SECTION A
Answer (c) 22 1
--------------------------------------------------------------------------------------------------------------
Answer (d) 1 : 2 1
------------------------------------------------------------------------------------------------------------
Answer (a) 9 1
--------------------------------------------------------------------------------------------------------------
𝜶
Answer (d) × πR2 1
𝟑𝟔𝟎
---------------------------------------------------------------------------------------------------------------
Answer (a) 2 cm 1
---------------------------------------------------------------------------------------------------------------
4
430/2/1 P.T.O
𝟏
Answer (c) 1
𝟑
-------------------------------------------------------------------------------------------------------------
Answer (d) 3 1
----------------------------------------------------------------------------------------------------------
𝟒
Answer (b) r3
𝟑
1
-----------------------------------------------------------------------------------------------------------
5
430/2/1 P.T.O
Answer (a) 2 1
---------------------------------------------------------------------------------------------------------
𝟏
Answer (c) 1
𝟎·𝟓
--------------------------------------------------------------------------------------------------------------
6
430/2/1 P.T.O
Answer (d) 9 1
-----------------------------------------------------------------------------------------------------------
Answer (a) 27 1
-------------------------------------------------------------------------------------------------------------
7
430/2/1 P.T.O
𝟒
Answer (b) 1
𝟐𝟓
---------------------------------------------------------------------------------------------------------------
8
430/2/1 P.T.O
SECTION B
BE BD
Solution In ABC, DE || AC = 1
EC DA
𝑩𝑬 BC 𝑩𝑫 BC
Also given, = = 1/2
𝑬𝑪 CP 𝑫𝑨 CP
630 = 2 × 32 × 5 × 7 1/2
HCF = 2 × 32 × 5 = 90 1
OR
For a number to end with zero it should have both 2 and 5 in its
prime factorization but 15n has only prime numbers 3 and 5 as its
factors so it can not end with zero. 1
----------------------------------------------------------------------------------------------------
9
430/2/1 P.T.O
Solution (a) PQ = QR ( 6 − 1) + ( −1 − 3) ( x − 1) + (8 − 3)
2 2 2 2
= 1
OR
AB = 4 2 + 32 = 5 1/2
BC = (– 1) 2 + (– 6) 2 = 37 1/2
CA = (1 + 2) 2 + (– 3) 2 = 3 2 1/2
4k + 2 = k + 5 k = 1 1/2
-------------------------------------------------------------------------------------------------------------
10
430/2/1 P.T.O
Solution Total number of cards = 52 – 13 = 39 1/2
Number of kings = 3 1/2
𝟑 𝟏
P(drawn card is a king) = 𝟑𝟗 𝐨𝐫 1
13
-----------------------------------------------------------------------------------------------------------------
SECTION C
𝝅 × 5𝟐 × 60° 𝟐𝟓𝝅
Solution Area of sector OABC = = cm2 1
360° 𝟔
𝝅 × 2𝟐 × 60° 𝟒𝝅
Area of sector OED = = cm2 1
360° 𝟔
𝟐𝟓𝝅 𝟒𝝅 𝟐𝟏 22
Area of shaded region = – = × = 11 cm2 1
𝟔 𝟔 𝟔 𝟕
-----------------------------------------------------------------------------------------------------------------
---
11
430/2/1 P.T.O
--------------------------------------------------------------------------------------------------------------
Solution (a)
Tangents from an external point are equal therefore
AP = AS, BP = BQ, QC = CR and DR = DS 1
AB + CD = (AP + PB) + (CR + RD) 1/2
= (AS + BQ) + (CQ + DS) 1/2
= (AS + DS) + (BQ + CQ) 1/2
= AD + BC
1/2
OR
𝟏 – cos 𝜽
Solution (a) LHS = 𝟏 + cos 𝜽
(𝟏 – cos 𝜽)𝟐
= 1
(𝟏 – cos 𝜽)(1 + cos 𝜽)
𝟏 cos 𝜽 𝟐
= (𝒔𝒊𝒏 𝜽 – ) 1/2
sin 𝜽
𝒄𝒐𝒔𝟐 𝑨 𝒔𝒊𝒏𝟐 𝑨
Solution (b)LHS = (1+ 𝒔𝒊𝒏𝟐 𝑨 ) (1+𝒄𝒐𝒔𝟐 𝑨) 1
𝒔𝒊𝒏𝟐 𝑨+ 𝒄𝒐𝒔𝟐 𝑨 𝒄𝒐𝒔𝟐 𝑨+ 𝒔𝒊𝒏𝟐 𝑨
=( )( )
𝒔𝒊𝒏𝟐 𝑨 𝒄𝒐𝒔𝟐 𝑨
1 1
= 1
sin 2 A cos 2 A
1 𝟏
= 2 2 𝟐
sin A (1 – sin A)
1 𝟏
= 2 4
= RHS 𝟐
sin A – sin A
-----------------------------------------------------------------------------------------------------------------
𝟑
= x = – 𝟐, = x = 5. 1
𝟑 𝟕 (– 7) − 𝒄𝒐𝒆𝒇𝒇𝒊𝒄𝒊𝒆𝒏𝒕 𝒐𝒇 𝒙
+=–𝟐+5=𝟐=– = 1/2
𝟐 𝒄𝒐𝒆𝒇𝒇𝒊𝒄𝒊𝒆𝒏𝒕 𝒐𝒇 𝒙𝟐
𝟑 𝟏𝟓 𝒄𝒐𝒏𝒔𝒕𝒂𝒏𝒕 𝒕𝒆𝒓𝒎
= – 𝟐 × 5 = – = 1/2
𝟐 𝒄𝒐𝒆𝒇𝒇𝒊𝒄𝒊𝒆𝒏𝒕 𝒐𝒇 𝒙𝟐
-----------------------------------------------------------------------------------------------------------------
𝟏
Solution Mid-point of AC = (𝟐 ,1) 1/2
𝟏
Mid-point of BD = ( ,1) 1/2
𝟐
Now AC = 9 + 4 = 13
and BD = 25 + 0 = 25 = 5
14
430/2/1 P.T.O
Let AB be the building and CD be the tower
60 60
In ∆BAC, tan 60 = x= = 20 3 _________ (i) 1+1/2
x 3
60 − h 20 3
In ∆BED, tan 30 = 60 – h = __________ (ii) 1+1/2
x √𝟑
OR
15
430/2/1 P.T.O
Solution Correct table 2
x – 35
Marks x f u= fu cf
10
0 – 10 5 3 –3 –9 3
10 – 20 15 5 –2 – 10 8
20 – 30 25 16 –1 – 16 24
30 – 40 35 12 0 0 36
40 – 50 45 13 1 13 49
50 – 60 55 20 2 40 69
60 – 70 65 6 3 18 75
70 – 80 75 5 4 20 80
80 56
𝟓𝟔
Mean = 35 + (10 × )= 42 1+1/2
𝟖𝟎
16
430/2/1 P.T.O
Solution (a) For figure 1
𝒂𝒓(𝜟 ADE) AE
Similarly = _________ (ii) 1/2
𝒂𝒓(𝜟 CDE) EC
BDE and CDE are on the same base DE and between the same parallel lines
BC and DE.
ar( BDE) = ar( CDE) _________ (iii) 1/2
From (i), (ii) and (iii)
𝑨𝑫 𝑨𝑬
= 𝑬𝑪 1/2
𝑫𝑩
-------------------------------------------------------------------------------------------------------
𝟕
Solution (a) (2a + 6d) = – 14 _________ (i) 1
𝟐
𝟏𝟏
(2a + 10d) = – 55 _________ (ii) 1
𝟐
3 5
Solving (i) and (ii) d = – , a= 1+1
2 2
n 3 n
Sn = [5 + (n – 1) (– )] = [13 – 3n] 1
2 2 4
17
430/2/1 P.T.O
OR
So, a1 = S1 = 3(1)2 + 1 = 4 1
S2 = a1 + a2 = 3(2)2 + 2 = 14 1
a2 = 10 1
Now a2 = a1 + d = 10 d = 6 1
a15 = a + 14d
= 4 + 14(6) = 88 1
----------------------------------------------------------------------------------------------------------------
SECTION E
18
430/2/1 P.T.O
1 1 2 1 √3 2 2
Solution (i) Total area of two segments = πr2 − r + πr2 − r = 256 1
4 2 6 4 3
1 1 1 √3 770
(ii) ( 4 π− 2 + 6 π − )r2 =
4 3
1
⇒ r = 26.1 cm (approx.)
1 1
(iii)(a) Area of segment with red roses = 4r2 − 2 r2 sq m 2
= 194.63 sq m (approx.)
OR
1
(iii)(b) Area of segment with yellow roses = 6r2 −
√3
r2 sq m 2
4
= 62.03 sq m (approx.)
Note: If the student has correctly written the area of two segments in part (i), then 2
marks to be awarded for part (iii), even if the student has not attempted
part (iii).
-----------------------------------------------------------------------------------------------------------------
19
430/2/1 P.T.O
𝟓
Solution (i) tan 30 = 𝑨𝑵 1/2
AN = 𝟓√𝟑 cm 1/2
20
430/2/1 P.T.O
OR
(iii) (b) ANO = 45 ⇒ AOB = 90 1/2
21
430/2/1 P.T.O
Solution (i) Volume of wood carved out to make one hollow
22 264
= ×2×2×3= cm3 or 37.7 cm3 1
7 7
(ii) LSA of cuboid = 2(14 x 4 + 17× 4) = 248 cm2. 1
22
430/2/1 P.T.O
Series WX1YZ/2 SET~2
J{UV (~w{Z`mXr)
MATHEMATICS (BASIC)
*
:3 : 80
Time allowed : 3 hours Maximum Marks : 80
NOTE :
(i) - 27
Please check that this question paper contains 27 printed pages.
(ii) - - -
-
Q.P. Code given on the right hand side of the question paper should be written on the title
page of the answer-book by the candidate.
(iii) - 38
Please check that this question paper contains 38 questions.
(iv) -
Please write down the serial number of the question in the answer-book before
attempting it.
(v) - 15 -
10.15 10.15 10.30 -
-
15 minute time has been allotted to read this question paper. The question paper will be
distributed at 10.15 a.m. From 10.15 a.m. to 10.30 a.m., the students will read the
question paper only and will not write any answer on the answer-book during this period.
SECTION A
(a) is inconsistent
9
(c) (d) 9
4
1
5. If the quadratic equation 9x2 + bx + = 0 has equal roots, then the
4
value of b is :
(a) 0 (b) 3 only
(c) 3 only (d) ±3
4 3
(a) r3 (b) r
3
2 3
(c) 3 r3 (d) r
3
(a) 2 (b) 3
(c) 4 (d) 5
(a) 27 (b) 29
29
(c) 18 (d)
3
25 4
(a) (b)
4 25
4 5
(c) (d)
5 4
10. In what ratio does x-axis divide the line segment joining the points
A(2, 3) and B(5, 6) ?
11. The sum of the first 21 terms of an A.P. : 16, 12, 8, 4, ..... is :
12. The area of a sector of angle (in degrees) of a circle with radius R is :
(a) 2 R (b) 2 R
180 360
2 2
(c) R (d) R
180 360
13. If the HCF of 72 and 234 is 18, then the LCM (72, 234) is :
(c) 65 (d) 90
19. Assertion (A) : The probability of getting a prime number, when a die is
2
thrown once, is .
3
SECTION B
This section comprises very short answer (VSA) type questions of 2 marks each.
21. (a)
Q and R are (6, 1), (1, 3) and (x, 8) respectively.
OR
(b) The vertices of a triangle are ( 2, 0), (2, 3) and (1, 3). Is the
triangle equilateral, isosceles or scalene ?
23. From a well-shuffled deck of 52 playing cards, all diamond cards are
removed. Now, a card is drawn from the remaining pack at random. Find
AM AB
24. In the given figure, LM || CB and LN || CD. Prove that .
AN AD
25. (a) Find the HCF of the numbers 540 and 630, using prime
factorization method.
OR
(b) Show that (15)n cannot end with the digit 0 for any natural
numb
This section comprises short answer (SA) type questions of 3 marks each.
OR
27. Find the zeroes of the polynomial p(x) = 3x2 + 5x 28 and verify the
relationship between its coefficients and zeroes.
28. Prove that the points A( 1, 0), B(3, 1), C(2, 2) and D( 2, 1) are the
vertices of a parallelogram ABCD. Is it also a rectangle ?
29. In the given figure, two concentric circles with centre O are shown. Radii
of the circles are 2 cm and 5 cm respectively. Find the area of the shaded
region.
irrational number.
OR
SECTION D
This section comprises long answer (LA) type questions of 5 marks each.
32. If a line is drawn parallel to one side of a triangle to intersect the other
two sides in distinct points, then prove that the other two sides are
divided in the same ratio.
17 terms is
OR
(b) A man repays a loan of < 3,250 by paying < 20 in the first month
and then increases the payment by < 15 every month. How long
34. Find the mean and the median of the following data :
35. (a) From the top of a building 60 m high, the angles of depression of
the top and bottom of a tower are observed to be 30 and 60
respectively. Find the height of the tower. Also, find the distance
between the building and the tower. (Use 3 = 1·732)
OR
(b) The angle of elevation of the top of a building from a point A on the
ground is 30 . On moving a distance of 30 m towards its base to the
(i) Find the volume of wood carved out to make one cylindrical hollow. 1
(ii) Find the lateral surface area of the cuboid to paint it with green
colour. 1
(iii) (a) Find the volume of wood in the remaining cuboid after
carving out seven cylindrical hollows. 2
OR
(iii) (b) Find the surface area of the top surface of the cuboid to be
painted yellow. 2
430/2/2 JJJJ Page 23 P.T.O.
Case Study 2
37. Circles play an important part in our life. When a circular object is hung
on the wall with a cord at nail N, the cords NA and NB work like
(iii) (a) Find the total length of cords NA, NB and the chord AB. 2
OR
38. Flower beds look beautiful growing in gardens. One such circular park of
radius r m, has two segments with flowers. One segment which subtends
an angle of 90 at the centre is full of red roses, while the other segment
with central angle 60 is full of yellow coloured flowers. [See figure]
Yellow flowers
Red roses
It is given that the combined area of the two segments (of flowers) is
2
256 sq m.
3
Based on the above, answer the following questions :
(i) Write an equation representing the total area of the two segments
in terms of r . 1
(ii) Find the value of 1
(iii) (a) Find the area of the segment with red roses. 2
OR
(iii) (b) Find the area of the segment with yellow flowers. 2
SECTION A
1
Answer (c) 1
0·5
--------------------------------------------------------------------------------------------------------------
Answer (d) 9 1
-----------------------------------------------------------------------------------------------------------
3
430/2/2 P.T.O
Answer (d) 3 1
----------------------------------------------------------------------------------------------------------
𝟒
Answer (b) 𝟑r3 1
-----------------------------------------------------------------------------------------------------------
Answer (a) 2 1
---------------------------------------------------------------------------------------------------------
Answer (a) 27 1
-------------------------------------------------------------------------------------------------------------
4
430/2/2 P.T.O
4
Answer (b) 1
25
---------------------------------------------------------------------------------------------------------------
Answer (d) 1 : 2 1
------------------------------------------------------------------------------------------------------------
𝜶
Answer (d) × πR2 1
𝟑𝟔𝟎
---------------------------------------------------------------------------------------------------------------
5
430/2/2 P.T.O
Answer (a) 2 cm 1
---------------------------------------------------------------------------------------------------------------
1
Answer (c) 26 1
-----------------------------------------------------------------------------------------------------------------
Answer (d) 8 1
-------------------------------------------------------------------------------------------------- ------------------
6
430/2/2 P.T.O
Answer (c) 65 1
---------------------------------------------------------------------------------------------------------------
7
430/2/2 P.T.O
SECTION B
Solution (a) PQ = QR ( 6 − 1) + ( −1 − 3) ( x − 1) + (8 − 3)
2 2 2 2
= 1
OR
AB = 4 2 + 32 = 5 1/2
BC = (– 1) 2 + (– 6) 2 = 37 1/2
CA = (1 + 2) 2 + (– 3) 2 = 3 2 1/2
-----------------------------------------------------------------------------------------------------------------
8
430/2/2 P.T.O
𝐴𝑁 AL
Solution In ACD, LN ⃦ CD 𝐴𝐷 = ________ (i) 1/2
AC
𝐴𝑀 AL
In ABC, LM ⃦ BC 𝐴𝐵 = ________ (ii) 1/2
AC
𝐴𝑁 AM
Using (i) and (ii) 𝐴𝐷 = 1/2
AB
𝐴𝑀 AB
𝐴𝑁 = 1/2
AD
--------------------------------------------------------------------------------------------------------------------
-----
630 = 2 × 32 × 5 × 7 1/2
HCF = 2 × 32 × 5 = 90 1
OR
9
430/2/2 P.T.O
𝟏 – cos 𝜽
Solution (a) LHS = 𝟏 + cos 𝜽
(𝟏 – cos 𝜽)𝟐
= (𝟏 – cos 𝜽)(1 + cos 𝜽) 1
𝟏 cos 𝜽 𝟐
= (𝒔𝒊𝒏 𝜽 – ) 1/2
sin 𝜽
OR
𝒄𝒐𝒔𝟐 𝑨 𝒔𝒊𝒏𝟐 𝑨
Solution (b)LHS = (1+ 𝒔𝒊𝒏𝟐 𝑨 ) (1+𝒄𝒐𝒔𝟐 𝑨) 1
𝒔𝒊𝒏𝟐 𝑨+ 𝒄𝒐𝒔𝟐 𝑨 𝒄𝒐𝒔𝟐 𝑨+ 𝒔𝒊𝒏𝟐 𝑨
=( )( )
𝑠𝑖𝑛2 𝐴 𝒄𝒐𝒔𝟐 𝑨
1 1
= 𝑠𝑖𝑛2 × 1
A cos2 A
1
= 𝑠𝑖𝑛2 1/2
A (1 – sin2 A)
1
= 𝑠𝑖𝑛2 = RHS 1/2
A – sin4 A
-----------------------------------------------------------------------------------------------------------------
7
Now taking = ,=–4
3
7 5 coeff. of x
+=3–4=–3= – 1/2
coeff . of x 2
7 28 const . term
= 3 (– 4) = – = 1/2
3
coeff . of x 2
--------------------------------------------------------------------------------------------------------------------
10
430/2/2 P.T.O
𝟏
Solution Mid-point of AC = (𝟐 ,1) 1/2
𝟏
Mid-point of BD = (𝟐 ,1) 1/2
Since Mid-point of AC = BD, therefore ABCD is a parallelogram. 1
Now AC = 9 + 4 = 13
and BD = 25 + 0 = 25 = 5
𝝅 × 5𝟐 × 60° 25
Solution Area of sector OABC = = cm2 1
360° 6
𝝅 × 2𝟐 × 60° 𝟒𝝅
Area of sector OED = = cm2 1
360° 𝟔
𝟐𝟓𝝅 𝟒𝝅 𝟐𝟏 22
Area of shaded region = – = × = 11 cm2 1
𝟔 𝟔 𝟔 𝟕
--------------------------------------------------------------------------------------------------------------------
-------
--------------------------------------------------------------------------------------------------------------------
11
430/2/2 P.T.O
Solution (a)
12
430/2/2 P.T.O
SECTION D
BDE and CDE are on the same base DE and between the same parallel
lines BC and DE.
ar( BDE) = ar( CDE) _________ (iii) 1/2
From (i), (ii) and (iii)
𝐴𝐷 𝐴𝐸
= 𝐸𝐶 1/2
𝐷𝐵
-------------------------------------------------------------------------------------------------------
7
Solution (a) [2a + 6d] = – 21 _______ (i) 1
2
17
[2a + 16d] = – 221 _______ (ii) 1
2
Solving (i) and (ii) d = – 2 and a = 3 1+1
n
Sn = [6 + (n – 1) (– 2)] 1/2
2
13
430/2/2 P.T.O
n
= (8 – 2n) or (4n – n2) 1/2
2
OR
14
430/2/2 P.T.O
5
Median = 95 + (35 – 22) = 99·3 1
15
------------------------------------------------------------------------------------------------------------------
60 − h 20 3
In ∆BED, tan 30 = 60 – h = __________ (ii) 1+1/2
x √𝟑
15
430/2/2 P.T.O
Let CD be the building
𝒉 1 h
In ∆DCA, tan 30 = = _________ (i) 1+1/2
x + 30
3 x + 30
h
In ∆DCB, tan 45 = h = x _________ (ii) 1
x
using equations (i) and (ii), h = x = 15 ( 3 + 1) 1/2
= 15 × 2·732 = 40.98 m
Height of building h = x = 40·98 m 1/2
Distance(d) of base from point A = x + 30 = 70.98 m 1/2
-----------------------------------------------------------------------------------------------------------------
SECTION E
16
430/2/2 P.T.O
Solution (i) Volume of wood carved out to make one hollow
22 264
= ×2×2×3= cm3 or 37.7 cm3 1
7 7
(ii) LSA of cuboid = 2(14 × 4 + 17× 4) = 248 cm2. 1
(iii)(a) Volume of 7 cylindrical hollows = 264 cm3. 1/2
Volume of original cuboid = 14 × 17 × 4 = 952 cm3. 1
Volume of remaining solid = 952 – 264 = 688 cm3. 1/2
OR
(iii) (b) Area of top surface to be painted = (l × b) – 7 × r2
22
= (14 × 17) – ( 7 × 4 × 7) 1
= 150 cm2 1
--------------------------------------------------------------------------------------------------------------
17
430/2/2 P.T.O
𝟓
Solution (i) tan 30 = 𝑨𝑵 1/2
AN = 5√3 cm 1/2
(ii) BNO = 30 ⇒ BNA = 60
AOB = 180 – 60 = 120 1
AN + NB + AB = 3 × 5 3 = 15 3 cm. 1/2
OR
(iii) (b) ANO = 45 ⇒ AOB = 90 1/2
Thus each angle of quad. AOBN is 90. 1
Also, OA = OB. ∴ OANB is a square. 1/2
--------------------------------------------------------------------------------------------------------------------
18
430/2/2 P.T.O
𝟏 𝟏 𝟏 √𝟑 2 𝟐
Solution (i) Total area of two segments = πr2 − 𝟐 r2 + 𝟔 πr2 − r = 256 𝟑 1
𝟒 𝟒
𝟏 𝟏 𝟏 √𝟑 𝟕𝟕𝟎
(ii) ( 𝟒 π− 𝟐 + 𝟔 π − )r2 = 1
𝟒 𝟑
⇒ r = 26.1 cm (approx.)
𝟏 𝟏
(iii)(a) Area of segment with red roses = 𝟒r2 − 𝟐 r2 sq m 2
= 194.63 sq m (approx.)
OR
𝟏
(iii)(b) Area of segment with yellow roses = 𝟔r2 −
√𝟑
r2 sq m 2
𝟒
= 62.03 sq m (approx.)
Note: If the student has correctly written the area of two segments in part (i), then 2
marks to be awarded for part (iii), even if the student has not attempted part (iii).
--------------------------------------------------------------------------------------------------------------------
19
430/2/2 P.T.O
Series WX1YZ/2 SET~3
J{UV (~w{Z`mXr)
MATHEMATICS (BASIC)
*
:3 : 80
Time allowed : 3 hours Maximum Marks : 80
NOTE :
(i) - 27
Please check that this question paper contains 27 printed pages.
(ii) - - -
-
Q.P. Code given on the right hand side of the question paper should be written on the title
page of the answer-book by the candidate.
(iii) - 38
Please check that this question paper contains 38 questions.
(iv) -
Please write down the serial number of the question in the answer-book before
attempting it.
(v) - 15 -
10.15 10.15 10.30 -
-
15 minute time has been allotted to read this question paper. The question paper will be
distributed at 10.15 a.m. From 10.15 a.m. to 10.30 a.m., the students will read the
question paper only and will not write any answer on the answer-book during this period.
SECTION A
2. The curved surface area of a cone of radius 7 cm is 550 cm2. Its slant
height is :
(a) 24 cm (b) 25 cm
(c) 14 cm (d) 20 cm
430/2/3 JJJJ Page 3 P.T.O.
3. Two coins are tossed together. The probability of getting atmost two
heads, is :
1 1
(a) (b)
2 4
3
(c) (d) 1
4
1
4. If the quadratic equation 9x2 + bx + = 0 has equal roots, then the
4
value of b is :
(a) 0 (b) 3 only
(c) 3 only (d) ±3
2 1 cos2 A
5. If tan A = , then the value of is :
5 1 sin2 A
25 4
(a) (b)
4 25
4 5
(c) (d)
5 4
6. Median and Mode of a distribution are 25 and 21 respectively. Mean of
the data using empirical relationship is :
(a) 27 (b) 29
29
(c) 18 (d)
3
7. The lines represented by the linear equations y = x and x = 4 intersect
at P. The coordinates of the point P are :
3 1 3
(c) ( 2 1) (d)
2 2 1
10. y-axis divides the line segment joining the points ( 6, 2) and (2, 6) in
the ratio :
(a) 1:3 (b) 3:2
(c) 3:1 (d) 2:3
(a) 9 (b) 0
1
(c) 8 (d)
9
12. The string of a kite in air is 50 m long and it makes an angle of 60 with
the horizontal. Assuming the string to be straight, the height of the kite
from the ground is :
100
(a) 50 3 m (b ) m
3
50
(c) m (d) 25 3 m
3
430/2/3 JJJJ Page 7 P.T.O.
13. The area of a sector of angle (in degrees) of a circle with radius R is :
(a) 2 R (b) 2 R
180 360
2 2
(c) R (d) R
180 360
14. From a point P, two tangents PQ and PR are drawn to a circle with
centre at O. T is a point on the major arc QR of the circle. If
QPR = 50 , then QTR equals :
(c) 65 (d) 90
(a) is inconsistent
(a) 2 (b) 3
(c) 4 (d) 5
4 3
(a) r3 (b) r
3
2 3
(c) 3 r3 (d) r
3
(a) Both Assertion (A) and Reason (R) are true and Reason (R) is the
correct explanation of the Assertion (A).
(b) Both Assertion (A) and Reason (R) are true, but Reason (R) is not
the correct explanation of the Assertion (A).
19. Assertion (A) : The probability of getting a prime number, when a die is
2
thrown once, is .
3
SECTION B
This section comprises very short answer (VSA) type questions of 2 marks each.
BE BC
23. In the given figure, DE || AC and . Prove that DC ||AP.
EC CP
24. (a) Find the HCF of the numbers 540 and 630, using prime
factorization method.
OR
n
(b) Show that (15) cannot end with the digit 0 for any natural
25. (a)
OR
(b) The vertices of a triangle are ( 2, 0), (2, 3) and (1, 3). Is the
triangle equilateral, isosceles or scalene ?
This section comprises short answer (SA) type questions of 3 marks each.
26. Show that the points A( 3, 2), B( 5, 5), C(2, 3) and D(4, 4) are
27. Find the zeroes of the polynomial p(x) = 2x2 7x 15 and verify the
1 cos 2
= (cosec cot )
1 cos
OR
1 1 1
1 1 =
tan 2 A cot 2 A sin2 A sin4 A
OR
30. In the given figure, two concentric circles with centre O are shown. Radii
of the circles are 2 cm and 5 cm respectively. Find the area of the shaded
region.
This section comprises long answer (LA) type questions of 5 marks each.
32. Find the mean and the median of the marks of 100 students of a class,
given in the following table :
Marks 0 5 5 10 10 15 15 20 20 25 25 30
Number of
4 11 13 15 31 26
students
33. (a) The sum of the first 8 terms of an A.P. is 100 and the sum of its
OR
(b) If the sum of the first p terms of an A.P. is the same as the sum of
its first q terms, (p q), then show that the sum of its first
(p + q) terms is zero.
34. (a) From the top of a building 60 m high, the angles of depression of
the top and bottom of a tower are observed to be 30 and 60
respectively. Find the height of the tower. Also, find the distance
between the building and the tower. (Use 3 = 1·732)
OR
(b) The angle of elevation of the top of a building from a point A on the
ground is 30 . On moving a distance of 30 m towards its base to the
point B, the angle of elevation changes to 45 . Find the height of
the building and the distance of its base from point A.
(Use 3 = 1·732)
35. If a line is drawn parallel to one side of a triangle to intersect the other
two sides in distinct points, then prove that the other two sides are
divided in the same ratio.
Case Study 1
36. Circles play an important part in our life. When a circular object is hung
on the wall with a cord at nail N, the cords NA and NB work like
(iii) (a) Find the total length of cords NA, NB and the chord AB. 2
OR
37. A wooden toy is shown in the picture. This is a cuboidal wooden block of
dimensions 14 cm 17 cm 4 cm. On its top there are seven cylindrical
hollows for bees to fit in. Each cylindrical hollow is of height 3 cm and
radius 2 cm.
(i) Find the volume of wood carved out to make one cylindrical hollow. 1
(ii) Find the lateral surface area of the cuboid to paint it with green
colour. 1
(iii) (a) Find the volume of wood in the remaining cuboid after
carving out seven cylindrical hollows. 2
OR
(iii) (b) Find the surface area of the top surface of the cuboid to be
painted yellow. 2
38. Flower beds look beautiful growing in gardens. One such circular park of
radius r m, has two segments with flowers. One segment which subtends
an angle of 90 at the centre is full of red roses, while the other segment
with central angle 60 is full of yellow coloured flowers. [See figure]
Yellow flowers
Red roses
It is given that the combined area of the two segments (of flowers) is
2
256 sq m.
3
Based on the above, answer the following questions :
(i) Write an equation representing the total area of the two segments
in terms of r . 1
(ii) Find the value of 1
(iii) (a) Find the area of the segment with red roses. 2
OR
(iii) (b) Find the area of the segment with yellow flowers. 2
SECTION A
Answer (b) 25 cm 1
---------------------------------------------------------------------------------------------------------
Answer (d) 1 1
-------------------------------------------------------------------------------------------------------
Answer (d) 3 1
----------------------------------------------------------------------------------------------------------
𝟒
Answer (b) 1
𝟐𝟓
--------------------------------------------------------------------------------------------------------------
-
430/2/3 3 P.T.O.
Answer (a) 27 1
-------------------------------------------------------------------------------------------------------------
𝟑
Answer (c) (√𝟐 – 1) 1
𝟐
Answer (c) 22 1
--------------------------------------------------------------------------------------------------------
430/2/3 4 P.T.O.
Answer.(c) 3 : 1 1
-----------------------------------------------------------------------------------------------------
Answer (a) 9 1
-------------------------------------------------------------------------------------------------------------
𝜶
Answer (d) × 𝛑𝐑𝟐 1
𝟑𝟔𝟎
--------------------------------------------------------------------------------------------------------------
430/2/3 5 P.T.O.
Answer (c) 65 1
--------------------------------------------------------------------------------------------------------------
-
𝟏
Answer (c) 1
𝟎·𝟓
--------------------------------------------------------------------------------------------------------------
Answer (a) 2 1
---------------------------------------------------------------------------------------------------------
430/2/3 6 P.T.O.
𝟒
Answer (b) 𝟑r3 1
-----------------------------------------------------------------------------------------------------------
----------------------------------------------------------------------------------------------------------
---------------------------------------------------------------------------------------------------------------
430/2/3 7 P.T.O.
SECTION B
Solution (k – 1) (– 3)2 + k (– 3) + 1 = 0 1
𝟒
k=𝟑 1
------------------------------------------------------------------------------------------------------
𝑩𝑬 BD
Solution In ABC, DE || AC = 1
𝑬𝑪 DA
𝑩𝑬 BC 𝑩𝑫 BC
Also given, = = 1/2
𝑬𝑪 CP 𝑫𝑨 CP
630 = 2 × 32 × 5 × 7 1/2
HCF = 2 × 32 × 5 = 90 1
430/2/3 8 P.T.O.
OR
For a number to end with zero it should have both 2 and 5 in its prime
factorization but 15n has only prime numbers 3 and 5 as its factors so it
can not end with zero. 1
----------------------------------------------------------------------------------------------------
OR
AB = √42 + 32 = 5 1/2
430/2/3 9 P.T.O.
SECTION C
Solution AB = √𝟒 + 49 = √𝟓𝟑
BC = √𝟒𝟗 + 4 = √𝟓𝟑
CD = √𝟒 + 49 = √𝟓𝟑
AD = √𝟒𝟗 + 4 = √𝟓𝟑
∴ AB = BC = CD = AD therefore ABCD is a rhombus. 2
Now AC = √𝟐𝟓 + 25 = 𝟓√𝟐
BD = √𝟖𝟏 + 81 = 𝟗√𝟐
As AC BD therefore it is not a square. 1
---------------------------------------------------------------------------------------------------------
--------------------------------------------------------------------------------------------------------------
430/2/3 10 P.T.O.
𝟏 – cos 𝜽
Solution (a) LHS = 𝟏 + cos 𝜽
(𝟏 – cos 𝜽)𝟐
= (𝟏 – cos 𝜽)(1 + cos 𝜽) 1
𝟏 cos 𝜽 𝟐
= (𝒔𝒊𝒏 𝜽 – ) 1/2
sin 𝜽
OR
𝒄𝒐𝒔𝟐 𝑨 𝒔𝒊𝒏𝟐 𝑨
Solution (b)LHS = (1+ 𝒔𝒊𝒏𝟐 𝑨 ) (1+𝒄𝒐𝒔𝟐 𝑨) 1
𝒔𝒊𝒏𝟐 𝑨+ 𝒄𝒐𝒔𝟐 𝑨 𝒄𝒐𝒔𝟐 𝑨+ 𝒔𝒊𝒏𝟐 𝑨
=( )( )
𝒔𝒊𝒏𝟐 𝑨 𝒄𝒐𝒔𝟐 𝑨
𝟏 𝟏
= 𝒔𝒊𝒏𝟐 × 1
A cos 𝟐 A
𝟏 𝟏
= 𝒔𝒊𝒏𝟐 A (1 – sin𝟐 A) 𝟐
𝟏 𝟏
= = RHS
𝒔𝒊𝒏𝟐 A – sin𝟒 A 𝟐
--------------------------------------------------------------------------------------------------------------
430/2/3 11 P.T.O.
Solution (a)
430/2/3 12 P.T.O.
𝝅 × 5𝟐 × 60° 25
Solution Area of sector OABC = = cm2 1
360° 6
𝝅 × 2𝟐 × 60° 𝟒𝝅
Area of sector OED = = cm2 1
360° 𝟔
𝟐𝟓𝝅 𝟒𝝅 𝟐𝟏 22
Area of shaded region = – = × = 11 cm2 1
𝟔 𝟔 𝟔 𝟕
-------------------------------------------------------------------------------------------------------
--------------------------------------------------------------------------------------------------------------
430/2/3 13 P.T.O.
SECTION D
136 1
Mean = 12·5 + 5 × = 19·3 1+
100 2
l= 20, cf =43, f=31
Median class : 20-25 1/2
5
Median = 20 + (50 – 43) = 21·1 1
31
---------------------------------------------------------------------------------------------------------
430/2/3 14 P.T.O.
8
Solution (a) (2a + 7d) = 100 _________ (i) 1
2
19
and (2a + 18d) = 551 _________ (ii) 1
2
Solving (i) and (ii), d = 3 and a = 2 1+1
n n
Sn = [4 + (n – 1) × 3] = (3n + 1) 1/2+1/2
2 2
OR
𝒑 𝒒
Solution [2a + (p – 1)d] = 𝟐 [2a + (q – 1) d] 1+1
𝟐
430/2/3 15 P.T.O.
Solution (a) For figure 1
60 − h 20 3
In ∆BED, tan 30 = 60 – h = __________ (ii) 1+1/2
x √𝟑
430/2/3 16 P.T.O.
Let CD be the building
𝒉 1 h
In ∆DCA, tan 30 = = _________ (i) 1+1/2
x + 30 3 x + 30
h
In ∆DCB, tan 45 = h = x _________ (ii) 1
x
using equations (i) and (ii), h = x = 15 ( 3 + 1) 1/2
= 15 × 2·732 = 40.98 m
Height of building h = x = 40·98 m 1/2
Distance(d) of base from point A = x + 30 = 70.98 m 1/2
-----------------------------------------------------------------------------------------------------------------
BDE and CDE are on the same base DE and between the same
parallel lines BC and DE.
ar( BDE) = ar( CDE) _________ (iii) 1/2
From (i), (ii) and (iii)
𝑨𝑫 𝑨𝑬
= 𝑬𝑪 1/2
𝑫𝑩
-------------------------------------------------------------------------------------------------------
430/2/3 17 P.T.O.
SECTION E
𝟓
Solution (i) tan 30 = 𝑨𝑵 1/2
AN = 𝟓√𝟑 cm 1/2
Hence, AB = 5 3 cm. 1
𝟏
AN + NB + AB = 3 × 5 3 = 15 3 cm. 𝟐
OR
430/2/3 18 P.T.O.
𝟏
(iii) (b) ANO = 45 ⇒ AOB = 90 𝟐
------------------------------------------------------------------------------------------------------------
430/2/3 19 P.T.O.
Solution (i) Volume of wood carved out to make one hollow
𝟐𝟐 𝟐𝟔𝟒
= ×2×2×3= cm3 or 37.7 cm3 1
𝟕 𝟕
= 150 cm2 1
--------------------------------------------------------------------------------------------------------------
430/2/3 20 P.T.O.
𝟏 𝟏 𝟏 √𝟑 2 𝟐
Solution (i) Total area of two segments = πr2 − 𝟐 r2 + 𝟔 πr2 − r = 256 𝟑 1
𝟒 𝟒
𝟏 𝟏 𝟏 √𝟑 𝟕𝟕𝟎
(ii) ( 𝟒 π− 𝟐 + 𝟔 π − )r2 = 1
𝟒 𝟑
⇒ r = 26.1 cm (approx.)
𝟏 𝟏
(iii)(a) Area of segment with red roses = 𝟒r2 − 𝟐 r2 sq m 2
= 194.63 sq m (approx.)
OR
𝟏
(iii)(b) Area of segment with yellow roses = 𝟔r2 −
√𝟑
r2 sq m 2
𝟒
= 62.03 sq m (approx.)
Note: If the student has correctly written the area of two segments in part (i), then 2
marks to be awarded for part (iii), even if the student has not attempted part (iii).
----------------------------------------------------------------------------------------------------------
430/2/3 21 P.T.O.
SET ~ 1
Series WX1YZ/4
àíZ-nÌ H$moS>
Q.P. Code 430/4/1
amob Z§. narjmWu àíZ-nÌ H$moS> H$mo CÎma-nwpñVH$m Ho$
Roll No. _wI-n¥ð> na Adí` {bI|Ÿ&
Candidates must write the Q.P. Code
on the title page of the answer-book.
J{UV (~w{Z`mXr)$
MATHEMATICS (BASIC)
#
{ZYm©[aV g_` … 3 KÊQ>o A{YH$V_ A§H$ … 80
Time allowed : 3 hours Maximum Marks : 80
ZmoQ> / NOTE :
(i) H¥$n`m Om±M H$a b| {H$ Bg àíZ-nÌ _| _w{ÐV n¥ð> 15 h¢Ÿ&
Please check that this question paper contains 15 printed pages.
(ii) àíZ-nÌ _| Xm{hZo hmW H$s Amoa {XE JE àíZ-nÌ H$moS> H$mo N>mÌ CÎma-nwpñVH$m Ho$ _wI-n¥ð> na
{bI|&
Q. P. Code given on the right hand side of the question paper should be
written on the title page of the answer-book by the candidate.
(iii) H¥$n`m Om±M H$a b| {H$ Bg àíZ-nÌ _| 38 àíZ h¢&
Please check that this question paper contains 38 questions.
(iv) H¥$n`m àíZ H$m CÎma {bIZm ewê$ H$aZo go nhbo, CÎma-nwpñVH$m _| àíZ H$m H«$_m§H$ Adí`
{bI|&
Please write down the Serial Number of the question in the answer-book
before attempting it.
(v) Bg àíZ-nÌ H$mo n‹T>Zo Ho$ {bE 15 {_ZQ> H$m g_` {X`m J`m h¡& àíZ-nÌ H$m {dVaU nydm©• _|
10.15 ~Oo {H$`m OmEJm& 10.15 ~Oo go 10.30 ~Oo VH$ N>mÌ Ho$db àíZ-nÌ H$mo n‹T>|Jo Am¡a
Bg Ad{Y Ho$ Xm¡amZ do CÎma-nwpñVH$m na H$moB© CÎma Zht {bI|Jo&
15 minute time has been allotted to read this question paper. The question
paper will be distributed at 10.15 a.m. From 10.15 a.m. to 10.30 a.m., the
students will read the question paper only and will not write any answer on
the answer-book during this period.
430/4/1 ~~~~ 1 [P.T.O.
General Instructions :
Read the following instructions carefully and follow them :
1. This question paper contains 38 questions. All questions are compulsory.
2. Question paper is divided into FIVE sections - Section A, B, C, D and E.
3. In section A, question number 1 to 18 are multiple choice questions (MCQs) and
question number 19 and 20 are Assertion - Reason based questions of 1 mark each.
4. In section B, question number 21 to 25 are very short answer (VSA) type questions
of 2 marks each.
5. In section C, question number 26 to 31 are short answer (SA) type questions
carrying 3 marks each.
6. In section D, question number 32 to 35 are long answer (LA) type questions
carrying 5 marks each.
7. In section E, question number 36 to 38 are case based integrated units of
assessment questions carrying 4 marks each. Internal choice is provided in 2
marks question in each case study.
8. There is no overall choice. However, an internal choice has been provided in 2
questions in Section B, 2 questions in Section C, 2 questions in Section D and 3
questions in Section E.
9. Draw neat figures wherever required. Take π = 22/7 wherever required if not
stated.
10. Use of calculators is not allowed.
SECTION - A
Section - A consists of Multiple Choice type questions of 1 mark each.
1. A quadratic polynomial the sum and product of whose zeroes are −3 and 2
respectively, is : 1
(a) x2 + 3x + 2 (b) x2 − 3x + 2 (c) x2 − 3x − 2 (d) x2 + 3x − 2
2. (HCF × LCM) for the numbers 70 and 40 is : 1
(a) 10 (b) 280 (c) 2800 (d) 70
3. If the radius of a semi-circular protractor is 7cm, then its perimeter is : 1
(a) 11 cm (b) 14 cm (c) 22 cm (d) 36 cm
4. The number (5 − 3 5 + 5 ) is : 1
(a) an integer (b) a rational number
(c) an irrational number (d) a whole number
5. If p (x) = x2 + 5x + 6, then p (− 2) is : 1
(a) 20 (b) 0 (c) − 8 (d) 8
6. Which of the following cannot be the probability of an event ? 1
5 1
(a) 0.1 (b) (c) 3% (d)
3 3
7. The pair of linear equations x + 2y + 5 = 0 and − 3x − 6y + 1 = 0 has : 1
(a) a unique solution (b) exactly two solutions
(c) infinitely many solutions (d) no solution
430/4/1 3 [P.T.O.
8. If ∆ABC ∼ ∆DEF and ∠A = 47o, ∠E = 83o, then ∠C is equal : 1
(a) 47o (b) 50o (c) 83o (d) 130o
11. The length of the arc of a circle of radius 14 cm which subtends an angle of
60o at the centre of the circle is : 1
44 88 308 616
(a) cm (b) cm (c) cm (d) cm
3 3 3 3
12. The angle of elevation of the top of a 30 m high tower at a point 30 m away
from the base of the tower is : 1
(a) 30o (b) 45o (c) 60o (d) 90o
15. A quadratic equation whose one root is 2 and the sum of whose roots is
zero, is : 1
(a) x2 + 4 = 0 (b) x2 − 2 = 0 (c) 4x2 − 1 = 0 (d) x2 − 4 = 0
18. The length of the tangent from an external point A to a circle, of radius
3 cm, is 4 cm. The distance of A from the centre of the circle is : 1
(a) 7 cm (b) 5 cm (c) 7 cm (d) 25 cm
430/4/1 5 [P.T.O.
(Assertion - Reason type questions)
In question numbers 19 and 20, a statement of Assertion (A) is followed by a
statement of Reason (R). Choose the correct option :
(a) Both Assertion (A) and Reason (R) are true and Reason (R) gives the
correct explanation of Assertion (A).
(b) Both Assertion (A) and Reason (R) are true but Reason (R) does not give
the correct explanation of Assertion (A).
(c) Assertion (A) is true but Reason (R) is false.
(d) Assertion (A) is false but Reason (R) is true.
19. Assertion (A) : A tangent to a circle is perpendicular to the radius through
the point of contact.
Reason (R) : The lengths of tangents drawn from an external point to a
circle are equal. 1
20. Assertion (A) : If one root of the quadratic equation 4x2 − 10x + (k − 4) = 0
is reciprocal of the other, then value of k is 8.
Reason (R) : Roots of the quadratic equation x2 − x + 1 = 0 are real. 1
SECTION - B
Section - B comprises of Very Short Answer (VSA) questions of 2 marks each.
1
21. If sinα = , then find the value of (3 cos α − 4 cos3 α). 2
2
22. (A) Find the coordinates of the point which divides the join of
A (−1, 7) and B (4, −3) in the ratio 2 : 3. 2
OR
(B) If the points A (2, 3), B (−5, 6), C (6, 7) and D (p, 4) are the vertices of
a parallelogram ABCD, find the value of p. 2
1
23. (A) Find the discriminant of the quadratic equation 3x2 − 2x + = 0 and
3
hence find the nature of its roots. 2
OR
(B) Find the roots of the quadratic equation x2 − x − 2 = 0. 2
430/4/1 7 [P.T.O.
25. In the adjoining figure, A, B and C are points on
OP, OQ and OR respectively such that AB||PQ and
AC||PR. Show that BC||QR. 2
SECTION - C
Section - C comprises of Short Answer (SA) type questions of 3 marks each.
26. Find the zeroes of the quadratic polynomial x2 + 6x + 8 and verify the
relationship between the zeroes and the coefficients. 3
1 + tan 2 A
27. Prove that 2
= sec2 A − 1 3
1 + cot A
28. (A) A lending library has a fixed charge for first three days and an
additional charge for each day thereafter. Rittik paid ` 27 for a book
kept for 7 days and Manmohan paid ` 21 for a book kept for 5 days.
Find the fixed charges and the charge for each extra day. 3
OR
(B) Find the values of ‘a’ and ‘b’ for which the system of linear equations
3x + 4y = 12, (a + b) x + 2 (a − b) y = 24
has infinite number of solutions. 3
30. Find the area of the sector of a circle of radius 7 cm and of central angle 90o.
Also, find the area of corresponding major sector. 3
430/4/1 9 [P.T.O.
SECTION - D
Section - D comprises of Long Answer (LA) type questions of 5 marks each.
33. (A) Find the sum of first 25 terms of the A.P. whose nth term is given
by an = 5 + 6n. Also, find the ratio of 20th term to 45th term. 5
OR
2
(B) In an A.P., if Sn = 3n + 5n and ak = 164, find the value of k. 5
34. The following table gives the monthly consumption of electricity of 100
families :
Monthly
Consumption 130-140 140-150 150-160 160-170 170-180 180-190 190-200
(in units)
Number of
5 9 17 28 24 10 7
families
Find the median of the above data. 5
430/4/1 11 [P.T.O.
SECTION - E
Section - E comprises of 3 Case Study / Passage Based questions of 4 marks
each.
36. Use of mobile screen for long hours makes your eye sight weak and give you
headaches. Children who are addicted to play “PUBG’’ can get easily
stressed out. To raise social awareness about ill effects of playing PUBG, a
school decided to start ‘BAN PUBG’ campaign, in which students are asked
to prepare campaign board in the shape of a rectangle. One such campaign
board made by class X student of the school is shown in the figure.
430/4/1 13 [P.T.O.
(b) If the cost of 1 dozen of bananas is ` 60, the cost of 1 apple is ` 15 and
cost of 1 mango is ` 20, find the total amount spent on 60 bananas,
36 apples and 42 mangoes. 2
38. Observe the figures given below carefully and answer the questions :
Figure A
A (i) A (ii)
Figure B
B (iii) B (iv)
Figure C
C (v) C (vi)
430/4/1 15 [P.T.O.
430/4/1
MARKING SCHEME
MATHEMATICS (BASIC)
SECTION A
Ans. (a) x2 + 3x + 2 1
__________________________________________________________________________
Ans. (d) 36 cm 1
__________________________________________________________________________
Ans. (b) 0 1
__________________________________________________________________________
5
Ans. (b) 1
3
__________________________________________________________________________
430/4/1 3 P.T.O.
Ans. (d) no solution 1
__________________________________________________________________________
Ans. (c) 3 1
__________________________________________________________________________
Ans. (c) 3 1
__________________________________________________________________________
44
Ans. (a) cm 1
3
__________________________________________________________________________
Ans. (c) 4 1
__________________________________________________________________________
430/4/1 4
1
Ans. (b) 1
26
__________________________________________________________________________
Ans. (d) x2 – 4 = 0 1
__________________________________________________________________________
Ans. (c) 2
2 x 3 + x2 = 3x2 – 5x 1
__________________________________________________________________________
Ans. (b) 5 cm 1
__________________________________________________________________________
430/4/1 5 P.T.O.
Ans. (b) Both Assertion (A) and Reason (R) are true but Reason (R) does not
give the correct explanation of Assertion (A) 1
__________________________________________________________________________
1 1
Solution: sin = = 30
2 2
3 cos – 4 cos3 = 3 cos 30 – 4 cos3 30
3
= 3 – 4
3
3
=
3 3
–
43 3 1
2 8
2 2
3 3 3 3 1
= – =0
2 2 2
__________________________________________________________________________
Solution:
2(4 ) 3(– 1) 8– 3
x= = =1 1
23 5
2(– 3) 3(7) 15
y= = =3 1
23 5
Coordinates of the required point are (1, 3)
OR
430/4/1 6
1
Solution: 3x2 – 2x + =0
3
1 1
a = 3, b = – 2, c =
3 2
1
Discriminant (D) = b2 – 4ac = (– 2)2 – 4(3) = 0 1
3
1
Roots are real and equal
2
OR
Solution: x2 – x – 2 = 0
(x – 2) (x + 1) = 0 1
1 1
x = 2, x = – 1 +
2 2
__________________________________________________________________________
Solution:
OTP = 90 (tangent radius at the point of contact) 1
Getting x = 120 1
__________________________________________________________________________
Solution: In POQ, AB PQ
430/4/1 7 P.T.O.
OA OB 1
= (By Thales Theorem) __________ (i)
AP BQ 2
In POR, AC PR
OA OC 1
= (By Thales Theorem)__________ (ii)
AP CR 2
OB OC 1
From (i) and (ii) =
BQ CR 2
1
In QOR, BC QR (By converse of Thales theorem)
2
__________________________________________________________________________
SECTION C
Solution: x2 + 6x + 8 = (x + 4) ( x + 2) 1
Zeroes are – 4, – 2 1
6 – Coeff. of x 1
Sum of zeroes = – 4 + (– 2) = – 6 = =
1 Coeff . of x 2 2
8 Constant term 1
Product of zeroes = (– 4) (– 2) = 8 = =
1 Coeff . of x 2 2
__________________________________________________________________________
sin 2 A
1
1 tan 2 A cos2 A
Solution: LHS = = 1
1 cot 2 A 1
cos2 A
sin 2 A
cos 2 A sin 2 A
cos 2 A 1
=
sin 2 A cos 2 A 2
sin 2 A
1
2 sin 2 A 1 - cos2 A
= cos A = 1
1 cos2 A cos2 A
sin 2 A
1 1
1 sec2 A 1 =RHS
cos2 A 2
__________________________________________________________________________
430/4/1 8
Solution: Let fixed charge be ₹ x
Let charge for each extra day be ₹ y
ATQ, x + 4y = 27 1
x + 2y = 21 1
1 1
On solving ; x = 15, y = 3 +
2 2
Fixed charge = ₹ 15
and charge for each extra day = ₹ 3
OR
a 1 b 1 c1
Solution: For Infinite number of solutions
a 2 b 2 c2
3 4 12
= = 1
a b 2a - b 24
3 1 1
= a+b=6
a b 2 2
2 1 1
= a–b=4
a–b 2 2
1 1
On solving, a = 5, b = 1 +
2 2
__________________________________________________________________________
Solution: S = {1, 2, 3, 4, 5, 6}
1
(i) P(an even prime number) = 1
6
2 1
(ii) P(a number greater than 4) = or 1
6 3
3 1
(iii) P(an odd number) = or 1
6 2
__________________________________________________________________________
430/4/1 9 P.T.O.
Solution: r = 7 cm, = 90
r 2 22 90
Area of sector = = 7 7 1
360 7 360
77 1
= cm2 or 38·5 cm2
2 2
22
Area of circle = r2 = 7 7 = 154 cm2 1
7
1
Area of major sector = 154 – 38·5 = 115·5 cm2
2
__________________________________________________________________________
Solution: 1
(For Fig.)
430/4/1 10
OP is the radius of smaller circle and AB is tangent at P.
AB is chord of larger circle and OP AB
AP = PB ( from centre bisects the chord) 1
2 2 2
In right AOP, AP = OA – OP
= (5)2 – (3)2 = 16
1
2
1
AP = 4 cm = PB
2
1
AB = 8 cm
2
__________________________________________________________________________
SECTION D
Solution:
1
(For Fig.)
h 1
In ABC, tan 60 = h= 3x 1+
x 2
h 1
In ABD, tan 30 = x + 40 = 3h 1+
x 40 2
1
Getting x = 20 m
2
1
and h= 20 3 m (Height of tower)
2
OR
Solution:
1
(For Fig.)
430/4/1 11 P.T.O.
7 1
In ABP, tan 45 = x=7 1+
x 2
h 1
In BCQ, tan 60 = h= 3x 1+
x 2
1
h=7 3m
2
Height of tower = PQ = 7 + h
1
= 7 + 7 3 = 7(1 + 3)m
2
__________________________________________________________________________
Solution: an = 5 + 6n
1
n = 1, a1(1st term) = 5 + 6(1) = 11
2
1
n = 2, a2 (2nd term) = 5 + 6(2) = 17
2
1
d = a2 – a1 = 17 – 11 = 6
2
n
Sn = [2a + (n – 1)d]
2
25
S25 = [2(11) + (25 – 1)6] 1
2
25
= [22 + 144]
2
1
S25 = 2075
2
a 20 5 6(20) 125 5
= = = 1+1
a 45 5 6(44) 275 11
The required ratio is 5:11
OR
Solution:
Sn = 3n2 + 5n
n = 1, S1(1st term) = 3(1)2 + 5(1) = 8 1
n = 2, S2(sum of 1st two terms) = 3(2)2 + 5(2)=22 1
1
a1 + a2 = 22 a2 = 22 – 8 = 14
2
d = a2 – a1 = 14 – 8 = 6 1
430/4/1 12
ak = 164
8 + (k – 1)6 = 164 1
1
k = 27
2
__________________________________________________________________________
Solution:
Monthly Number of
cf
Consumption families
130 – 140 5 5
140 – 150 9 14 1
150 – 160 17 31 (For Table.)
160 – 170 28 59
170 – 180 24 83
180 – 190 10 93
190 – 200 7 100
100
N
= 100 = 50
2 2
Median class is 160 – 170 1
l = 160, h = 10, cf = 31, f = 28 1
N
– cf
Median = l + 2 h
f
50 – 31 19 1
= 160 10 = 160 + 10 1
28 28 2
1
= 166.8
2
__________________________________________________________________________
430/4/1 13 P.T.O.
Solution:
7
h = 7 m, r = m
2
Total surface area = 2rh + 2(2r2) = 2r(h + 2r)
22 7 7 1
= 2 7 2 1
7 2 2 2
1
= 308 m2
2
2
Volume of the boiler = r 2 h 2 r 3
3
22 7 7 4 22 7 7 7 1
7 1
7 2 2 3 7 2 2 2 2
2695 3 1
m or 449.16 m 3
6 2
r 2h 1
=
2 2
r3
3
3h 3(7) 3 1
= = = or 3 : 1
2r 7 1 2
2
2
SECTION E
430/4/1 14
Solution:
We know that diagonals of a rectangle bisect each other.
1 7 1 5
(i) Required point = Mid-point of AC = , = (4, 3) 1
2 2
(ii) AC = (7 – 1)2 (5 – 1)2 = 2 13 1
1
(iii) (a) AB = (7 – 1)2 (1 – 1)2 = 6
2
1
BC = (7 – 7)2 (5 – 1)2 = 4
2
Area (ABCD) = AB BC = 6 4 = 24 1
OR
(iii) (b) AB = (7 – 1)2 (1 – 1)2 = 6 1
AB 6 3
= = 1
AC 2 13 13
required ratio is 3 : 13
__________________________________________________________________________
430/4/1 15 P.T.O.
Solution:
(i) HCF (36, 60) = 12 1
Khushi can invite at the most 12 guests
(ii) 36 12 = 3, 60 12 = 5
Each guest will get 3 apples and 5 bananas 1
430/4/1 16
Solution:
1 1
(i) Figure A and Figure C +
2 2
(ii) Figure C 1
(iii) (a) Triangles are congruent Corresponding angles are equal 1
1
Triangles are similar.
2
Conversely, if triangles are similar then ratio of corresponding
sides is same which does not imply corresponding sides are equal
1
Triangles may not be congruent.
2
Note: Any suitable counter example can be given
OR
(iii) (b) One pair of corresponding side must be equal 2
__________________________________________________________________________
430/4/1 17 P.T.O.
SET ~ 2
Series WX1YZ/4
àíZ-nÌ H$moS>
Q.P. Code 430/4/2
amob Z§. narjmWu àíZ-nÌ H$moS> H$mo CÎma-nwpñVH$m Ho$
Roll No. _wI-n¥ð> na Adí` {bI|Ÿ&
Candidates must write the Q.P. Code
on the title page of the answer-book.
J{UV (~w{Z`mXr)$
MATHEMATICS (BASIC)
#
{ZYm©[aV g_` … 3 KÊQ>o A{YH$V_ A§H$ … 80
Time allowed : 3 hours Maximum Marks : 80
ZmoQ> / NOTE :
(i) H¥$n`m Om±M H$a b| {H$ Bg àíZ-nÌ _| _w{ÐV n¥ð> 15 h¢Ÿ&
Please check that this question paper contains 15 printed pages.
(ii) àíZ-nÌ _| Xm{hZo hmW H$s Amoa {XE JE àíZ-nÌ H$moS> H$mo N>mÌ CÎma-nwpñVH$m Ho$ _wI-n¥ð> na
{bI|&
Q. P. Code given on the right hand side of the question paper should be
written on the title page of the answer-book by the candidate.
(iii) H¥$n`m Om±M H$a b| {H$ Bg àíZ-nÌ _| 38 àíZ h¢&
Please check that this question paper contains 38 questions.
(iv) H¥$n`m àíZ H$m CÎma {bIZm ewê$ H$aZo go nhbo, CÎma-nwpñVH$m _| àíZ H$m H«$_m§H$ Adí`
{bI|&
Please write down the Serial Number of the question in the answer-book
before attempting it.
(v) Bg àíZ-nÌ H$mo n‹T>Zo Ho$ {bE 15 {_ZQ> H$m g_` {X`m J`m h¡& àíZ-nÌ H$m {dVaU nydm©• _|
10.15 ~Oo {H$`m OmEJm& 10.15 ~Oo go 10.30 ~Oo VH$ N>mÌ Ho$db àíZ-nÌ H$mo n‹T>|Jo Am¡a
Bg Ad{Y Ho$ Xm¡amZ do CÎma-nwpñVH$m na H$moB© CÎma Zht {bI|Jo&
15 minute time has been allotted to read this question paper. The question
paper will be distributed at 10.15 a.m. From 10.15 a.m. to 10.30 a.m., the
students will read the question paper only and will not write any answer on
the answer-book during this period.
430/4/2 ~~~~ 1 [P.T.O.
General Instructions :
Read the following instructions carefully and follow them :
1. This question paper contains 38 questions. All questions are compulsory.
2. Question paper is divided into FIVE sections - Section A, B, C, D and E.
3. In section A, question number 1 to 18 are multiple choice questions (MCQs) and
question number 19 and 20 are Assertion - Reason based questions of 1 mark each.
4. In section B, question number 21 to 25 are very short answer (VSA) type questions
of 2 marks each.
5. In section C, question number 26 to 31 are short answer (SA) type questions
carrying 3 marks each.
6. In section D, question number 32 to 35 are long answer (LA) type questions
carrying 5 marks each.
7. In section E, question number 36 to 38 are case based integrated units of
assessment questions carrying 4 marks each. Internal choice is provided in 2
marks question in each case study.
8. There is no overall choice. However, an internal choice has been provided in 2
questions in Section B, 2 questions in Section C, 2 questions in Section D and 3
questions in Section E.
9. Draw neat figures wherever required. Take π = 22/7 wherever required if not
stated.
10. Use of calculators is not allowed.
SECTION - A
Section - A consists of Multiple Choice type questions of 1 mark each.
1
1. Let E be an event such that P(not E) = , then P(E) is equal to : 1
5
1 2 4
(a) (b) (c) 0 (d)
5 5 5
2. If p (x) = x2 + 5x + 6, then p (− 2) is : 1
(a) 20 (b) 0 (c) − 8 (d) 8
3. The mode of the numbers 2, 3, 3, 4, 5, 4, 4, 5, 3, 4, 2, 6, 7 is : 1
(a) 2 (b) 3 (c) 4 (d) 5
4. How many tangents can be drawn to a circle from a point on it ? 1
(a) One (b) Two (c) Infinite (d) Zero
5. A quadratic equation whose one root is 2 and the sum of whose roots is
zero, is : 1
(a) x2 + 4 = 0 (b) x2 − 2 = 0 (c) 4x2 − 1 = 0 (d) x2 − 4 = 0
6. Which of the following is not a quadratic equation ? 1
(a) 2(x −1)2 = 4x2 − 2x + 1 (b) 2x − x 2 = x2 + 5
(c) ( 2 x + 3 )2 + x2 = 3x2 − 5x (d) (x2 +2x)2 = x4 + 3 + 4x3
430/4/2 3 [P.T.O.
8. (HCF × LCM) for the numbers 30 and 70 is : 1
(a) 2100 (b) 21 (c) 210 (d) 70
2 o 4 o
12. 3 sin0 − 5 cos0 is equal to : 1
2 −4 −2
(a) (b) (c) 0 (d)
3 5 15
17. The length of the tangent from an external point A to a circle, of radius
3 cm, is 4 cm. The distance of A from the centre of the circle is : 1
(a) 7 cm (b) 5 cm (c) 7 cm (d) 25 cm
430/4/2 5 [P.T.O.
(Assertion - Reason type questions)
In question numbers 19 and 20, a statement of Assertion (A) is followed by a
statement of Reason (R). Choose the correct option :
(a) Both Assertion (A) and Reason (R) are true and Reason (R) gives the
correct explanation of Assertion (A).
(b) Both Assertion (A) and Reason (R) are true but Reason (R) does not give
the correct explanation of Assertion (A).
(c) Assertion (A) is true but Reason (R) is false.
(d) Assertion (A) is false but Reason (R) is true.
19. Assertion (A) : If one root of the quadratic equation 4x2 − 10x + (k − 4) = 0
is reciprocal of the other, then value of k is 8.
Reason (R) : Roots of the quadratic equation x2 − x + 1 = 0 are real. 1
SECTION - B
Section - B comprises of Very Short Answer (VSA) questions of 2 marks each.
1
21. (A) Find the discriminant of the quadratic equation 3x2 − 2x + = 0 and
3
hence find the nature of its roots. 2
OR
(B) Find the roots of the quadratic equation x2 − x − 2 = 0. 2
1
23. If sin α = , then find the value of (3 cos α − 4 cos3 α). 2
2
24. (A) Find the coordinates of the point which divides the join of
A (−1, 7) and B (4, −3) in the ratio 2 : 3. 2
OR
430/4/2 7 [P.T.O.
(B) If the points A (2, 3), B (−5, 6), C (6, 7) and D (p, 4) are the vertices of
a parallelogram ABCD, find the value of p. 2
SECTION - C
Section - C comprises of Short Answer (SA) type questions of 3 marks each.
26. Find the area of the sector of a circle of radius 7 cm and of central angle 90o.
Also, find the area of corresponding major sector. 3
1 + tan 2 A
29. Prove that = sec2 A − 1 3
1 + cot A
2
31. (A) If we add 1 to the numerator and subtract 1 from the denominator, a
1
fraction reduces to 1. It becomes if we only add 1 to the
2
denominator. What is the fraction ? 3
OR
(B) For which value of ‘k’ will the following pair of linear equations have
no solution ? 3
3x + y =1
(2k − 1) x + (k − 1) y = 2k + 1
430/4/2 9 [P.T.O.
SECTION - D
Section - D comprises of Long Answer (LA) type questions of 5 marks each.
32. (A) Find the sum of first 51 terms of an A.P. whose second and third terms
are 14 and 18, respectively. 5
OR
(B) The first term of an A.P. is 5, the last term is 45 and the sum is 400.
Find the number of terms and the common difference. 5
33. The distribution below gives the weights of 30 students of a class. Find the
median weight of the students : 5
Weight
40-45 45-50 50-55 55-60 60-65 65-70 70-75
in kg
Number
2 3 8 6 6 3 2
of Students
.
430/4/2 11 [P.T.O.
SECTION - E
Section - E comprises of 3 Case Study / Passage Based questions of 4 marks
each.
36. Observe the figures given below carefully and answer the questions :
Figure A
A (i) A (ii)
Figure B
B (iii) B (iv)
Figure C
C (v) C (vi)
(i) Name the figure(s) wherein two figures are similar. 1
(ii) Name the figure(s) wherein the figures are congruent. 1
(iii) (a) Prove that congruent triangles are also similar but not the
converse. 2
OR
(b) What more is least needed for two similar triangles to be
congruent ? 2
430/4/2 13 [P.T.O.
37. Use of mobile screen for long hours makes your eye sight weak and give you
headaches. Children who are addicted to play “PUBG’’ can get easily
stressed out. To raise social
awareness about ill effects of
playing PUBG, a school decided
to start ‘BAN PUBG’ campaign,
in which students are asked to
prepare campaign board in the
shape of a rectangle. One such
campaign board made by class X
student of the school is shown in
the figure.
430/4/2 15 [P.T.O.
430/4/2
MARKING SCHEME
MATHEMATICS (BASIC)
SECTION A
4
Ans. (d) 1
5
__________________________________________________________________________
Ans. (b) 0 1
__________________________________________________________________________
Ans. (c) 4 1
__________________________________________________________________________
Ans. (d) x2 – 4 = 0 1
__________________________________________________________________________
Ans. (c) 2
2 x 3 + x2 = 3x2 – 5x 1
__________________________________________________________________________
430/4/2 3 P.T.O.
Ans. (b) x2 – 2x – 1 1
__________________________________________________________________________
44
Ans. (a) cm 1
3
__________________________________________________________________________
Ans. (d) 36 cm 1
__________________________________________________________________________
4
Ans. (b) – 1
5
__________________________________________________________________________
430/4/2 4
1
Ans. (a) 1
52
__________________________________________________________________________
Ans. (c) 3 1
__________________________________________________________________________
Ans. (b) 5 cm 1
__________________________________________________________________________
430/4/2 5 P.T.O.
Ans. (c) Assertion (A) is true but Reason (R) is false 1
__________________________________________________________________________
Ans. (b) Both Assertion (A) and Reason (R) are true but Reason (R) does not
give the correct explanation of Assertion (A) 1
__________________________________________________________________________
SECTION B
1
Solution: 3x2 – 2x + =0
3
1 1
a = 3, b = – 2, c =
3 2
1
Discriminant (D) = b2 – 4ac = (– 2)2 – 4(3) = 0 1
3
1
Roots are real and equal
2
OR
Solution: x2 – x – 2 = 0
(x – 2) (x + 1) = 0 1
1 1
x = 2, x = – 1 +
2 2
__________________________________________________________________________
430/4/2 6
Solution: In POQ, AB PQ
OA OB 1
= (By Thales Theorem) __________ (i)
AP BQ 2
In POR, AC PR
OA OC 1
= (By Thales Theorem)__________ (ii)
AP CR 2
OB OC 1
From (i) and (ii) =
BQ CR 2
1
In QOR, BC QR (By converse of Thales theorem)
2
__________________________________________________________________________
1 1
Solution: sin = = 30
2 2
3 cos – 4 cos3 = 3 cos 30 – 4 cos3 30
3
= 3
3
– 4
3
=
3 3
–
43 3 1
8
2 2 2
3 3 3 3 1
= – =0
2 2 2
__________________________________________________________________________
Solution:
2(4 ) 3(– 1) 8– 3
x= = =1 1
23 5
2(– 3) 3(7) 15
y= = =3 1
23 5
Coordinates of the required point are (1, 3)
OR
430/4/2 7 P.T.O.
2 6 3 7 – 5 p 6 4
, = , 1
2 2 2 2
–5p
= 4 p = 13 1
2
__________________________________________________________________________
430/4/2 8
, are zeroes of p(x)
– (– 5) 1
+ = =5
1 2
1
= 6
2
1 1 5
Sum of zeroes of the req. polynomial = + = = 1
6
1 1 1 1 1
Product of zeroes of the req. polynomial = . = =
6 2
Required Polynomial is
5 1 5 1 1
k x 2 – x or x2 – x + or 6x2 –5x + 1
6 6 6 6 2
__________________________________________________________________________
Solution: S = {1, 2, 3, 4, 5, 6}
1
(i) P(an even prime number) = 1
6
2 1
(ii) P(a number greater than 4) = or 1
6 3
3 1
(iii) P(an odd number) =or 1
6 2
__________________________________________________________________________
sin 2 A
1
1 tan 2 A cos2 A
Solution: LHS = = 1
1 cot 2 A 1
cos2 A
sin 2 A
cos 2 A sin 2 A
cos 2 A 1
=
sin A cos 2 A
2
2
sin 2 A
430/4/2 9 P.T.O.
1
cos 2
A sin 2 A 1 - cos2 A
= = 1
1 cos2 A cos2 A
sin 2 A
1 1
2
1 sec2 A 1 =RHS
cos A 2
__________________________________________________________________________
Solution: 1
(For Fig.)
430/4/2 10
In right AOP, AP2 = OA2 – OP2
= (5)2 – (3)2 = 16
1
2
1
AP = 4 cm = PB
2
1
AB = 8 cm
2
__________________________________________________________________________
x 1
Solution: Let the fraction be
y 2
x 1 1
ATQ, =1 x–y=–2 __________ (i)
y–1 2
x 1 1
Also, = 2x – y = 1 __________ (ii)
y 1 2 2
Solving (i) and (ii) x = 3, y = 5 1
3 1
Fraction =
5 2
OR
430/4/2 11 P.T.O.
Solution: (a) a2 = 14 a + d = 14 1
a3 = 18 a + 2d = 18 1
1 1
d = 4, a = 10 +
2 2
51
S51 = [2(10) + (51 – 1)4] 1
2
51 51
= [20 + 200] = 220 = 5610 1
2 2
OR
Solution:
Weight(in kg) 40 – 45 45 – 50 50 – 55 55 – 60 60 – 65 65 – 70 70 – 75
No. of students 2 3 8 6 6 3 2
c.f. 2 5 13 19 25 28 30
1 1
(For Table)
430/4/2 12
N
= 30 = 15
2 2
Median class = 55 – 60 1
l = 55, h = 5, cf = 13, f = 6
N
– cf
Median = l 2 h
f
15 – 13
= 55 5 2
6
= 56· 67 1
__________________________________________________________________________
Solution:
7
h = 7 m, r = m
2
Total surface area = 2rh + 2(2r2) = 2r(h + 2r)
22 7 7 1
= 2 7 2 1
7 2 2 2
1
= 308 m2
2
2
Volume of the boiler = r 2 h 2 r 3
3
22 7 7 4 22 7 7 7 1
7 1
7 2 2 3 7 2 2 2 2
2695 3 1
m or 449.16 m 3
6 2
430/4/2 13 P.T.O.
r 2h 1
=
2 2
r3
3
3h 3(7) 3 1
= = = or 3 : 1
2r 7 1 2
2
2
__________________________________________________________________________
Solution:
1
(For Fig.)
h 1
In ABC, tan 60 = h= 3x 1+
x 2
h 1
In ABD, tan 30 = x + 40 = 3h 1+
x 40 2
1
Getting x = 20 m
2
1
and h= 20 3 m (Height of tower)
2
OR
Solution:
1
(For Fig.)
430/4/2 14
7 1
In ABP, tan 45 = x=7 1+
x 2
h 1
In BCQ, tan 60 = h= 3x 1+
x 2
1
h=7 3m
2
Height of tower = PQ = 7 + h
1
= 7 + 7 3 = 7(1 + 3)m
2
__________________________________________________________________________
SECTION D
430/4/2 15 P.T.O.
Solution:
1 1
(i) Figure A and Figure C +
2 2
(ii) Figure C 1
(iii) (a) Triangles are congruent Corresponding angles are equal 1
1
Triangles are similar.
2
Conversely, if triangles are similar then ratio of corresponding
sides is same which does not imply corresponding sides are equal
1
Triangles may not be congruent.
2
Note: Any suitable counter example can be given
OR
(iii) (b) One pair of corresponding side must be equal 2
__________________________________________________________________________
430/4/2 16
Solution:
We know that diagonals of a rectangle bisect each other.
1 7 1 5
(i) Required point = Mid-point of AC = , = (4, 3) 1
2 2
(ii) AC = (7 – 1)2 (5 – 1)2 = 2 13 1
1
(iii) (a) AB = (7 – 1)2 (1 – 1)2 = 6
2
1
BC = (7 – 7)2 (5 – 1)2 = 4
2
Area (ABCD) = AB BC = 6 4 = 24 1
OR
(iii) (b) AB = (7 – 1)2 (1 – 1)2 = 6 1
AB 6 3
= = 1
AC 2 13 13
required ratio is 3 : 13
__________________________________________________________________________
Solution:
(i) HCF (36, 60) = 12 1
Khushi can invite at the most 12 guests
430/4/2 17 P.T.O.
(ii) 36 12 = 3, 60 12 = 5
Each guest will get 3 apples and 5 bananas 1
430/4/2 18
SET ~ 3
Series WX1YZ/4
àíZ-nÌ H$moS>
Q.P. Code 430/4/3
amob Z§. narjmWu àíZ-nÌ H$moS> H$mo CÎma-nwpñVH$m Ho$
Roll No. _wI-n¥ð> na Adí` {bI|Ÿ&
Candidates must write the Q.P. Code
on the title page of the answer-book.
J{UV (~w{Z`mXr)$
MATHEMATICS (BASIC)
#
{ZYm©[aV g_` … 3 KÊQ>o A{YH$V_ A§H$ … 80
Time allowed : 3 hours Maximum Marks : 80
ZmoQ> / NOTE :
(i) H¥$n`m Om±M H$a b| {H$ Bg àíZ-nÌ _| _w{ÐV n¥ð> 15 h¢Ÿ&
Please check that this question paper contains 15 printed pages.
(ii) àíZ-nÌ _| Xm{hZo hmW H$s Amoa {XE JE àíZ-nÌ H$moS> H$mo N>mÌ CÎma-nwpñVH$m Ho$ _wI-n¥ð> na
{bI|&
Q. P. Code given on the right hand side of the question paper should be
written on the title page of the answer-book by the candidate.
(iii) H¥$n`m Om±M H$a b| {H$ Bg àíZ-nÌ _| 38 àíZ h¢&
Please check that this question paper contains 38 questions.
(iv) H¥$n`m àíZ H$m CÎma {bIZm ewê$ H$aZo go nhbo, CÎma-nwpñVH$m _| àíZ H$m H«$_m§H$ Adí`
{bI|&
Please write down the Serial Number of the question in the answer-book
before attempting it.
(v) Bg àíZ-nÌ H$mo n‹T>Zo Ho$ {bE 15 {_ZQ> H$m g_` {X`m J`m h¡& àíZ-nÌ H$m {dVaU nydm©• _|
10.15 ~Oo {H$`m OmEJm& 10.15 ~Oo go 10.30 ~Oo VH$ N>mÌ Ho$db àíZ-nÌ H$mo n‹T>|Jo Am¡a
Bg Ad{Y Ho$ Xm¡amZ do CÎma-nwpñVH$m na H$moB© CÎma Zht {bI|Jo&
15 minute time has been allotted to read this question paper. The question
paper will be distributed at 10.15 a.m. From 10.15 a.m. to 10.30 a.m., the
students will read the question paper only and will not write any answer on
the answer-book during this period.
430/4/3 ~~~~ 1 [P.T.O.
General Instructions :
Read the following instructions carefully and follow them :
1. This question paper contains 38 questions. All questions are compulsory.
2. Question paper is divided into FIVE sections - Section A, B, C, D and E.
3. In section A, question number 1 to 18 are multiple choice questions (MCQs) and
question number 19 and 20 are Assertion - Reason based questions of 1 mark each.
4. In section B, question number 21 to 25 are very short answer (VSA) type questions
of 2 marks each.
5. In section C, question number 26 to 31 are short answer (SA) type questions
carrying 3 marks each.
6. In section D, question number 32 to 35 are long answer (LA) type questions
carrying 5 marks each.
7. In section E, question number 36 to 38 are case based integrated units of
assessment questions carrying 4 marks each. Internal choice is provided in 2
marks question in each case study.
8. There is no overall choice. However, an internal choice has been provided in 2
questions in Section B, 2 questions in Section C, 2 questions in Section D and 3
questions in Section E.
9. Draw neat figures wherever required. Take π = 22/7 wherever required if not
stated.
10. Use of calculators is not allowed.
SECTION - A
Section - A consists of Multiple Choice type questions of 1 mark each.
1. The value of k for which the equations 3x − y + 8 = 0 and 6x − ky + 16 = 0
represent coincident lines is : 1
1 1
(a) (b) − (c) 2 (d) − 2
2 2
2. A circle of radius 5.2 cm has two tangents AB and CD parallel to each
other. What is the distance between the two tangents ? 1
(a) 5.2 cm (b) 10.4 cm (c) 20.8 cm (d) can’t find
3. The number of polynomials having zeroes − 3 and 4 is : 1
(a) 1 (b) 2 (c) 3 (d) more than 3
4. If the perimeter and the area of a circle are numerically equal, then the
radius of the circle is : 1
(a) 2 units (b) π units (c) 4 units (d) 2 π units
5. A quadratic equation whose one root is 2 and the sum of whose roots is
zero, is : 1
(a) x2 + 4 = 0 (b) x2 − 2 = 0 (c) 4x2 − 1 = 0 (d) x2 − 4 = 0
6. Which of the following is not a quadratic equation ? 1
(a) 2(x −1)2 = 4x2 − 2x + 1 (b) 2x − x 2 = x2 + 5
(c) ( 2 x + 3 )2 + x2 = 3x2 − 5x (d) (x2 +2x)2 = x4 + 3 + 4x3
430/4/3 3 [P.T.O.
7. How many tangents can be drawn to a circle from a point on it ? 1
(a) One (b) Two (c) Infinite (d) Zero
17. What is the length of arc of a circle of radius 7 cm which subtends an angle
of 90o at the centre of the circle ? 1
77 11
(a) 22 cm (b) 11 cm (c) cm (d) cm
2 2
430/4/3 5 [P.T.O.
(Assertion - Reason type questions)
In question numbers 19 and 20, a statement of Assertion (A) is followed by a
statement of Reason (R). Choose the correct option :
(a) Both Assertion (A) and Reason (R) are true and Reason (R) gives the
correct explanation of Assertion (A).
(b) Both Assertion (A) and Reason (R) are true but Reason (R) does not give
the correct explanation of Assertion (A).
(c) Assertion (A) is true but Reason (R) is false.
(d) Assertion (A) is false but Reason (R) is true.
19. Assertion (A) : A tangent to a circle is perpendicular to the radius through
the point of contact.
Reason (R) : The lengths of tangents drawn from an external point to a
circle are equal. 1
20. Assertion (A) : If one root of the quadratic equation 4x2 − 10x + (k − 4) = 0
is reciprocal of the other, then value of k is 8.
Reason (R) : Roots of the quadratic equation x2 − x + 1 = 0 are real. 1
SECTION - B
Section - B comprises of Very Short Answer (VSA) questions of 2 marks each.
1
21. (A) Find the discriminant of the quadratic equation 3x2 − 2x + = 0 and
3
hence find the nature of its roots. 2
OR
(B) Find the roots of the quadratic equation x2 − x − 2 = 0. 2
430/4/3 7 [P.T.O.
24. (A) Find the coordinates of the point which divides the join of
A (−1, 7) and B (4, −3) in the ratio 2 : 3. 2
OR
(B) If the points A (2, 3), B (−5, 6), C (6, 7) and D (p, 4) are the vertices
of a parallelogram ABCD, find the value of p. 2
SECTION - C
Section - C comprises of Short Answer (SA) type questions of 3 marks each.
1 + sec θ sin 2 θ
26. Prove that = 3
sec θ 1 − cos θ
28. Find the zeroes of the quadratic polynomial x2 + 6x + 8 and verify the
relationship between the zeroes and the coefficients. 3
29. Find the area of the sector of a circle of radius 7 cm and of central angle
90o. Also, find the area of corresponding major sector. 3
30. (A) Prove that the lengths of tangents drawn from an external point to a
circle are equal. 3
OR
(B) Two concentric circles with centre O are of
radii 3 cm and 5 cm. Find the length of chord
AB of the larger circle which touches the
smaller circle at P. 3
430/4/3 9 [P.T.O.
31. (A) A lending library has a fixed charge for first three days and an
additional charge for each day thereafter. Rittik paid ` 27 for a book
kept for 7 days and Manmohan paid ` 21 for a book kept for 5 days.
Find the fixed charges and the charge for each extra day. 3
OR
(B) Find the values of ‘a’ and ‘b’ for which the system of linear equations
3x + 4y = 12, (a + b) x + 2 (a − b) y = 24
has infinite number of solutions. 3
SECTION - D
Section - D comprises of Long Answer (LA) type questions of 5 marks each.
33. The following table gives the monthly consumption of electricity of 100
families :
Monthly
Consumption 130-140 140-150 150-160 160-170 170-180 180-190 190-200
(in units)
Number of
5 9 17 28 24 10 7
families
Find the median of the above data. 5
34. (A) The first term of an A.P. is − 5 and the last term is 45. If the sum of all
the terms of the A.P. is 120, find the number of terms and the common
difference. 5
OR
(B) If the sum of first 7 terms of an A.P. is 49 and that of first 17 terms is
289, find the sum of first n terms. 5
430/4/3 11 [P.T.O.
35. (A) As observed from the top of a 75 m high light house from the sea-
level, the angles of depression of two ships are 30o and 45o. If one ship
is exactly behind the other on the same side of the light house, find the
distance between the two ships. (use 3 = 1.73) 5
OR
(B) From a point P on the ground, the angle of elevation of the top of a
10 m tall building is 30o. A flag is hoisted at the top of the building and
o
the angle of elevation of the top of the flagstaff from P is 45 . Find the
length of the flagstaff and the distance of the building from the point P.
(use 3 = 1.73) 5
SECTION - E
Section - E comprises of 3 Case Study / Passage Based questions of 4 marks
each.
37. Observe the figures given below carefully and answer the questions :
Figure A
A (i) A (ii)
430/4/3 13 [P.T.O.
Figure B
B (iii) B (iv)
Figure C
C (v) C (vi)
(i) Name the figure(s) wherein two figures are similar. 1
(ii) Name the figure(s) wherein the figures are congruent. 1
(iii) (a) Prove that congruent triangles are also similar but not the converse. 2
OR
(b) What more is least needed for two similar triangles to be
congruent ? 2
38. Use of mobile screen for long hours
makes your eye sight weak and give you
headaches. Children who are addicted to
play “PUBG’’ can get easily stressed out.
To raise social awareness about ill effects
of playing PUBG, a school decided to
start ‘BAN PUBG’ campaign, in which
students are asked to prepare campaign
board in the shape of a rectangle. One
such campaign board made by class X student of the school is shown in the
figure.
Based on the above information, answer the following questions :
(i) Find the coordinates of the point of intersection of diagonals AC and
BD. 1
(ii) Find the length of the diagonal AC. 1
(iii) (a) Find the area of the campaign Board ABCD. 2
OR
(b) Find the ratio of the length of side AB to the length of the
diagonal AC. 2
430/4/3 15 [P.T.O.
430/4/3
MARKING SCHEME
MATHEMATICS (BASIC)
SECTION A
Ans. (c) 2 1
__________________________________________________________________________
Ans. (d) x2 – 4 = 0 1
__________________________________________________________________________
Ans. (c) 2
2 x 3 + x2 = 3x2 – 5x 1
__________________________________________________________________________
430/4/3 3 P.T.O.
Ans. (a) One 1
__________________________________________________________________________
5
Ans. (b) 1
3
__________________________________________________________________________
Ans. (c) 4 1
__________________________________________________________________________
Ans. (c) 3 1
__________________________________________________________________________
Ans. (b) 0 1
__________________________________________________________________________
430/4/3 4
Ans. (b) 50 1
__________________________________________________________________________
1
Ans. (b) 1
26
__________________________________________________________________________
Ans. (b) 11 cm 1
__________________________________________________________________________
1
Ans. (c) – 1
4
__________________________________________________________________________
Ans. (b) Both Assertion (A) and Reason (R) are true but Reason (R) does not
give the correct explanation of Assertion (A) 1
__________________________________________________________________________
430/4/3 5 P.T.O.
Ans. (c) Assertion (A) is true but Reason (R) is false 1
__________________________________________________________________________
SECTION B
1
Solution: 3x2 – 2x + =0
3
1 1
a = 3, b = – 2, c =
3 2
1
Discriminant (D) = b2 – 4ac = (– 2)2 – 4(3) = 0 1
3
1
Roots are real and equal
2
OR
Solution: x2 – x – 2 = 0
(x – 2) (x + 1) = 0 1
1 1
x = 2, x = – 1 +
2 2
__________________________________________________________________________
Solution: In POQ, AB PQ
OA OB 1
= (By Thales Theorem) __________ (i)
AP BQ 2
In POR, AC PR
OA OC 1
= (By Thales Theorem)__________ (ii)
AP CR 2
430/4/3 6
OB OC 1
From (i) and (ii) =
BQ CR 2
1
In QOR, BC QR (By converse of Thales theorem)
2
__________________________________________________________________________
Solution:
OTP = 90 (tangent radius at the point of contact) 1
Getting x = 120 1
__________________________________________________________________________
Solution:
2(4 ) 3(– 1) 8– 3
x= = =1 1
23 5
2(– 3) 3(7) 15
y= = =3 1
23 5
Coordinates of the required point are (1, 3)
OR
430/4/3 7 P.T.O.
3 1 1 1
Solution: sin P = , cos P = +
2 2 2 2
3 1 3
2 sin P cos P = 2 =
1
2 2 2
__________________________________________________________________________
SECTION C
1
1
1 sec cos = 1 + cos
Solution: LHS = = 1
sec 1
cos
(1 – cos ) (1 cos )
= 1
(1 – cos )
1 – cos2 sin 2
= = = RHS 1
1 – cos 1 – cos
__________________________________________________________________________
Solution: x2 + 6x + 8 = (x + 4) ( x + 2) 1
Zeroes are – 4, – 2 1
6 – Coeff. of x 1
Sum of zeroes = – 4 + (– 2) = – 6 = =
1 Coeff . of x 2 2
430/4/3 8
8 Constant term 1
Product of zeroes = (– 4) (– 2) = 8 ==
1 Coeff . of x 2 2
__________________________________________________________________________
Solution: 1
(For Fig.)
430/4/3 9 P.T.O.
Solution: Join OA and OP
1
OP AB (radius tangent at the point of contact)
2
OP is the radius of smaller circle and AB is tangent at P.
AB is chord of larger circle and OP AB
AP = PB ( from centre bisects the chord) 1
2 2 2
In right AOP, AP = OA – OP
= (5)2 – (3)2 = 16
1
2
1
AP = 4 cm = PB
2
1
AB = 8 cm
2
__________________________________________________________________________
a 1 b 1 c1
Solution: For Infinite number of solutions
a 2 b 2 c2
430/4/3 10
3 4 12
= = 1
a b 2a - b 24
3 1 1
= a+b=6
a b 2 2
2 1 1
= a–b=4
a–b 2 2
1 1
On solving, a = 5, b = 1 +
2 2
__________________________________________________________________________
SECTION D
Solution:
7
h = 7 m, r = m
2
Total surface area = 2rh + 2(2r2) = 2r(h + 2r)
22 7 7 1
= 2 7 2 1
7 2 2 2
1
= 308 m2
2
2
Volume of the boiler = r 2 h 2 r 3
3
22 7 7 4 22 7 7 7 1
7 1
7 2 2 3 7 2 2 2 2
2695 3 1
m or 449.16 m 3
6 2
430/4/3 11 P.T.O.
r 2h 1
=
2 2
r3
3
3h 3(7) 3 1
= = = or 3 : 1
2r 7 1 2
2
2
__________________________________________________________________________
Solution:
Monthly Number of
cf
Consumption families
130 – 140 5 5
140 – 150 9 14 1
150 – 160 17 31 (For Table.)
160 – 170 28 59
170 – 180 24 83
180 – 190 10 93
190 – 200 7 100
100
N
= 100 = 50
2 2
Median class is 160 – 170 1
l = 160, h = 10, cf = 31, f = 28 1
N
– cf
Median = l + 2 h
f
50 – 31 19 1
= 160 10 = 160 + 10 1
28 28 2
1
= 166.8
2
430/4/3 12
__________________________________________________________________________
Solution: a = – 5, an = 45
a + (n – 1)d = 45
(n – 1)d = 50 1
n
Sn = 120 [2a + (n – 1)d] = 120
2
n
[2(– 5) + 50] = 120 2
2
n
40 = 120 n = 6 1
2
(n – 1)d = 50 d = 10 1
OR
7 1
Solution: S7 = 49 [2a + 6d] = 49 a + 3d = 7 1
2 2
17 1
S17 = 289 [2a + 16d] = 289 a + 8d = 17 1
2 2
1 1
On solving, d = 2, a = 1 +
2 2
n
Sn = [2(1) + (n – 1)2] = n2 1
2
__________________________________________________________________________
Solution:
1
(For Fig)
430/4/3 13 P.T.O.
Let AC = x, CD = y
75
In ABC, tan 45 = 1
x
1
x = 75
2
75 1 75
In ABD, tan 30 = = 1
xy 3 xy
1
x + y = 75 3
2
y = 75 3–1 1
2
1
y = 54·75 m
2
Distance between two ships = 54·75 m
OR
Solution:
1
(For Fig)
10
In APC, tan 30 = 1
x
1
x = 10 3
2
10 h
In APB, tan 45 =
x
x = 10 + h 1
1
10 3 = 10 + h
2
430/4/3 14
h = 10 3–1
1
= 10 0·73 = 7·3 m
2
Length of flagstaff = 7·3 m
Distance of building from P = x = 10 3 m
1
= 17.3 m
2
1
only mark to be deducted for not using 3 = 1.73
2
__________________________________________________________________________
SECTION E
Solution:
(i) HCF (36, 60) = 12 1
Khushi can invite at the most 12 guests.
(ii) 36 12 = 3, 60 12 = 5
Each guest will get 3 apples and 5 bananas 1
430/4/3 15 P.T.O.
Solution:
1 1
(i) Figure A and Figure C +
2 2
(ii) Figure C 1
(iii) (a) Triangles are congruent Corresponding angles are equal 1
430/4/3 16
1
Triangles are similar.
2
Conversely, if triangles are similar then ratio of corresponding
sides is same which does not imply corresponding sides are equal
1
Triangles may not be congruent.
2
Note: Any suitable counter example can be given
OR
(iii) (b) One pair of corresponding side must be equal 2
__________________________________________________________________________
Solution:
We know that diagonals of a rectangle bisect each other.
1 7 1 5
(i) Required point = Mid-point of AC = , = (4, 3) 1
2 2
(ii) AC = (7 – 1)2 (5 – 1)2 = 2 13 1
1
(iii) (a) AB = (7 – 1)2 (1 – 1)2 = 6
2
1
BC = (7 – 7)2 (5 – 1)2 = 4
2
Area (ABCD) = AB BC = 6 4 = 24 1
430/4/3 17 P.T.O.
OR
(iii) (b) AB = (7 – 1) (1 – 1)2 = 6
2
1
AB 6 3
= = 1
AC 2 13 13
required ratio is 3 : 13
__________________________________________________________________________
430/4/3 18
Series WX1YZ/5 SET~1
J{UV (~w{Z`mXr)
MATHEMATICS (BASIC)
*
:3 : 80
Time allowed : 3 hours Maximum Marks : 80
NOTE :
(i) - 23
Please check that this question paper contains 23 printed pages.
(ii) - - -
-
Q.P. Code given on the right hand side of the question paper should be written on the title
page of the answer-book by the candidate.
(iii) - 38
Please check that this question paper contains 38 questions.
(iv) -
Please write down the serial number of the question in the answer-book before
attempting it.
(v) - 15 -
10.15 10.15 10.30 -
-
15 minute time has been allotted to read this question paper. The question paper will be
distributed at 10.15 a.m. From 10.15 a.m. to 10.30 a.m., the students will read the
question paper only and will not write any answer on the answer-book during this period.
1 1
(a) (b)
52 13
1 1
(c) (d)
26 6
3 4
6. LCM of (2 3 5) and (2 5 7) is :
(a) 40 (b) 560
(c) 1680 (d) 1120
a
11. If sin = , then sec is equal to (0 90 ) :
b
a b
(a) (b)
b2 a2 b2 a2
b2 a2 b2 a 2
(c) (d)
b a
(a) 12 cm (b) 6 cm
Frequency 10 12 14 13 17
3 (sec2 1) cos2
16. If sin = , then equals :
4 sin
3 3
(a) (b)
5 4
4 9
(c) (d)
3 16
AB PQ
17. In two triangles , it is given that . For these
BC PR
two triangles to be similar, which of the following should be true ?
(a) A= P (b) B= Q
(c) B= P (d) CA = QR
18. Mean and median of some data are 32 and 30 respectively. Using
empirical relation, mode of the data is :
(a) 36 (b) 26
(c) 30 (d) 20
19. Assertion (A) : When two coins are tossed together, the probability of
1
getting no tail is .
4
20. Assertion (A) : The surface area of largest sphere that can be inscribed in
a2 cm2.
4
Reason (R) : The surface area of a sphere of radius r is r3.
3
SECTION B
This section comprises very short answer (VSA) type questions of 2 marks each.
sin 30 tan 45
sec 30 cot 45
OR
(b) For A = 30 and B = 60 , verify that :
sin (A + B) = sin A cos B + cos A sin B.
24. (a) Show that A(1, 2), B(5, 4), C(3, 8) and D( 1, 6) are vertices of a
parallelogram ABCD.
OR
(b) Show that the points A(3, 0), B(6, 4) and C( 1, 3) are vertices of a
right-angled triangle.
1 1 1
25. Find the sum of the first 15 terms of the A.P. : , , , ...... .
15 12 10
SECTION C
This section comprises short answer (SA) type questions of 3 marks each.
26. (a) Sabina went to a bank ATM to withdraw < 2,000. She received
< 50 and < 100 notes only. If Sabina got 25 notes in all, how many
notes of < 50 and < 100 did she receive ?
OR
(b) Five years ago, Amit was thrice as old as Baljeet. Ten years hence,
Amit shall be twice as old as Baljeet. What are their present ages ?
Class 0 15 15 30 30 45 45 60 60 75 75 90
Frequency 12 15 11 20 16 6
OR
(b) In the given figure, -points of sides
BC and AC respectively. If given vertices are A(4, 2), B(2, 2) and
1
C( 6, 7), then verify the result DE = AB.
2
OR
(b) AB and CD are arcs of two concentric circles of radii 3·5 cm and
10·5 cm respectively and centred at O. Find the area of the shaded
region if AOB = 60 . Also, find the length of arc CD.
35. (a) The difference of two numbers is 5 and the difference of their
1
reciprocals is . Find the numbers.
10
OR
(b) Find all the values of k for which the quadratic equation
2x2 + kx + 8 = 0 has equal roots. Also, find the roots.
SECTION E
This section comprises 3 case study based questions of 4 marks each.
Case Study 1
36. Rainbow is an arch of colours that is visible in the sky after rain or when
water droplets are present in the atmosphere. The colours of the rainbow
are generally, red, orange, yellow, green, blue, indigo and violet. Each
colour of the rainbow makes a parabola. We know that any quadratic
polynomial p(x) = ax2 + bx + c (a 0) represents a parabola on the graph
paper.
OR
Case Study 2
Case Study 3
38. Some students were asked to list their favourite colour. The measure of
each colour is shown by the central angle of a pie chart given below :
SECTION A
Ans. (b) 2 1
-------------------------------------------------------------------------------------------------------------------
1
Ans. (a) 1
52
------------------------------------------------------------------------------------------------------------------
6
Ans. (a) – 1
5
-------------------------------------------------------------------------------------------------------------------
15
Ans. (d) 1
4
----------------------------------------------------------------------------------------------------------------
430/5/1 4 P.T.O.
Ans. (c) real and equal 1
-------------------------------------------------------------------------------------------------------------------
b
Ans. (b) 1
2 2
b –a
-------------------------------------------------------------------------------------------------------------------
------------------------------------------------------------------------------------------------------------------
430/5/1 5 P.T.O.
Ans. (b) 6 cm 1
------------------------------------------------------------------------------------------------------------------
Ans. (d) 60 – 80 1
-------------------------------------------------------------------------------------------------------------------
Ans. (b) 1
-------------------------------------------------------------------------------------------------------------------
Ans. (c) B = P 1
-------------------------------------------------------------------------------------------------------------------
Ans. (b) 26 1
------------------------------------------------------------------------------------------------------------------
430/5/1 6 P.T.O.
Ans. (b) Both Assertion (A) and Reason (R) are true, but Reason (R) is not
the correct explanation of Assertion (A). 1
-------------------------------------------------------------------------------------------------------------------
430/5/1 7 P.T.O.
1
1
2 1
Solution. Required value = 1
2 2
1
3
3 3 1
=
2(2 3 ) 2
OR
1
Solution. (b) LHS = sin 90 = 1
2
1 1 3 3
RHS = × + × 1
2 2 2 2
1
= 1
2
⇒ LHS = RHS
------------------------------------------------------------------------------------------------------
5 10 3 1 1
Solution. sin 60 = OA = cm +
OA 3 2 2
5 5 3 1 1
tan 60 = AB = cm= AC +
AB 3 2 2
-------------------------------------------------------------------------------------------------------------------
24. (a) Show that A(1,2),B(5,4),C(3,8) and D(-1,6) are vertices of a
parallelogram ABCD.
3 1 8 2
Solution. Mid point of AC = , = (2, 5) 1
2 2
430/5/1 8 P.T.O.
5 – 1 4 6 1
Mid point of BD= , =(2, 5)
2 2 2
1
Mid point of AC = Mid point of BD
2
Hence, ABCD is a parallelogram.
OR
(b) Show that the points A(3,0),B(6,4) and C(-1,3) are the vertices of a right
angled triangle.
1
Solution. AB2 = 32 + 42 = 25
2
1
BC2 = 72 + 12 = 50
2
1
AC2 = 42 + 32 = 25
2
1
⇒ BC2 = AB2 + AC2
2
ABC is a right-angled triangle.
----------------------------------------------------------------------------------------------------------------
1 1 1 1
Solution. Here d = – =
12 15 60 2
15 2 1
S15 = 14 1
2 15 60
15 22 11 1
=
2 60 4 2
------------------------------------------------------------------------------------------------------------
- SECTION C
OR
430/5/1 9 P.T.O.
(b) Five years ago Amit was thrice as old as Baljeet. Ten years hence, Amit
shall be twice as old as Baljeet . What are their present ages?
Solution: Let Amit’s present age be x years and Baljeet’s present age be y
years.
ATQ (x – 5) = 3(y – 5) x – 3y = – 10 1
and (x + 10) = 2(y + 10) x – 2y = 10 1
Solving equations to get y = 20, x = 50 1
Amit’s present age = 50 years and Baljeet’s present age = 20 years
--------------------------------------------------------------------------------------------------------------
Solution.
x – 37·5
Class x F u= fu
15
0 – 15 7·5 12 –2 – 24
15 – 30 22·5 15 –1 – 15
30 – 45 37·5 11 0 0
45 – 60 52·5 20 1 20
60 – 75 67·5 16 2 32
75 – 90 82·5 6 3 18
80 31
For Correct Table: 2 Marks
Mean = a
fu h
f
31
= 37·5 + 15 × = 43·3 1
80
--------------------------------------------------------------------------------------------------------------------
430/5/1 10 P.T.O.
Solution.
Let AP : PB = k : 1
– 4k 3 5 1
=–2 k= 1+
k 1 2 2
1
So, AP : PB = 5 : 2
2
10 – 8 2
Hence, =a ⇒ a= 1
7 7
OR
–9
Solution: Point D is – 2, 1
2
– 9
Point E is – 1, 1
2
1 1
DE = 12 0 2 = 1 and AB = 22 02 = 2 +
2 2
1
DE = AB
2
-------------------------------------------------------------------------------------------------------------------
430/5/1 11 P.T.O.
1 1 1
Solution. LHS = – sin A – cos A
sin A cos A 2
1 – sin 2 A 1 – cos2 A
= ( ) × ( )
sin A cos A
cos2 A sin 2 A sinA cos A
= = 1
sin A cos A 1
sin A cos A 1
= 2 2
sin A cos A 2
1
= = = RHS 1
sin A cos A
cos A sin A
-------------------------------------------------------------------------------------------------------------------
-------------------------------------------------------------------------------------------------------------------
SECTION D
Solution.
For Figure:
1
430/5/1 12 P.T.O.
h
In ∆BAD, tan 45 = ⇒ h = x + 48 _________ (i) 1
x 48
h 1
In ∆BAC, tan 60 = ⇒ h= 3 x _________ (ii) 1+
x 2
1 1
Solving (i) and (ii), x = 24( 3 + 1) = 65·57m +
2 2
1
and h = x + 48 = 113·57 m
2
(Note: only ½ mark to be deducted for not using 3 =1.732)
-------------------------------------------------------------------------------------------------------------------
22 90
Solution: Area of sector AOB = ×7 × 7 × 1
7 360
77 1
= cm2
2 2
1 49
Area of AOB = ×7 ×7 = cm2 1
2 2
77 49 28
Shaded area = – = = 14 cm2 1
2 2 2
22 90 1
Length of arc AB = 2 × ×7 × = 11 cm 1
7 360 2
430/5/1 13 P.T.O.
Solution. Here, OA = 3·5 cm, OC = 10·5 cm
60
Shaded area = × (10·52 – 3·52) 2
360
22 1
= × × 98 1
7 6
154 2 1
= cm or 51·3 cm2
3 2
22 60
Length of arc CD = 2 × ×10·5 × 1
7 360
1
= 11 cm
2
-------------------------------------------------------------------------------------------------------------------
Solution.
For
Figure: 1
1
Given : In ABC, DE ⃦ BC
2
AD AE 1
To prove : =
DB EC 2
1
Const.: Join BE, CD. Draw DM AC and EN AB
2
430/5/1 14 P.T.O.
1
ar ( ADE) AD EN AD
Proof : = 2 = _________ (i) 1
ar ( BDE) 1
DB EN DB
2
ar ( ADE) AE 1
Similarly = _________ (ii)
ar ( CDE) EC 2
BDE and CDE are on the same base DE and between the
same parallel lines BC and DE.
1
∴ ar( BDE) = ar( CDE) _________ (iii)
2
AD AE 1
From (i), (ii) and (iii), we get =
DB EC 2
-------------------------------------------------------------------------------------------------------------------
1
Solution. Let the numbers be x and x + 5.
2
1 1 1 1 1
– 1
x x 5 10 x 5 x
50 = x2 + 5x or x2 + 5x – 50 = 0 1
(x + 10) (x – 5) = 0 1
1
x = – 10, 5
2
The numbers are – 10, – 5 or 5, 10 1
430/5/1 15 P.T.O.
SECTION E
430/5/1 16 P.T.O.
(iii)(b) -4 is a zero of the given polynomial 21-7p = 0 1
p=3 1
------------------------------------------------------------------------------------------------------------------
430/5/1 17 P.T.O.
120 1
Solution. (i) P (favourite colour being white) = or 1
360 3
60 60 1
(ii) P (favourite colour being blue or green) = or 1
360 3
(iii) (a) Let total number of students be x
1
= 1
2
1
⇒ x = 60 or total 60 students participated in survey.
2
OR
60 30 1
(iii)(b) P (favourite colour being red or blue) = = 1+1
360 4
430/5/1 18 P.T.O.
Series WX1YZ/5 SET~2
J{UV (~w{Z`mXr)
MATHEMATICS (BASIC)
*
:3 : 80
Time allowed : 3 hours Maximum Marks : 80
NOTE :
(i) - 23
Please check that this question paper contains 23 printed pages.
(ii) - - -
-
Q.P. Code given on the right hand side of the question paper should be written on the title
page of the answer-book by the candidate.
(iii) - 38
Please check that this question paper contains 38 questions.
(iv) -
Please write down the serial number of the question in the answer-book before
attempting it.
(v) - 15 -
10.15 10.15 10.30 -
-
15 minute time has been allotted to read this question paper. The question paper will be
distributed at 10.15 a.m. From 10.15 a.m. to 10.30 a.m., the students will read the
question paper only and will not write any answer on the answer-book during this period.
SECTION A
This section comprises multiple choice questions (MCQs) of 1 mark each.
AB PQ
2. In two triangles , it is given that . For these
BC PR
two triangles to be similar, which of the following should be true ?
(a) A= P (b) B= Q
(c) B= P (d) CA = QR
7 7
(c) (d)
3 40
4. If the lines represented by equations 3x + 2my = 2 and 2x + 5y + 1 = 0 are
parallel, then the value of m is :
2 5
(a) (b)
5 4
3 15
(c) (d)
2 4
5. ABC
If AB = 10 cm, then DE equals :
(a) 8 cm (b) 7·5 cm
(c) 15 cm (d) 5 3 cm
Frequency 10 12 14 13 17
(a) 12 cm (b) 6 cm
(a) DE BC
1
(b) DE = BC
2
(c) DE = BC
(d) DE is not parallel to BC (DE BC)
10. The 8th term of an A.P. is 17 and its 14th term is 29. The common
difference of this A.P. is :
(a) 3 (b) 2
(c) 5 (d) 2
11. In the given figure, O is the centre of a circle, PQ is a chord and the
tangent PT at P makes an angle of 50 with PQ. The measure of POQ is :
b2 a2 b2 a 2
(c) (d)
b a
17. The sum of the first 100 even natural numbers is :
(a) 10100 (b) 2550
(c) 5050 (d) 10010
18. The value(s) of k for which the roots of quadratic equation x 2 + 4x + k = 0
are real, is :
(a) k 4 (b) k 4
(c) k 4 (d) k 4
Questions number 19 and 20 are Assertion and Reason based questions carrying
1 mark each. Two statements are given, one labelled as Assertion (A) and the
other is labelled as Reason (R). Select the correct answer to these questions from
the codes (a), (b), (c) and (d) as given below.
(a) Both Assertion (A) and Reason (R) are true and Reason (R) is the
correct explanation of the Assertion (A).
(b) Both Assertion (A) and Reason (R) are true, but Reason (R) is not
the correct explanation of the Assertion (A).
(c) Assertion (A) is true, but Reason (R) is false.
(d) Assertion (A) is false, but Reason (R) is true.
430/5/2 JJJJ Page 9 P.T.O.
19. Assertion (A) : When two coins are tossed together, the probability of
1
getting no tail is .
4
20. Assertion (A) : The surface area of largest sphere that can be inscribed in
a2 cm2.
4
Reason (R) : The surface area of a sphere of radius r is r3.
3
SECTION B
This section comprises very short answer (VSA) type questions of 2 marks each.
2 2 4
21. Find the sum of the first 20 terms of the A.P. : , 0, , , ...... .
3 3 3
22. (a) Show that A(1, 2), B(5, 4), C(3, 8) and D( 1, 6) are vertices of a
parallelogram ABCD.
OR
(b) Show that the points A(3, 0), B(6, 4) and C( 1, 3) are vertices of a
right-angled triangle.
sin 30 tan 45
sec 30 cot 45
OR
(b) For A = 30 and B = 60 , verify that :
sin (A + B) = sin A cos B + cos A sin B.
SECTION C
This section comprises short answer (SA) type questions of 3 marks each.
26. Prove that :
sin cos sin cos 2 sec2
+ =
sin cos sin cos tan2 1
27. ABC is an isosceles triangle with AB = AC, circumscribed about a circle.
Prove that BC is bisected at E.
Class 0 15 15 30 30 45 45 60 60 75 75 90
Frequency 12 15 11 20 16 6
31. (a) Determine the ratio in which the point P(a, 2) divides the
line segment joining the points A( 4, 3) and B(2, 4). Also, find the
OR
(b) In the given figure, -points of sides
BC and AC respectively. If given vertices are A(4, 2), B(2, 2) and
1
C( 6, 7), then verify the result DE = AB.
2
SECTION D
This section comprises long answer (LA) type questions of 5 marks each.
32. (a) A 2-digit number is four times the sum of its digits and twice the
product of its digits. Find the number.
OR
33. If a line is drawn parallel to one side of a triangle to intersect the other
two sides at distinct points, then prove that the other two sides are
divided in the same ratio.
34. A man in a boat rowing away from a lighthouse 100 m high takes
2 minutes to change the angle of elevation of the top of lighthouse from
60 to 45 . Find the speed of the boat. (Use 3 = 1·73)
OR
(b) AB and CD are arcs of two concentric circles of radii 3·5 cm and
10·5 cm respectively and centred at O. Find the area of the shaded
region if AOB = 60 . Also, find the length of arc CD.
Case Study 1
(i) What is the volume of the material used in making the mallet ? 1
(ii) The bowl is to be polished from inside. Find the inner surface area
of the bowl. 1
(iii) (a) Find the volume of metal used to make the bowl. 2
OR
(iii) (b) Find total surface area of the mallet. (Use = 3·14) 2
37. Some students were asked to list their favourite colour. The measure of
each colour is shown by the central angle of a pie chart given below :
(iii) (a) If 15 students liked the colour yellow, how many students
participated in the survey ? 2
OR
(iii) (b) What is the probability of the favourite colour being red or
blue ? 2
38. Rainbow is an arch of colours that is visible in the sky after rain or when
water droplets are present in the atmosphere. The colours of the rainbow
are generally, red, orange, yellow, green, blue, indigo and violet. Each
colour of the rainbow makes a parabola. We know that any quadratic
polynomial p(x) = ax2 + bx + c (a 0) represents a parabola on the graph
paper.
(i) The graph of a rainbow y = f(x) is shown in the figure. Write the
number of zeroes of the curve. 1
(ii) If the graph of a rainbow does not intersect the x-axis but
intersects y-axis at one point, then how many zeroes will it have ? 1
OR
(iii) (b) The polynomial x2 2x (7p + 3) represents a rainbow. If
4 is a zero of it, find the value of p. 2
Solution (b) 26 1
----------------------------------------------------------------------------------------------------------
Solution (c) B = P 1
Solution (c) B = P 1
-------------------------------------------------------------------------------------
𝟕
Solution (c) 1
𝟑
----------------------------------------------------------------------------------------------------
15
4
𝟏𝟓
Solution (d) 1
𝟒
----------------------------------------------------------------------------------------------------
430/5/2 3 P.T.O.
Solution (c) real and equal 1
------------------------------------------------------------------------------------------------------------
S80 1
Solution (b) 6 cm 1
------------------------------------------------------------------------------------------------------------------------------
Solution (b) 2 1
430/5/2 4 P.T.O.
Solution (b) 100 1
--------------------------------------------------------------------------------------------------------
𝟏
Solution (b) 1
𝟗
----------------------------------------------------------------------------------------------------
6
Solution (a) – 1
5
-------------------------------------------------------------------------------------------------------
430/5/2 5 P.T.O.
b
Solution (b) 1
2 2
b –a
--------------------------------------------------------------------------------------------------------
Solution (b) k 4 1
----------------------------------------------------------------------------------------------------------
Questions number 19 and 20 are Assertion and Reason based questions carrying
1 mark each. Two statements are given, one labelled as Assertion (A) and the
other is labelled as Reason (R). Select the correct answer to these questions from
the codes (a), (b), (c) and (d) as given below.
(a) Both Assertion (A) and Reason (R) are true and Reason (R) is
the correct explanation of the Assertion (A).
(b) Both Assertion (A) and Reason (R) are true, but Reason (R) is
not the correct explanation of the Assertion (A).
(c) Assertion (A) is true, but Reason (R) is false.
(d) Assertion (A) is false, but Reason (R) is true.
Solution: (b) Both Assertion (A) and Reason (R) are true, but Reason(R)is not the
correct explanation of Assertion(A). 1
---------------------------------------------------------------------------------------------------------------
430/5/2 6 P.T.O.
SECTION B
1
𝟐
Solution. d=– 2
𝟑
𝟐𝟎 𝟒 𝟐
S20 = + 19 – 1
𝟐 𝟑 𝟑
1
340
=– 2
𝟑
-------------------------------------------------------------------------------------------------------
3 1 8 2
Solution : Mid point of AC = , = (2, 5) 1
2 2
5 – 1 4 6 1
Mid point of BD = , = (2, 5)
2 2 2
1
So, mid point of AC = mid point of BD
2
OR
(b) Show that the points A(3,0), B(6,4) and C(-1,3) are the vertices of a right
angled triangle.
Solution : AB2 = 32 + 42 = 25
BC2 = 72 + 12 = 50
1
AC2 = 42 + 32 = 25
2
1
⟹ BC2 = AB2 + AC2
2
430/5/2 7 P.T.O.
1
1
2 1
Solution : Required Value = 1
2 2
1
3
3 3 1
=
2 (2 3 ) 2
OR
1
Solution: LHS = sin 90 = 1
2
1 1 3 3
RHS = × + × 1
2 2 2 2
1
=1
2
LHS = RHS
---------------------------------------------------------------------------------------------------------------
1
Solution. 660 = 22 × 3 ×5 ×11
2
1
704 = 26 × 11
2
HCF (660, 704) = 22 × 11 = 44 1
---------------------------------------------------------------------------------------------------------------
𝟓 𝟏𝟎√𝟑 1 1
Solution: sin 60 = OA = cm +
OA 𝟑 2 2
√ 1 1
tan 60 = AB = cm = AC +
2 2
430/5/2 8 P.T.O.
SECTION C
Solution. Let the number of ₹50 notes be x and the number of ₹100 notes be y.
ATQ x + y = 25 _______ (i) 1
∴ The number of ₹50 notes =10 and number of ₹100 notes =15.
OR
430/5/2 9 P.T.O.
(b) Five years ago Amit was thrice as old as Baljeet. Ten years hence, Amit shall
be twice as old as Baljeet . What are their present ages?
Solution: Let Amit’s present age be x years and Baljeet’s present age be y years.
ATQ (x – 5) = 3(y – 5) x – 3y = – 10 1
and (x + 10) = 2(y + 10) x – 2y = 10 1
Solving equations to get y = 20, x = 50 1
Amit’s present age = 50 years and Baljeet’s present age = 20 years.
--------------------------------------------------------------------------------------------------------
x – 37·5
Class x f u= fu
Solution 15
0 – 15 7·5 12 –2 – 24
15 – 30 22·5 15 –1 – 15
30 – 45 37·5 11 0 0
45 – 60 52·5 20 1 20
60 – 75 67·5 16 2 32
75 – 90 82·5 6 3 18
For CorrectTable:
80 31
2 marks
∑ 𝑓𝑢 31
Mean = a + ∑𝑓
× ℎ = 37·5 + 15 × = 43·3 1
80
---------------------------------------------------------------------------------------------------------------
Solution: k P(a,-2) 1
A (– 4,3) • B (2, – 4)
430/5/2 10 P.T.O.
Let AP : PB = k : 1
– 4k 3
=–2 k= 1+
k 1
1
So, AP : PB = 5 : 2
2
10 – 8 2
Hence, =a ⇒ a= 1
7 7
OR
–9
Solution: Point D is – 2, 1
2
–9
Point E is – 1, 1
2
1 1
DE = √1 + 0 = 1 and AB = √2 + 0 = 2 +
2 2
DE = AB
----------------------------------------------------------------------------------------------------------
SECTION D
430/5/2 11 P.T.O.
x = 20 (x-12) and y = 20 – 8 = 12 1
The dimensions of the rectangle are 20 cm , 12 cm
------------------------------------------------------------------------------------------------------
Solution.
For figure:
1
1
Given: In ABC, DE ‖BC
2
AD AE 1
To prove : =
DB EC 2
1
Const. : Join BE, CD. Draw DM AC and EN AB
2
1
ar ( ADE) AD EN AD
Proof : = 2 = _________ (i) 1
ar ( BDE) 1 DB
DB EN
2
ar ( ADE) AE 1
Similarly = _________ (ii)
ar ( CDE) EC 2
BDE and CDE are on the same base DE and between
the same parallel lines BC and DE.
1
ar( BDE) = ar( CDE) _________ (iii)
2
AD AE 1
From (i), (ii) and (iii) , we get =
DB EC 2
---------------------------------------------------------------------------------------------------------------
Solution:
For figure: 1
100 1
In ∆ BAC, tan 60 = x 3 = 100 ________ (i) 1+
x 2
430/5/2 12 P.T.O.
100
In ∆BAD, tan 45 = x + y = 100 ________ (ii) 1
xy
Solving equations (i) and (ii) to get ,
x=
100
3
3 = 57·67 m and y = 42·33 m 1
1
Hence, Speed of boat = 21·17 m/minute (approx.)
2
(Note: mark to be deducted for not using √3 =1.73)
------------------------------------------------------------------------------------------------------------
22 90
Solution : Area of sector AOB = ×7 × 7 × 1
7 360
77 1
= cm2
2 2
1 49
Area of AOB = ×7 ×7 = cm2 1
2 2
Shaded area = – = = 14 cm2 1
22 90 1
Length of arc AB = 2 × ×7 × = 11 cm 1
7 360 2
OR
430/5/2 13 P.T.O.
Solution : Here OA = 3·5 cm, OC = 10·5 cm
Shaded area = × (10·52 – 3·52) 2
22 1
= × × 98 1
7 6
1
= cm2 or 51·3 cm2
2
22 60
Length of arc CD = 2 × ×10·5 × 1
7 360
1
= 11 cm
2
----------------------------------------------------------------------------------------------------------
SECTION E
430/5/2 14 P.T.O.
Solution. (i) Volume of material = 3·14 × 2 ×2 × 10 = 125·6 cm3 1
(ii) Inner SA of the bowl = 2 × 3·14 × 25 = 157 cm 2 1
2 3 3
(iii) (a) Volume of the metal = ×3·14 × (6 – 5 ) 1
3
= 190·5 cm3 1
OR
= 150·7 cm2 1
------------------------------------------------------------------------------------------------
430/5/2 15 P.T.O.
120 1
Solution. (i) P (favourite colour being white) = or 1
360 3
60 60 1
(ii) P (favourite colour being blue or green) = or 1
360 3
(iii) (a) Let total number of students be x
1
= 1
2
1
⇒ x = 60
2
OR
60 30 1
(iii) (b) P (favourite colour being red or blue) = = 1+1
360 4
--------------------------------------------------------------------------------------------------
430/5/2 16 P.T.O.
Solution. (i) Two zeroes 1
(ii) 0 or no zero 1
1 1
(iii) (a) Getting 2a+b = -6 and -3a+b = -6 +
2 2
1 1
Solving to get a = 0 and b = -6 +
2 2
OR
430/5/2 17 P.T.O.
Series WX1YZ/5 SET~3
J{UV (~w{Z`mXr)
MATHEMATICS (BASIC)
*
:3 : 80
Time allowed : 3 hours Maximum Marks : 80
NOTE :
(i) - 23
Please check that this question paper contains 23 printed pages.
(ii) - - -
-
Q.P. Code given on the right hand side of the question paper should be written on the title
page of the answer-book by the candidate.
(iii) - 38
Please check that this question paper contains 38 questions.
(iv) -
Please write down the serial number of the question in the answer-book before
attempting it.
(v) - 15 -
10.15 10.15 10.30 -
-
15 minute time has been allotted to read this question paper. The question paper will be
distributed at 10.15 a.m. From 10.15 a.m. to 10.30 a.m., the students will read the
question paper only and will not write any answer on the answer-book during this period.
Frequency 10 12 14 13 17
3 (sec2 1) cos2
7. If sin = , then equals :
4 sin
3 3
(a) (b)
5 4
4 9
(c) (d)
3 16
th th
8. The 8 term of an A.P. is 17 and its 14 term is 29. The common
difference of this A.P. is :
(a) 3 (b) 2
(c) 5 (d) 2
430/5/3 JJJJ Page 5 P.T.O.
9. In the given figure, O is the centre of the circle and PA is a tangent to the
circle. If OAB = 60 , then OPA is equal to :
(a) 60 (b) 30
(c) 15 (d) 20
10. One card is drawn at random from a well shuffled deck of 52 playing
cards. The probability that it is a red king is :
1 1
(a) (b)
52 26
2 2
(c) (d)
26 13
12. ABC
If AB = 10 cm, then DE equals :
(a) 8 cm (b) 7·5 cm
(c) 15 cm (d) 5 3 cm
b2 a2 b2 a 2
(c) (d)
b a
15. The distance between the points A(0, 6) and B( 6, 2) is :
(a) 6 units (b) 2 6 units
(c) 2 13 units (d) 13 2 units
16. The value(s) of k for which the roots of quadratic equation x2 + 4x + k = 0
are real, is :
(a) k 4 (b) k 4
(c) k 4 (d) k 4
4 2 3 2
17. HCF of (3 2 7 ) and (3 5 7) is :
(a) 630 (b) 63
(c) 729 (d) 567
18. If one zero of the quadratic polynomial kx2 + 3x + k is 2, then the value
of k is :
6 6
(a) (b)
5 5
5 5
(c) (d)
6 6
Questions number 19 and 20 are Assertion and Reason based questions carrying
1 mark each. Two statements are given, one labelled as Assertion (A) and the
other is labelled as Reason (R). Select the correct answer to these questions from
the codes (a), (b), (c) and (d) as given below.
(a) Both Assertion (A) and Reason (R) are true and Reason (R) is the
correct explanation of the Assertion (A).
(b) Both Assertion (A) and Reason (R) are true, but Reason (R) is not
the correct explanation of the Assertion (A).
(c) Assertion (A) is true, but Reason (R) is false.
(d) Assertion (A) is false, but Reason (R) is true.
430/5/3 JJJJ Page 9 P.T.O.
19. Assertion (A) : The surface area of largest sphere that can be inscribed in
a2 cm2.
4
Reason (R) : The surface area of a sphere of radius r is r3.
3
20. Assertion (A) : When two coins are tossed together, the probability of
1
getting no tail is .
4
Reason (R) : The probability P(E) of an event E satisfies 0 P(E) 1.
SECTION B
This section comprises very short answer (VSA) type questions of 2 marks each.
21. In the given figure, tangents AB and AC are drawn to a circle centred at
O. If OAB = 60 and OB = 5 cm, find lengths OA and AC.
29 25 23
22. Find the sum of the first 20 terms of the A.P. : , 9, , , ...... .
3 3 3
sin 30 tan 45
sec 30 cot 45
OR
(b) For A = 30 and B = 60 , verify that :
sin (A + B) = sin A cos B + cos A sin B.
430/5/3 JJJJ Page 11 P.T.O.
24. Find LCM of 480 and 256 using prime factorization.
25. (a) Show that A(1, 2), B(5, 4), C(3, 8) and D( 1, 6) are vertices of a
parallelogram ABCD.
OR
(b) Show that the points A(3, 0), B(6, 4) and C( 1, 3) are vertices of a
right-angled triangle.
SECTION C
This section comprises short answer (SA) type questions of 3 marks each.
Class 0 15 15 30 30 45 45 60 60 75 75 90
Frequency 12 15 11 20 16 6
27. (a) Determine the ratio in which the point P(a, 2) divides the
line segment joining the points A( 4, 3) and B(2, 4). Also, find
OR
30. (a) Sabina went to a bank ATM to withdraw < 2,000. She received
< 50 and < 100 notes only. If Sabina got 25 notes in all, how many
notes of < 50 and < 100 did she receive ?
OR
(b) Five years ago, Amit was thrice as old as Baljeet. Ten years hence,
Amit shall be twice as old as Baljeet. What are their present ages ?
32. (a) Divide 16 into two parts such that twice the square of the greater
part, exceeds the square of the smaller part by 164.
OR
(b) A motor boat whose speed is 18 km/h in still water takes 1 hour
more to go 24 km upstream, than to return to the same point. Find
the speed of the stream and total time of the journey.
33. If a line is drawn parallel to one side of a triangle to intersect the other
two sides at distinct points, then prove that the other two sides are
divided in the same ratio.
34. From the top of a building 50 m high, the angles of depression of the top
and bottom of a tower are observed to be 30 and 60 . Find the height of
the tower and distance between the building and the tower.
(Take 3 = 1·73)
OR
SECTION E
This section comprises 3 case study based questions of 4 marks each.
Case Study 1
36. Singing bowls (hemispherical in shape) are commonly used in sound
healing practices. Mallet (cylindrical in shape) is used to strike the bowl
in a sequence to produce sound and vibration.
37. Rainbow is an arch of colours that is visible in the sky after rain or when
water droplets are present in the atmosphere. The colours of the rainbow
are generally, red, orange, yellow, green, blue, indigo and violet. Each
colour of the rainbow makes a parabola. We know that any quadratic
polynomial p(x) = ax2 + bx + c (a 0) represents a parabola on the graph
paper.
38. Some students were asked to list their favourite colour. The measure of
each colour is shown by the central angle of a pie chart given below :
(iii) (a) If 15 students liked the colour yellow, how many students
participated in the survey ? 2
OR
(iii) (b) What is the probability of the favourite colour being red or
blue ? 2
---------------------------------------------------------------------------------------------------------------------
3. A circle is of radius 3 cm. The distance between two of its parallel tangents is:
(a) 12 cm (b) 6 cm
(c) 3 cm (d) 4·5 cm
Answer: (b) 6 cm 1
---------------------------------------------------------------------------------------------------------------------
4. The median class for the data given below is :
Answer: (c) B = P 1
--------------------------------------------------------------------------------------------------------------------
430/5/3 3 P.T.O.
7.
(a) (b)
(c) (d)
3
Answer : (b) 1
4
---------------------------------------------------------------------------------------------------------------------
8. The 8th term of an A.P. is 17 and its 14th term is 29. The common
difference of this A.P. is :
(a) 3 (b) 2
(c) 5 (d) -2
Answer : (b) 2 1
---------------------------------------------------------------------------------------------------------------------
9. In the given figure, O is the centre of the circle and PA is a tangent to the
circle. If OAB = 60 , then OPA is equal to :
(a) 60 (b) 30
(c) 15 (d) 20
Answer : (b) 30 0 1
---------------------------------------------------------------------------------------------------------------------
10. One card is drawn at random from a well shuffled deck of 52 playing cards. The
probability that it is a red king is :
(a) (b)
(c) (d)
1
Answer: (b) 1
26
---------------------------------------------------------------------------------------------------------------------
11. If the lines represented by equations 3x + 2my = 2 and 2x + 5y + 1 = 0 are parallel, then
the value of m is :
(a) (b)
(c) (d)
430/5/3 4 P.T.O.
15
Answer: (d) 1
4
---------------------------------------------------------------------------------------------------------------------
12. ABC ~∆ DEF and their perimeter are 32 cm and 24 cm respectively.
If AB = 10 cm, then DE equals :
(a) 8 cm (d) 7.5 cm
(c) 15 cm (d) 5√3 cm
Answer: (b) 7·5 cm 1
---------------------------------------------------------------------------------------------------------------------
13. The two roots of the equation 3x2 - 2 √6 x + 2 = 0 are:
(a) real and distinct
(b) not real
(c) real and equal
(d) rational
Answer: (c) real and equal 1
---------------------------------------------------------------------------------------------------------------------
14.
Answer: 1
---------------------------------------------------------------------------------------------------------------------
15. The distance between the points A (0, 6) and B (-6, 2) is :
(a) 6 units (b) 2 √6 units
(c) 2 √13units (d) 13 √2 units
Answer : (c) 2 13 units 1
---------------------------------------------------------------------------------------------------------------------
16. The value(s) of k for which the roots of quadratic equation x2 + 4x + k = 0 are real, is :
(a) k≥ 4 (b) k≤4
(c) k≥ ̶4 (d) k≤ ̶ 4
Answer: (b) k 4 1
---------------------------------------------------------------------------------------------------------------------
17. HCF of (34 x 22 x 73) and (32 x 5 x 7) is :
(a) 630 (b) 63
(c) 729 (d) 567
Answer: (b) 63 1
---------------------------------------------------------------------------------------------------------------------
18. If one zero of the quadratic polynomial kx2 + 3x + k is 2, then the value of k is:
(a) (b)
(b) (d)
430/5/3 5 P.T.O.
6
Answer: (a) – 1
5
---------------------------------------------------------------------------------------------------------------------
Questions number 19 and 20 are Assertion and Reason based questions carrying 1 mark
each. Two statements are given, one labelled as Assertion (A) and the other is labelled as
Reason (R). Select the correct answer to these questions from the codes (a), (b), (c) and (d)
as given below.
(a) Both Assertion (A) and Reason (R) are true and Reason (R) is the correct
explanation of the Assertion (A).
(b) Both Assertion (A) and Reason (R) are true, but Reason (R) is not the correct
explanation of the Assertion (A).
(c) Assertion (A) is true, but Reason (R) is false.
(d) Assertion (A) is false, but Reason (R) is true.
19. Assertion (A) : The surface area of largest sphere that can be inscribed in a hollow cube of
side ‘a’ cm is a2 cm2.
4
Reason (R) : The surface area of a sphere of radius ‘r’ is 3 𝜋 r 3 .
Answer: (c) Assertion (A) is true, but Reason (R) is false. 1
---------------------------------------------------------------------------------------------------------------------
20. Assertion (A) :When two coins are tossed together, the probability of getting no tail is .
---------------------------------------------------------------------------------------------------------------------
−29 −25 −23
22. Find the sum of the first 20 terms of the A.P. : 3 , -9 , 3 , 3 ,….
𝟐 1
Solution: d=𝟑
2
20 58 2
S20 = – 19 1
2 3 3
1
200 2
= –
3
430/5/3 6 P.T.O.
---------------------------------------------------------------------------------------------------------------------
sin 30⁰+tan 45⁰
23. (a) Evaluate :
sec 30⁰+cot 45⁰
1
1
2 1
Solution: (a) Required value is 1
2 2
1
3
3 3 1
=
2 2 3 2
-----------------------------------------------------------------------------------------------------------------
24. Find LCM of 480 and 256 using prime factorization.
1
Solution: 480 = 25 ×3 × 5
2
1
256 = 28
2
8
LCM (480, 256) = 2 ×3 × 5 = 3840 1
---------------------------------------------------------------------------------------------------------------------
25. (a) Show that A(1, 2), B(5, 4), C(3, 8) and D(-1, 6) are vertices of a
parallelogram ABCD.
3 1 8 2
Solution: Mid-point of AC= , = (2, 5) 1
2 2
5 – 1 4 6 1
Mid-point of BD= , = (2, 5)
2 2 2
1
Mid-point of AC = Mid-point of BD
2
Hence, ABCD is a parallelogram
OR
(b) Show that the points A(3, 0), B(6, 4) and C(-1, 3) are the vertices of a right
angled triangle.
AB2 = 32 + 42 = 25
1
Solution:
2
BC2 = 72 + 12 = 50
1
2
430/5/3 7 P.T.O.
AC2 = 42 + 32 = 25
1
2
BC2 = AB2 + AC2
1
2
ABC is right-angled triangle.
---------------------------------------------------------------------------------------------------------------------
SECTION C
26.
Solution.
x – 37·5
Class x f u= fu
15
0 – 15 7·5 12 –2 – 24
15 – 30 22·5 15 –1 – 15
30 – 45 37·5 11 0 0
45 – 60 52·5 20 1 20
60 – 75 67·5 16 2 32
75 – 90 82·5 6 3 18
80 31
For Correct Table: 2 Marks
Mean = a
fu h
f
31
= 37·5 + 15 × = 43·3 1
80
---------------------------------------------------------------------------------------------------------------------
27.
Solution:
430/5/3 8 P.T.O.
–9
Solution: Point D is – 2, 1
2
– 9
Point E is – 1, 1
2
1 1
DE = 12 0 2 = 1 and AB = 22 02 = 2 +
2 2
1
DE = AB
2
1
BE = EC or BC is bisected at E.
2
---------------------------------------------------------------------------------------------------------------------
𝑠𝑒𝑐𝜃−1 𝑠𝑒𝑐𝜃+1
29. Prove that : √𝑠𝑒𝑐𝜃+1 + √𝑠𝑒𝑐𝜃−1 =2cosec 𝜃
sec – 1 sec 1 2 sec
Solution: LHS = + = 1
sec 1 sec – 1 sec2 – 1
2 sec
= 1
tan
2 cos
= = 2 cosec = RHS 1
cos . sin
-------------------------------------------------------------------------------------------------------------
430/5/3 9 P.T.O.
30. (a) Sabina went to a bank ATM to withdraw ₹2,000. She received ₹ 50 and ₹
100 notes only. If Sabina got 25 notes in all, how many notes of ₹ 50 and ₹
100 did she receive?
Solution: (a) Let number of ₹50 notes be x and number of ₹100 notes be y.
ATQ, x + y = 25 _______ (i) 1
And, 50x + 100y = 2000 _______ (ii) 1
Solving (i) and (ii), x= 10, y = 15 1
Number of ₹50 notes = 10 and Number of ₹100 notes = 15
OR
(b) Five years ago, Amit was thrice as old as Baljeet. Ten years hence, Amit shall be twice as old
as Baljeet. What are their present ages?
Solution :( b) Let Amit’s present age be x years and Baljeet’s present age be y years.
ATQ, (x – 5) = 3(y – 5) x – 3y = – 10 1
And (x + 10) = 2(y + 10) x – 2y = 10 1
Solving equations to get, y = 20, x = 50 1
Amit’s present age = 50 years and Baljeet’s present age = 20 years
---------------------------------------------------------------------------------------------------------------------
31. Prove that 11 + 3 √2 is an irrational number, given that √2 is an irrational
number.
Solution: Let us assume that 11+3√2be a rational number.
a
11+3√2= , where a and b are integers, b≠0 1
b
a-11b
√2 = 1
3𝑏
RHS is a rational number but LHS is irrational. 1
Our assumption was wrong. Hence, 11+ 3√2 is irrational.
---------------------------------------------------------------------------------------------------------------------
SECTION D
32. (a) Divide 16 into two parts such that twice the square of the greater part,
exceeds the square of the smaller part by 164
Solution: (a) Let the greater part be x then the other part is 16 - x
ATQ , 2x2 = (16 -x) 2 + 164 1
x + 32x – 420 = 0
2 1
(x + 42) (x – 10) = 0 1
x = – 42, x = 10 1
x = 10 (x-42)
Two parts are 10 and 6. 1
OR
(b) A motor boat whose speed is 18 km/h in still water takes 1 hour more to go 24 km
upstream, than to return to the same point. Find the speed of the stream and total time of the
journey.
Solution: Let the speed of the stream be x km/hour.
24 24
ATQ, – =1 2
18 – x 18 x
430/5/3 10 P.T.O.
x2 + 48x – 324 = 0 1
(x + 54) (x – 6) = 0
x = 6 (x-54) 1
24 24
Total time of journey = = 3 hours 1
12 24
---------------------------------------------------------------------------------------------------------------------
33. If a line is drawn parallel to one side of a triangle to intersect the other two
sides at distinct points, then prove that the other two sides are divided in
the same ratio.
Solution.
For figure:
1 mark
1
Given: In ABC, DE // BC
2
AD AE 1
To prove: =
DB EC 2
1
Const.: Join BE, CD. Draw DM AC, EN AB
2
1
AD EN
ar ( ADE) AD
Proof: = 2 = _________ (i) 1
ar ( BDE) 1
DB EN DB
2
ar ( ADE) AE 1
Similarly, = _________ (ii)
ar ( CDE) EC 2
BDE and CDE are on the same base DE and between the same parallel
lines BC and DE.
1
ar ( BDE) = ar( CDE) _________ (iii)
2
𝐴𝐷 𝐴𝐸 1
From (i), (ii) and (iii) =
𝐷𝐵 𝐸𝐶 2
-------------------------------------------------------------------------------------------------------------
34. From the top of a building 50 m high, the angles of depression of the top and
bottom of a tower are observed to be 300 and 600. Find the height of the
tower and distance between the building and the tower. (Take √3 = 1·73)
Solution:
For Figure:
1 mark
430/5/3 11 P.T.O.
Let AB be the building and CD be the tower.
50 – h x 1
In ∆BED, tan 30 = 50 – h = _______ (i) 1+
x 3 2
50 50 1
In ∆ BAC, tan 60 = x= ________ (ii) 1+
x 3 2
100 1
Solving (i) and (ii) to get h= or 33·33 m
3 2
1
and x = 28·83 m
2
-------------------------------------------------------------------------------------------------------------
35. (a) In the given figure, AB is a chord of a circle of radius 7 cm and
centred at O. Find the area of the shaded region if AOB = 900. Also, find length of minor
arc AB.
22 90
Solution: Area of sector AOB = ×7 × 7 × 1
7 360
77 1
= cm2
2 2
1 49
Area of AOB = ×7 ×7 = cm2 1
2 2
77 49 28
Shaded area = 2 – 2 = 2 = 14 cm2 1
22 90 1
Length of arc AB = 2 × ×7 × = 11 cm 1+
7 360 2
OR
(b) AB and CD are arcs of two concentric circles of radii 3·5 cm and 10·5 cm
respectively and centered at O. Find the area of the shaded region if ⎿AOB = 600. Also, find
the length of arc CD.
430/5/3 13 P.T.O.
Solution. (i) Two zeroes 1
(ii) 0 or no zero 1
1 1
(iii) (a) Getting 2a+b = -6 and -3a+b = -6 +
2 2
1 1
Solving to get a = 0 and b = -6 +
2 2
OR
430/5/3 14 P.T.O.
120 1
Solution. (i) P (favourite colour being white) = or 1
360 3
60 60 1
(ii) P (favourite colour being blue or green) = or 1
360 3
(iii) (a) Let total number of students be x
15 1 1
= 1
𝑥 4 2
1
⇒ x = 60
2
OR
60 30 1
(iii)(b) P (favourite colour being red or blue) = = 1+1
360 4
430/5/3 15 P.T.O.
MARKING SCHEME
MATHEMATICS (BASIC) 430/6/1
SECTION A
Answer (c) 82 1
----------------------------------------------------------------------------------------------------------------
Answer (b) 5 1
-----------------------------------------------------------------------------------------------------------------
8
Answer (b) 7 1
---------------------------------------------------------------------------------------------------------------
3
430/6/1
77
Answer (c) 2 cm2 1
------------------------------------------------------------------------------------------------------------------
1
Answer (a) 26 1
------------------------------------------------------------------------------------------------------------------
Answer (a) 0 1
------------------------------------------------------------------------------------------------------------------
4
430/6/1
Answer (d) 4 1
------------------------------------------------------------------------------------------------------------------
Answer (a) 28 x 32 1
-----------------------------------------------------------------------------------------------------------------
Answer (a) 0 1
------------------------------------------------------------------------------------------------------------------
Answer (b) 7 1
------------------------------------------------------------------------------------------------------------------
Answer (d) – 2 1
----------------------------------------------------------------------------------------------------------------
5
430/6/1
3
Answer (a) k = 2 1
-------------------------------------------------------------------------------------------------------------
3 3
Answer (b) - 4, 4 1
------------------------------------------------------------------------------------------------------------------
Answer (d) – 1 1
-----------------------------------------------------------------------------------------------------------------
6
430/6/1
1
Answer (d) 3 1
-----------------------------------------------------------------------------------------------------------------
Answer (b) x + y = 19 1
------------------------------------------------------------------------------------------------------------------
Answer (b)Both Assertion(A) and Reason (R) are true but Reason (R) is not the correct
explanation of Assertion (A). 1
------------------------------------------------------------------------------------------------------------------
-
Answer (c) Assertion(A) is true, but Reason (R) is false. 1
------------------------------------------------------------------------------------------------------------------
7
430/6/1
SECTION B
= 5(√2) -3(1)2+5(1)
2
1
12
1
= 12 2
--------------------------------------------------------------------------------------------------------
1
Solution (a) Sum of zeroes = 6+ (–3) = 3 2
1
Product of zeroes = 6(– 3) = – 18 2
Solution (b) x2 + 4x – 12 = (x + 6) (x – 2) 1
Zeroes are – 6, 2 1
----------------------------------------------------------------------------------------------------------------
Solution a = 5, b = – 10, c = k ½
Roots are real and equal
D = 0 b2 – 4ac = 0 ½
2
(– 10) – 4(5) (k) = 0 100 – 20k = 0 ½
k=5 ½
OR
11
Solution P(2 digit number) = 20 1
9
P(number less than 10) = 20 1
------------------------------------------------------------------------------------------------------------------
Solution
. figure ½
OQ = √252 – 242 1
OQ = 7 cm ½
------------------------------------------------------------------------------------------------------------------
SECTION C
9
430/6/1
Solution
Family size 1–3 3–5 5–7 7–9 9 – 11
Number of families 7 8 2 2 1
Cf 7 15 17 19 20
For correct cf 1
Median class 3 – 5 ½
𝑁
–C
2
Median = l + xh
𝑓
10 – 7
=3+ x2 1
8
= 3·75 ½
------------------------------------------------------------------------------------------------------------------
Solution
(a)ABCD is a parallelogram (1 for figure)
To prove: ABE CFB
In ABE and CFB,
A = C (opp. angles of parallelogram) 1/2
AEB = CBF (alt. int. angles ) 1/2
⸫ ABE CFB (AA similarity) 1
10
430/6/1
OR
Solution
11
430/6/1
𝑐𝑜𝑠 𝐴
𝑐𝑜𝑡 A – cos A – cos A
sin A
Solution LHS = 𝑐𝑜𝑡 𝐴 + cos A = 𝑐𝑜𝑠 𝐴 ½
+ cos A
sin A
1−𝑠𝑖𝑛 𝐴
= 1+𝑠𝑖𝑛 𝐴 1
(1 – sin A)(1 + sin A)
= (1 + sin A)(1 + sin A) 1
1 – sin2 𝐴 𝑐𝑜𝑠2 𝐴
= (1 + sin A)2 = (1 + sin A)2 ½
OR
Solution
For fig. 1
3
In APQ, tan 30 = 𝐴𝑄 1
1 3
= 𝐴𝑄 AQ = 3√3 ½
√3
3
In PBQ, tan 45 = 𝐵𝑄 1
BQ = 3 ½
⸫AB = AQ + BQ = 3√3 + 3 ½
= 3(1·73 + 1) = 8·19
Width of river = 8·19 m ½
12
430/6/1
OR
For fig. 1
20
In ABD, tan 45 = 1
𝑥
x = 20 ½
20 + h
In ACD, tan 60 = 𝑥 1
√3x = 20 + h ½
⸫ h = 20 (√3 - 1) m ½
h = 14.6 m ½
------------------------------------------------------------------------------------------------------------------
Solution a = 22, an = – 6, Sn = 64
𝑛
Sn = 64 2 [22 - 6] = 64 2
n=8 1
22 + (8-1) d = -6 1
d=–4 1
------------------------------------------------------------------------------------------------------------------
13
430/6/1
Solution Height of conical part = 10 – 5 = 5 cm ½
Volume of 1 ice cream cone
1 2
= 3r2h + 3r3 1
1
= 3r2(h + 2r)
1 22
=3x x 5 x 5[5 + 10] 1½
7
22 × 25 ×15
= cm3 1
21
Volume of 7 ice cream cones
22 × 25 ×15 1
=7× 21 2
1
= 2750 cm3 2
----------------------------------------------------------------------------------------------------------- ------
Solution
For figure1
1
Given In ABC, DE // BC 2
𝐴𝐷 𝐴𝐸 1
To prove : 𝐷𝐵 = 𝐸𝐶 2
1
Const. : Join BE, CD. Draw DM ⊥ AC and EN ⊥ AB 2
1
𝑎𝑟(𝛥 ADE) × AD × EN 𝐴𝐷
2
Proof : 𝑎𝑟(𝛥 BDE) = 1 = 𝐷𝐵 _________ (i) 1
× DB × EN
2
𝑎𝑟(𝛥 ADE) 𝐴𝐸 1
similarly = 𝐸𝐶 _________ (ii)
𝑎𝑟(𝛥 CDE) 2
BDE and CDE are on the same base DE and between the same parallel lines BC and DE.
1
ar( ADE) = ar( CDE) _________ (iii) 2
𝐴𝐷 𝐴𝐸 1
From (i), (ii) and (iii) = 𝐸𝐶
𝐷𝐵 2
14
430/6/1
OR
Solution In PQR, 1 = 2
⸫ PQ = PR (sides opposite to equal angles) 1
𝑄𝑅 𝑄𝑇
Now 𝑄𝑆 = 𝑃𝑅
𝑄𝑆 𝑃𝑅 𝑄𝑆 𝑃𝑄
⸫ = 𝑄𝑇 𝑄𝑅 = 𝑄𝑇 (as PR = PQ) _________ (i) 2
𝑄𝑅
In PQS and TQR,
Q = Q (common)
𝑄𝑆 𝑃𝑄 1
= (from (i)) 12
𝑄𝑅 𝑄𝑇
1
⸫ PQS TQR (SAS similarity) 2
------------------------------------------------------------------------------------------------------------------
SECTION E
15
430/6/1
Solution (i) Area of square ABCD = (8)2 = 64 cm2 1
(ii) AC = √(8)2 + (8)2 =√128 = 8√2 cm 1
(iii) We know that diagonals of square bisect each other at 90
AOB = 90
𝜋𝑟 2 𝜃
Area of sector OPRQ = 360°
22 90
= x 4 x 4 x 360 1
7
88
= cm2 1
7
OR
22 352
(iii) Area of circle = r2 = x4x4= cm2 1
7 7
352 96
Required area = 64 – = cm2 1
7 7
------------------------------------------------------------------------------------------------------------------
16
430/6/1
Solution (i) Let fixed charge= ₹ x and charges per km = ₹ y
1
x + 10y = 105, x + 15y = 155
2
On solving, x = 5
1
⸫Fixed charge = ₹ 5 2
(ii) on solving, we get y =10
Charge per km = ₹10 1
(iii) x + 10y = 20 + 10(10) = ₹ 120 1+1
OR
(iii) Required amount = x + 10y + x + 25y = 2x + 35y 1
= 2(5) + 35(10) = 10 + 350 = ₹ 360 1
------------------------------------------------------------------------------------------------------------------
17
430/6/1
Solution (i) AC = AB + BC = 20 + 20 = 40 m 1
(ii) Shortest distance OB = √252 − 202 = 15 m 1
660
(iii) Circumference = 2(15) = 30 m or m 1+1
7
OR
2 4950
Area = (15) = 225 sq. m or 7 sq. m 1+1
--------------------------------------------------------------------------------------------------------------
18
430/6/1
MARKING SCHEME
MATHEMATICS (BASIC) 430/6/3
Answer (b) 24 × 73 1
2
3
430/6/3
5
Answer 3
(d) 2 , -1 1
Answer (a) 4 1
7
Answer (b) 5 1
8
Answer (b) x + y = 19 1
9
Answer (a) 0 1
4
430/6/3
10
Answer 77
(c) 2
cm2 1
11
Answer 1
(a) 26 1
13
Answer (d) 4 1
5
430/6/3
14
Answer (d) -2 1
15
Answer 1
(d) 3 1
16
Answer 3
(a) k = 2 1
17
Answer (d) -1 1
18
6
430/6/3
19
Answer (b) Both Assertion (A) and Reason (R) are true, but Reason (R) is not the correct explanation of
Assertion (A). 1
SECTION B
21
Solution 5 1
(i) P (Number divisible by 6) = or 1
30 6
5 1
(ii) P (Number greater than 25) = 30
or 6
1
22
7
430/6/3
22
=5(√2) -3(1)2+5(1)
2 1
12
1
= 12 2
24
Solution . Figure ½
OQ = √252 – 242 1
OQ = 7 cm ½
25
Solution 1
(a) Sum of zeroes=6+ (–3) = 3 2
1
Product of zeroes = 6(– 3) = – 18 2
8
430/6/3
𝑝−7𝑞
Clearly 4𝑞
is rational but √5 is irrational
Our assumption is wrong 7 +4√5 is irrational. 1
27
Solution 𝑥–2–x
=3 1
𝑥(𝑥 – 2)
3x2 – 6x + 2 = 0 1
6 ± 2√3 3 ± √3
x= or 1
6 3
28
𝑐𝑜𝑠 𝐴
Solution 𝑐𝑜𝑡 A – cos A – cos A
sin A
(a)LHS = 𝑐𝑜𝑡 𝐴 + cos A = 𝑐𝑜𝑠 𝐴 ½
+ cos A
sin A
1−𝑠𝑖𝑛 𝐴
= 1+𝑠𝑖𝑛 𝐴 1
(1 – sin A)(1 + sin A)
= (1 + sin A)(1 + sin A) 1
1 – sin2 𝐴 𝑐𝑜𝑠2 𝐴
= (1 + sin A)2 = (1 + sin A)2 ½
OR
28
Solution (b) LHS = (sec + tan ) (1 – sin )
1 sin 𝜃
= (𝑐𝑜𝑠 𝜃 + cos 𝜃) (1 – sin ) 1
1 + sin 𝜃 (1 – sin2 𝜃)
=( ) (1 – sin ) = ½+½
cos 𝜃 cos 𝜃
𝑐𝑜𝑠2 𝜃
= = cos = RHS 1
cos 𝜃
29
Solution
(a)ABCD is a parallelogram (1 for figure)
To prove: ABE CFB
In ABE and CFB,
A = C (opp. angles of parallelogram) ½
AEB = CBF (alt. int. angles ) ½
⸫ ABE CFB (AA similarity) 1
OR
9
430/6/3
29 (b)
Solution
(5, 3) (4, 5)
10
430/6/3
Let C divides AB in the ratio 1 : 2 ½
1 ×4 + 2 × 5 1 × 5 + 2 × 3 14 11
⸫ C( 1 + 2 , 1 + 2 ), i.e., C ( 3 , 3 ) 1
Let D divides AB in the ratio 2 : 1 ½
2×4+1×5 2×5+1×3 13 13
⸫ D( 2 + 1 , 2 + 1 ), i.e., D( 3 , 3 ) 1
SECTION D
32
Solution (a)
1 for figure
1
Given In ABC, DE // BC 2
𝐴𝐷 𝐴𝐸 1
To prove : =
𝐷𝐵 𝐸𝐶 2
1
Const.: Join BE, CD. Draw DM ⊥ AC and EN ⊥ AB 2
1
𝑎𝑟(𝛥 ADE) × AD × EN
𝐴𝐷
Proof : 𝑎𝑟(𝛥 BDE) = 21 = _________ (i) 1
× DB × EN 𝐷𝐵
2
𝑎𝑟(𝛥 ADE) 𝐴𝐸 1
similarly = _________ (ii)
𝑎𝑟(𝛥 CDE) 𝐸𝐶 2
BDE and CDE are on the same base DE and between the same parallel lines BC and DE.
1
ar( ADE) = ar( CDE) _________ (iii) 2
𝐴𝐷 𝐴𝐸 1
From (i), (ii) and (iii) 𝐷𝐵
= 𝐸𝐶 2
OR
11
430/6/3
𝑄𝑆 𝑃𝑅 𝑄𝑆 𝑃𝑄
𝑄𝑅 = 𝑄𝑇 𝑄𝑅 = 𝑄𝑇 (as PR = PQ) _________ (i) 2
In PQS and TQR,
Q = Q (common)
𝑄𝑆 𝑃𝑄 1
= (from (i)) 12
𝑄𝑅 𝑄𝑇
1
⸫ PQS TQR (SAS similarity) 2
33
Solution (a)
fig. 1
3
In APQ, tan 30 = 𝐴𝑄 1
1 3
= 𝐴𝑄 AQ = 3√3 ½
√3
3
In PBQ, tan 45 = 𝐵𝑄 1
BQ = 3 ½
⸫ AB = AQ + BQ = 3√3 + 3 ½
= 3(1·73 + 1) = 8·19 ½
Width of river = 8·19 m
OR
fig. 1
20
In ABD, tan 45 = 1
𝑥
x = 20 ½
12
430/6/3
20 + h
In ACD, tan 60 = 1
𝑥
√3x = 20 + h ½
⸫ h = 20 (√3 - 1) m ½
h = 14.6 m ½
34
Solution a4 + a8 = 24, a + 3d + a + 7d = 24 1
1
2a + 10d = 24 or a + 5d = 12 ________ (i)
2
a6 + a10 = 44 a + 5d + a + 9d = 44
2a + 14d = 44 or a + 7d = 22 ________ (ii) 1
1 1
Solving (i) and (ii), d = 5, a = – 13 +
2 2
1
⸫ AP is – 13, – 8, – 3, 2, 7, ........ 2
25
S25 = [2a + 24d]
2
25 1
= 2 [– 26 + 120] 2
1
= 1175 2
35
Solution Total surface area of the article = CSA of cylinder + CSA of 2 hemispheres 1
= 2rh + 2(2r2)
22 7 22 7 7
= 2× 7
×2 × 10 + 4 × 7
×2×2 1+1
22 7 7
=2 × 7 ×2 (10 + 2 × 2
) 1
= 22(10 + 7)
= 22 × 17 = 374 cm2 1
13
430/6/3
36
Solution (i) AC = AB + BC = 20 + 20 = 40 m 1
(ii) Shortest distance OB = √252 − 202= 15 m 1
𝟔𝟔𝟎
(iii) Circumference = 2(15) = 30 m or 𝟕 m 1+1
(OR)
2 𝟒𝟗𝟓𝟎
(iii) Area = (15) = 225 sq. m or sq. m 1+1
𝟕
14
430/6/3
37
Solution 2
(i) Area of square ABCD = (8) = 64 cm2 1
(ii) AC = √(8)2 + (8)2 =√128 = 8√2 cm 1
(iii) We know that diagonals of square bisect each other at 90
AOB = 90
𝜋𝑟 2 𝜃
Area of sector OPRQ = 360°
22 90
= 7 x4x4 x 360 1
88
= 7 cm2 1
15
430/6/3
OR
22 352
(iii) Area of circle = r2 = x 4 x 4= cm2 1
7 7
352 96
Required area = 64 – = cm2 1
7 7
38
16
430/6/3
(ii) on solving, we get y =10
Charge per km = ₹10 1
17
430/6/3
MARKING SCHEME
MATHEMATICS (BASIC) 430/6/3
Answer (b) 24 × 73 1
2
3
430/6/3
5
Answer 3
(d) 2 , -1 1
Answer (a) 4 1
7
Answer (b) 5 1
8
Answer (b) x + y = 19 1
9
Answer (a) 0 1
4
430/6/3
10
Answer 77
(c) 2
cm2 1
11
Answer 1
(a) 26 1
13
Answer (d) 4 1
5
430/6/3
14
Answer (d) -2 1
15
Answer 1
(d) 3 1
16
Answer 3
(a) k = 2 1
17
Answer (d) -1 1
18
6
430/6/3
19
Answer (b) Both Assertion (A) and Reason (R) are true, but Reason (R) is not the correct explanation of
Assertion (A). 1
SECTION B
21
Solution 5 1
(i) P (Number divisible by 6) = or 1
30 6
5 1
(ii) P (Number greater than 25) = 30
or 6
1
22
7
430/6/3
22
=5(√2) -3(1)2+5(1)
2 1
12
1
= 12 2
24
Solution . Figure ½
OQ = √252 – 242 1
OQ = 7 cm ½
25
Solution 1
(a) Sum of zeroes=6+ (–3) = 3 2
1
Product of zeroes = 6(– 3) = – 18 2
8
430/6/3
𝑝−7𝑞
Clearly 4𝑞
is rational but √5 is irrational
Our assumption is wrong 7 +4√5 is irrational. 1
27
Solution 𝑥–2–x
=3 1
𝑥(𝑥 – 2)
3x2 – 6x + 2 = 0 1
6 ± 2√3 3 ± √3
x= or 1
6 3
28
𝑐𝑜𝑠 𝐴
Solution 𝑐𝑜𝑡 A – cos A – cos A
sin A
(a)LHS = 𝑐𝑜𝑡 𝐴 + cos A = 𝑐𝑜𝑠 𝐴 ½
+ cos A
sin A
1−𝑠𝑖𝑛 𝐴
= 1+𝑠𝑖𝑛 𝐴 1
(1 – sin A)(1 + sin A)
= (1 + sin A)(1 + sin A) 1
1 – sin2 𝐴 𝑐𝑜𝑠2 𝐴
= (1 + sin A)2 = (1 + sin A)2 ½
OR
28
Solution (b) LHS = (sec + tan ) (1 – sin )
1 sin 𝜃
= (𝑐𝑜𝑠 𝜃 + cos 𝜃) (1 – sin ) 1
1 + sin 𝜃 (1 – sin2 𝜃)
=( ) (1 – sin ) = ½+½
cos 𝜃 cos 𝜃
𝑐𝑜𝑠2 𝜃
= = cos = RHS 1
cos 𝜃
29
Solution
(a)ABCD is a parallelogram (1 for figure)
To prove: ABE CFB
In ABE and CFB,
A = C (opp. angles of parallelogram) ½
AEB = CBF (alt. int. angles ) ½
⸫ ABE CFB (AA similarity) 1
OR
9
430/6/3
29 (b)
Solution
(5, 3) (4, 5)
10
430/6/3
Let C divides AB in the ratio 1 : 2 ½
1 ×4 + 2 × 5 1 × 5 + 2 × 3 14 11
⸫ C( 1 + 2 , 1 + 2 ), i.e., C ( 3 , 3 ) 1
Let D divides AB in the ratio 2 : 1 ½
2×4+1×5 2×5+1×3 13 13
⸫ D( 2 + 1 , 2 + 1 ), i.e., D( 3 , 3 ) 1
SECTION D
32
Solution (a)
1 for figure
1
Given In ABC, DE // BC 2
𝐴𝐷 𝐴𝐸 1
To prove : =
𝐷𝐵 𝐸𝐶 2
1
Const.: Join BE, CD. Draw DM ⊥ AC and EN ⊥ AB 2
1
𝑎𝑟(𝛥 ADE) × AD × EN
𝐴𝐷
Proof : 𝑎𝑟(𝛥 BDE) = 21 = _________ (i) 1
× DB × EN 𝐷𝐵
2
𝑎𝑟(𝛥 ADE) 𝐴𝐸 1
similarly = _________ (ii)
𝑎𝑟(𝛥 CDE) 𝐸𝐶 2
BDE and CDE are on the same base DE and between the same parallel lines BC and DE.
1
ar( ADE) = ar( CDE) _________ (iii) 2
𝐴𝐷 𝐴𝐸 1
From (i), (ii) and (iii) 𝐷𝐵
= 𝐸𝐶 2
OR
11
430/6/3
𝑄𝑆 𝑃𝑅 𝑄𝑆 𝑃𝑄
𝑄𝑅 = 𝑄𝑇 𝑄𝑅 = 𝑄𝑇 (as PR = PQ) _________ (i) 2
In PQS and TQR,
Q = Q (common)
𝑄𝑆 𝑃𝑄 1
= (from (i)) 12
𝑄𝑅 𝑄𝑇
1
⸫ PQS TQR (SAS similarity) 2
33
Solution (a)
fig. 1
3
In APQ, tan 30 = 𝐴𝑄 1
1 3
= 𝐴𝑄 AQ = 3√3 ½
√3
3
In PBQ, tan 45 = 𝐵𝑄 1
BQ = 3 ½
⸫ AB = AQ + BQ = 3√3 + 3 ½
= 3(1·73 + 1) = 8·19 ½
Width of river = 8·19 m
OR
fig. 1
20
In ABD, tan 45 = 1
𝑥
x = 20 ½
12
430/6/3
20 + h
In ACD, tan 60 = 1
𝑥
√3x = 20 + h ½
⸫ h = 20 (√3 - 1) m ½
h = 14.6 m ½
34
Solution a4 + a8 = 24, a + 3d + a + 7d = 24 1
1
2a + 10d = 24 or a + 5d = 12 ________ (i)
2
a6 + a10 = 44 a + 5d + a + 9d = 44
2a + 14d = 44 or a + 7d = 22 ________ (ii) 1
1 1
Solving (i) and (ii), d = 5, a = – 13 +
2 2
1
⸫ AP is – 13, – 8, – 3, 2, 7, ........ 2
25
S25 = [2a + 24d]
2
25 1
= 2 [– 26 + 120] 2
1
= 1175 2
35
Solution Total surface area of the article = CSA of cylinder + CSA of 2 hemispheres 1
= 2rh + 2(2r2)
22 7 22 7 7
= 2× 7
×2 × 10 + 4 × 7
×2×2 1+1
22 7 7
=2 × 7 ×2 (10 + 2 × 2
) 1
= 22(10 + 7)
= 22 × 17 = 374 cm2 1
13
430/6/3
36
Solution (i) AC = AB + BC = 20 + 20 = 40 m 1
(ii) Shortest distance OB = √252 − 202= 15 m 1
𝟔𝟔𝟎
(iii) Circumference = 2(15) = 30 m or 𝟕 m 1+1
(OR)
2 𝟒𝟗𝟓𝟎
(iii) Area = (15) = 225 sq. m or sq. m 1+1
𝟕
14
430/6/3
37
Solution 2
(i) Area of square ABCD = (8) = 64 cm2 1
(ii) AC = √(8)2 + (8)2 =√128 = 8√2 cm 1
(iii) We know that diagonals of square bisect each other at 90
AOB = 90
𝜋𝑟 2 𝜃
Area of sector OPRQ = 360°
22 90
= 7 x4x4 x 360 1
88
= 7 cm2 1
15
430/6/3
OR
22 352
(iii) Area of circle = r2 = x 4 x 4= cm2 1
7 7
352 96
Required area = 64 – = cm2 1
7 7
38
16
430/6/3
(ii) on solving, we get y =10
Charge per km = ₹10 1
17
430/6/3